PT 365 questions

¡Supera tus tareas y exámenes ahora con Quizwiz!

What is Cushing's syndrome?

1. (hypercortisolism) 2. Caused from excess pituitary gland ACTH production 3. Weight gain 4. Buffalo Hump- fat accumulation in the upper back 5. Moon face- fat accumulation in the face 6. Kyphosis and easy bruising 7. hypokalemia

levels of dependence, adaptation and independence with SCI

C1-4: dependent C5-6: adaptations/equipment needed C7 on: independence with most C8: independence with ramps and uneven transfers

Is electrical stimulation allowed if a patient has a pacemaker?

No. All applications of electrical stimulation are contraindicated in the presence of a pacemaker.

What is turf toe?

Sprain of the big toe due to hyper extension, such as when pushing off into a sprint.

The hypothalamus regulates which of the following bodily functions via the autonomic nervous system and by activation of which branch?

Stimulation of the sympathetic nervous system (SNS): pupil dilation, increases blood pressure, increases respiration rate, increases glucose production, decreases digestion, increases sweating, increases muscle activation (maintenance of fight or flight response)

CHF symptoms

accumulation of fluid in the body tissues resulting in symptoms like -pitting edema, -ascites (fluid collects in spaces within your abdomen) -frothy sputum, -orthopnea (difficulty breathing while laying down)

What is slipped capital femoral epiphysis?

common hip disorder seen in overweight adolescents - occurs when capital femoral epiphysis displaces from the femoral neck - obesity is a big risk factor - can lead to avascular necrosis Are commonly demonstrate a Trendelenburg gait and limited range of motion in abduction, flexion, and internal rotation.

CV effects with aging

decreased HR, SV, CO increased BP vital capacity decreases glucose tolerance decreases

don and doff order of items

don: gown, mask, goggles, gloves doff: gloves, goggles, gown, mask

CV effects of aging

everything decreases decreased HR, BP, CO, SV

hypercapnia

excessive carbon dioxide in the blood

What are the pelvic floor muscles?

levator ani made up of: Iliococcygeus, pubococcygeus, puborectalis

common metastasis sites from breast cancer

lungs, bones, CNS, liver

what is the strongest type of evidence?

meta-analysis/ systematic review

what is an autonomous bladder?

non-reflex/flaccid needs manual pressure to squeeze the bladder and void or catheterization

what does a normal lymph node feel like?

soft, mobile, and less than 1cm in size abnormal can be: firm to hard mobile or nonmobile tender or nontender

which vena cava syndrome is emergent in cancer?

superior vena cava also: Sudden loss of limb function Spinal cord compression Fever in immune-compromised patients

BMI levels

underweight: <18.5 normal: 18.5-24.9 overweight: 25-29.9 obese: >30

what are x rays good at seeing?

very good a looking at bones, however they don't see cartilage, muscles, tendons, or ligaments (soft tissue) very well.

Metabolic syndrome criteria

Waist circumference >40 inches in men and >35 in women, triglycerides >150, HDL <40 in men and <50 in women( fasting plasma glucose >110.

Lumbar traction body weight percentage parameters

disc protrusion is 25 percent BWT in prone. joint distraction is 50 percent BWT in prone

What is Paget's disease?

disordered skeletal remodeling in focal areas of the bone. will cause increased skin temperature over the affected long bones as well as deep, boring pain. Areas most commonly involved are the pelvis, skull, femur, tibia, spine, shoulders, and ribs.

What prosthetic causes can cause circumduction of gate?

Long prosthesis, locked knee unit, and adequate knee flexion, inadequate suspension, small or loose socket, plantar flexed foot, increased knee flexion resistance.

A patient with subluxation of the shoulder presents to the PT clinic. During evaluation, the therapist notices atrophy of the flexor surface of the upper arm along with weakness of elbow flexion with a supinated forearm. Which of the following findings will MOST LIKELY be associated with this finding? Loss of 2-point discrimination on the lateral aspect of the upper arm Loss of 2-point discrimination on the lateral aspect of the forearm Loss of 2-point discrimination on the posterior aspect of the arm Loss of 2-point discrimination on the posterior aspect of the forearm

Loss of 2-point discrimination on the lateral aspect of the forearm With an injury to the musculocutaneous nerve, the patient is unable to flex the elbow with the forearm supinated and may have some instability in the shoulder with atrophy of the flexor surface of the upper arm. The sensory supply of the musculocutaneous nerve is to the lateral aspect of the forearm.

lab findings in kidney failure

-elevated BUN and urea levels -increased creatinine (waste product that comes from the normal wear and tear on muscles of the body.) -decreased urine output

Chains with spinal stenosis typically have which type of low back direction bias?

flexion bias

should you use fascilitation with DMD patients?

no, this will overwork and tire muscles. just work on prevention. prevention of contractures, etc. let them to be able to move as best they csn, however they can.

position of the wrist for power grip

slight extension and ulnar deviation

What are the Canadian C-spine rules?

X-ray if traumatic and greater than or equal to 65 years old Dangerous mechanism of injury such as a fall from an elevation of 3 feet or greater or 3 to 5 stairs, or an axial load to the head or with rollover or ejection from car

what is gout?

a disease in which defective metabolism of uric acid causes arthritis, especially in the smaller bones of the feet, deposition of chalkstones, and episodes of acute pain. Common findings include pain, redness and swelling of the first metatarsal joint.

limited ROM with hip avascular necrosis of the femoral head (Legg-calves-perthes) vs SCFE

-LCP: EXABIR decreased. also pain at inner side of knee -SCFE: "FABIR" (F flexion, AB abduction, IR internal rotation) all have a decreased ROM at the hip with avascular necrosis -pain in the groin and/or medial thigh in both diagnoses, and tenderness with palpation at the hip joint

Common signs of rheumatoid arthritis

-swelling in the distal joints of arms and legs -redness -takes about an hour to get out of bed in the morning -severe pain and stiffness -rheumatoid nodules -hammer toe (shortened extensor tendon) -Volar subluxation of the MCP joints along with Ulnar drift

Reflex testing scores

0 = no response; always abnormal. 1+ = a slight but definitely present response; may or may not be normal. 2+ = a brisk response; normal. 3+ = a very brisk response; may or may not be normal. 4+ = a tap elicits a repeating reflex (clonus); always abnormal.

CRPS 1 vs CRPS 2

1 = no evidence of nerve injury 2 = evidence of nerve injury form of chronic pain and inflammation that usually affects an arm or a leg (unilateral) after an injury are indicative of CRPS -spontaneous pain -changes skin skin color and temp -swelling abnormal sweating -abnormal nail and hair growth -tingling/numb is nerve involved TX: -continuous contact to avoid "make and break" irritation -desensitizing the foot with textures is okay -TENS, US and vibration help pain and edema -DO NOT do painful stretching/manipulations which will exacerbate issue

Salter-Harris Classification of fractures

1- Slipped (S) 2- Sbove (A) 3- beLow (L) 4- Through (T) 5-cRushed (ER) (can arrest normal further growth) Metaphysis Physis Epiphysis

What is the normal time for a PR interval

0.12-0.20 seconds

with cranial nerve does eyelid elevation?

3 - oculomotor

otitis media

infection of the middle ear causing fever and ear pain.

What makes up the unhappy triad in the knee

ACL, MCL, medial meniscus

PCA stroke symptoms

C/L homonymous hemianopsia visual agnosia, prosopagnosia, memory deficits

T waves vs hypo and hyperkalemia

Hypokalemia: inverted T wave Hyperkalemia: Peaked T wave

ICF model vs ICIDH

ICF: Health Condition → Body Function and Structure → Participation → Personal Factors ICIDH: Disease → Impairment → Activity-Disability → Environmental-Personal Factors

Where does the borg scale start and finish

It ranges from 6 to 20 on the scale

What is the most effective form of diagnostic imaging for patients with multiple sclerosis?

MRI

v fib

cant even get a full spike QRS complex

FEV1/FVC value in those with COPD

less than 70%

what cobb angle in scoliosis identifies need for surgery?

over 40 degrees >10 is scoliosis

type 1 vs type 2 diabetes

- Type 1: insulin dependent, pancreas doesnt secrete beta cells. 5-10% of cases; autoimmune, genetic, environ. factors - Type 2: non-insulin dependent (insulin resistant) , 90-95% of cases; older age, tissues dont use insulin or pancreas doesnt make enough. obesity, family hx, prior hx of gestational diabetes, impaired glucose tol., phys. inactivity, race/ethnicity

Ultrasound parameters

Lower numbers of frequency penetrate deeper i.e. 1 MHz penetrates deeply compared to 3 MHz. Higher intensity results in quicker treatment times due to increased rate of heating i.e. 1.5 W/cm2 would have a higher rate of heating compared to .5 W/cm2

Airborne precautions

N-95, gown, gloves private room with - air pressure MTV Measles, TB, Varicella chicken pox, smallpox, SARS, herpes zoster

pinch an inch test

+ on LLQ for diverticulitis + on RLQ for appendicitis

lymphatic system functions

-absorbs fats from digestion/intestines -absorbs proteins from tissues -absorbs extra interstitial fluid -produces antibodies and memory for immune defense

orthopnea vs platypnea

-orthopnea (difficulty breathing while laying down) -platypnea (difficulty breathing in sitting)

What is the necessary frequency electrical stimulation to achieve a smooth muscle contraction?

30 pulses per second

Where is the apical pulse found?

5th intercostal space left midsclavicular line -apex of heart. (directly below L nipple.)

What is a boggy end feel?

A boggy end-feel is produced by fluid or blood in the joint.

complications of NSAIDs

Abdominal pain (GI issues), increased BP (from renal vasoconstriction), peripheral edema, and melena (dark stool from digested blood)

aquatic therapy effects on CV system and respiratory

CV: thing happy relaxing by pool HR decreases BP decreases SV increases CO increases RESP: hard to breathe in against pressure decreased vital capacity increased work of breathing

continuous vs pulsed US

Continuous is used for thermal effects, scar tissue, muscle contracture, adhesions, chronic things > 3 weeks). Pulsed is used for mechanical effects in acute (3 weeks or less) conditions or wound healing

Malingering

Deliberate faking of a physical or psychological disorder motivated by gain.

Using moderate to heavy weights during resistance training is contraindicated for how long after an open heart surgery?

For the first two months. Prior to those weights can be restricted to 5 to 8 pounds.

when youll see hyperglycemia vs hypoglycemia

HYPER: dehydration/diuretics HYPO: addisons (low cortisol)

What is hyperacusis?

Hyperacusis is defined as the collapsed tolerance to normal environmental sounds. increased sensitivity to sound (from facial nerve damage)

What is McBurney's sign?

It is a deep tenderness or pain at McBurney's point, one-third the distance from the right anterior iliac spine and the navel. This is indicative of acute appendicitis.

What is Bakody's sign?

Placing arm on top of head reduces peripheral symptoms in the upper extremity Suggestive of lower cervical radiculopathy AKA shoulder abduction test

Which muscles does myopathy typically impact?

Proximal muscle is greater than distal.

Nodes at the PIP and DIP joints

Seen in OA Bouchards: PIP Herberdens: DIP

What is the antibacterial agent that is most effective for infection control to a full thickness burn with a thick layer of eschar?

Sulfamylon Things like silver nitrate and nitrofurazone act more superficially

What type of drugs are used to lower blood pressure

Thiazide diuretics (decreases blood plasma, so decreased pressure), ACE inhibitor, calcium channel blockers are anti-hypertensive drugs.

Edema stages

1+: 1/4 inch 2+: 1/2 inch 3+: 3/4 inch / 15-30 sec rebound. 4+: 1 inch

Physical therapist determines the pulse pressure for a patient with congestive heart failure. Which of the following blood pressure measurements would represent the highest pulse pressure? 130/60 mmHg 145/100 mmHg 120/75 mmHg 140/115 mmHg

130/60 mmHg -pulse pressure is the difference between systolic and diastolic blood pressure. It represents the force that the heart generates each time it contracts and is measured in arterial stiffness. The normal resting pulse pressure ranges from 30 to 50 mmHg

What is the loose packed position of the elbow

70 degrees flexion, 10 degrees supination

A PT student is studying the effects and characteristics of ultrasound. Which of the following statements regarding therapeutic ultrasound is true? A 40% duty cycle would have an off time of 4msec and an on time of 6msec A 40% duty cycle would have an off time of 6msec and an on time of 4msec A 40% duty cycle would have an off time of 10msec and an on time of 4 msec A 40% duty cycle would have an off time of 4msec and an on time of 10msec

A 40% duty cycle would have an off time of 6msec and an on time of 4msec Duty cycle is the fraction of time the US energy is on over one pulse period (time on + time off). Duty cycle= Duration of pulse/Pulse period x 100 Therefore, a 40% duty cycle would have an on time of 4msec and on off time of 6msec.

technique to help decrease spacticity of muscles

prolonged icing

What is INR value

An increase in INR indicates the blood is thinner than normal. vomiting/dehydration with deplete vitamin K levels, thinning blood. A decrease in INR would cause the blood to become thicker and would increase the risk of patient's developing a clot. INR healthy: .9-1.1 INR warfarin: 2-3.5 (bc warfarin is a blood thinner)

what is ultrasound good for?

Diagnosing tendon tears, soft tissue abnormalities, cartilage; etc.

Physical therapist is treating a five-year-old child with a diagnosis of cerebral palsy involving both of his lower extremities. Which of the following terms best describes this type of cerebral palsy? Mono plegic Diplegic Hemiplegic Quadriplegic

Diplegic -Mono plegic is one limb -Diplegic is lower limbs -hemiplegic involves the upper and lower on one side of the body -Quadriplegic is all four limbs

What could you test via the dorsal column-medial lemniscal pathway?

Discriminative sensations. I.e. two point discrimination, vibration, joint proprioception, monofilament testing.

What is digitalis?

Drug for chronic heart failure that decreases HR and increases strength of contraction

Glascow Coma Scale

EYE OPENING 4 - spontaneous 3 - open to speech 2 - open to pain 1 - no response VERBAL 5 - alert and oriented 4 - disoriented conversation 3 - inappropriate words 2 - nonsensical sounds 1 - no response MOTOR 6 - spontaneous 5 - localizes pain 4 - withdraws to pain 3 - decorticate posturing (flexion response) 2 - decerebrate posturing 1 - no movement score 3-8: severe 9-12: mod 13-15: mild

Explain the different types of validity: face, content, predictive, concurrent.

Face: this indicates that a measure appears to measure what it is intended to measure. It is the weakest form of validity. Content: used to determine if the items that make up an instrument represent all possible content that defines a variable of interest. Predictive: type of criterion validity and is used to determine if an experimental measure can predict a future outcome. Concurrent: type of criterion validity. Used when comparing two measures at the same Time to determine if the experimental measure can be used as a substitute for the reference measure or gold standard.

Describe the different heart blocks

First degree: increase in PR interval lengths with no drop to beats second-degree Mobitz type one wenckeBach: increased PR interval lengths with a dropped QRS beat second-degree Mobitz type two: normal PR interval lengths with a dropped QRS beat third degree: no relationship between P waves and QRS complexes.

Common signs and symptoms of a UTI

Frequent urination, Hematuria, foul odor to urine, painful urination (dysuria), cloudy urine, fever, confusion in the elderly

causes of secondary osteoporosis?

GI diseases; hyperthyroidism; hyperparathyroidism; chronic renal failure; excessive alcohol consumption; use of certain medications such as glucocorticoids

A physical therapist is reading the lab report of a patient admitted in the hospital. In which of the following scenarios would the patient be advised to NOT exercise? Platelet count: 22,000/mm3; Hb: 9g/dL; WBC: 6000/mm3 Hb: 7g/dL; Hematocrit: 23%; WBC: 4800/mm3 Platelet count: 29,000/mm3; Hb: 8g/dL; Hematocrit: 30% Hematocrit: 25%; Platelet count: 20,000/mm3; WBC: 5200/mm3

Hb: 7g/dL; Hematocrit: 23%; WBC: 4800/mm3 Guidelines for exercise based on laboratory values suggest no exercises for the following findings: WBC <5000/mm3; Hematocrit <25%; Hb <8g/dL; Platelets <20,000- AAROM or AROM but no antigravity or resisted exercise.

common drugs

Heparin/Warfarin: anti-coagulants Digoxin: for CHF Lasix: diuretic naproxen: NSAID oxy: opioid - can reduce RR

Physical therapist observes the gate of a child is spastic cerebral palsy. The therapist notes that the child exhibits excessive lordosis during ambulation. Which of the following surgical procedures would best address this postural deformity? Hamstring tendon lengthening Adductor longus tendon lengthening Iliopsoas tendon lengthening Lumbar laminectomy

Iliopsoas tendon lengthening This is the only option that would help decrease the amount of hip flexion and lumbar lordosis being exhibited during ambulation, since the iliopsoas muscle originates from your lower vertebrae and iliac crest and insert at the greater trochanter of your femur, helping with hip flexion.

What is Lyrica or pre-Gabalin and what are its side effects?

It is a medication used to help treat diabetic neuropathy. -Serious side effects include heart failure, greater difficulty walking long distances, and lymphedema.i

L vs R sided MCA stroke

L: affects speech/reception. brocas and wernickes R: perceptual issues. like anosognosia, apraxias, agnosias, unilateral neglect, depth perception, etc.

What is Kehr's sign?

LUQ pain that radiates to the left shoulder when pt is lying supine and legs are elevated.

Droplet precautions

Mask within 3 feet Contact precautions with skin lesions private room - normal air SPIDERMAN strep, sepsis, pneumonia, pertussis, influenza, diptheria, epiglottis, rubella, meningitis, mumps, adenovirus

Physical therapist completes documentation after administering an ultrasound treatment. Which of the following treatment parameters would be the least important to document? Patient position Treatment time Intensity Duty cycle

Patient position. -important things to document following modalities treatment include area of the body treated, duration, frequency, intensity, duty cycle, patient response.

Physical therapist attempt to offer advice to a patient recently diagnosed with DeQuervains tenosynovitis in the right thumb. Which recommendation would likely be the most beneficial? Reduce cell phone use Lower the height of the computer keyboard Wear gloves when exposed to cold Increase dietary iron intake

Reduce cell phone use DeQuervains tenosynovitis results from an inflammatory process involving the tendons in the synovium of the abductor pollicis longus and extensor pollicis brevis muscle at the base of the thumb. The on-site is typically due to repetitive activities involving the sum abduction and extension. Finkelstein's test can be used to diagnosis condition. -telephone used commonly involves repetitive use of the thumb, such as when typing or scrolling.

R.O.M.E. acronym

Resp opp, Met equal respiratory opposite: high pH=low CO2, low pH=high CO2 metabolic equal: high pH=high HCO3, low pH=low HCO3

A physician hypothesizes that delayed union may be the result of disrupted blood supply. Which fraction would be the most consistent with this clinical scenario? Calcaneus fracture Intertrochanteric hip fracture Scaphoid fracture pPoximal humerus fracture

Scaphoid fracture The scaphoid is the largest carpal bone in the proximal row and is commonly injured when falling on an outstretched hand. Localized tenderness in the anatomic snuffbox is the most typical presentation with this type of injury. Scaphoid fractures can be particularly problematic since disruption in the sky for its blood supply can result in avascular necrosis.

UE flexion synergy

Scapular retraction/elevation, shoulder abduction, external rotation, elbow flexion*, forearm supination, wrist and finger flexion*

What are the average numbers for stride length, step length, and cadence?

Stride length: 57 inches (144cm) Step length: 28 inches (72cm) Cadence: 90-120 steps/min

what are tympanic sounds?

hollow, high, drumlike sounds would be with increased air in lungs

Common side effects to diuretics

hypokalemia and hyponatremia so you would maybe add supplements to diet include weakness, nausea, polyuria, hyperglycemia (impaired glucose uptake), muscle cramps, skin rash, increased sensitivity to sunlight (with thiazide diuretics), dizziness, light-headedness, joint pain. -increased PVCs due to hypokalemia (flat or inverted T waves)

ideomotor apraxia vs ideational apraxia

ideomotor= pt cannnot perform the task on command but can do the task when left alone ideational = pt can not perform the task at all

Describe emotional pain

if The adjectives used to describe the patient's pain are subjective in nature, and suggest a variety of origins -Agonizing, nauseating, exhausting, tiring and unbearable are all the words a patient might use to describe pain in emotional terms.

Does increased density in the lungs increase or decrease breath sounds?

increase so emphysema, lots of air, decreases breath sounds

What is SICK syndrome?

"SICK" Scapula Syndrome refers to Scapular malpositioning Inferior medial border prominence Coracoid pain and malposition dysKinesis of scapular movement. It's an overuse Syndrome.

What is Klumpke's paralysis?

(C8-T1) lower brachial plexus injury, results from sudden upward pull of shoulder leads to paralysis of the intrinsic muscle of the hand (small muscles) which leads to claw hand and often, a sympathetic palsy (Horners Syndrome) if the T1 root is involved

When to stop exercise looking at EKG strip

-3 or more PVCs in a row (emergency!) -7 or more PVCs in a minute (emergency!) -multifocal PVCs -PVCs increase with activity

What does APGAR stand for? When do you use it? How do you score it?

-Appearance, Pulse pressure, Grimace, Activity of movement, Respirations -Within 5 minutes of birth -Score 0-10, 10 is highest 7 or more is normal

GOLD classification of COPD

1 - Mild: FEV1 > 80% 2 - Moderate: FEV1 between 50% and 80% 3 - Severe: FEV1 between 30% and 50% 4 - Very severe: FEV1 < 30%f

Normal respiratory rates for adults, elementary age, toddlers and infants.

1.A normal range for an adult (age 18 years and older) is 12 to 20 breaths/minute. 2. A normal range for a child in elementary school (age 6-12 years) is 18 to 30 breaths/minute. 3. A normal range for a toddler (age 1-3 years) is 24 to 40 breaths/minute. 4. A normal range for an infant (age birth to 1 year) is 30 to 60 breaths/minute. (+6 and 10)

A medical record indicates that a child's oral temperature was recorded at 100.6°F. Which value would be most expected if the patient's rectal temperature was taken? 98.6°F 99.6°F 100.6°F 101.6°F

101.6°F -Body temperature represents a balance between the heat produced in the heat loss by the body -the rectal temperature value is typically 0.5-1.0°F higher than the obtained oral temperature value.

AV heart block types

1st degree: increased PR interval with no drops 2nd degree type 2: longer and longer PR before drop 2nd degree type 2: normal PR with drop 3rd degree: no relationship between P and QRS complexes

At what angle would the Physical Therapist ideally expect greatest laxity while performing the knee valgus stress test?

25 degrees of knee flexion

Physical therapist attempt to determine the relative increase in tissue temperature using ultrasound. Which parameters would result in the greatest amount of tissue heating? 1 MHz, .5 W/cm2 1 MHz, 2.0 W/cm2 3 MHz, .5 W/cm2 3 MHz, 2.0 W/cm2

3 MHz, 2.0 W/cm2 -the greatest amount of tissue heating can be achieved with ultrasound by using a high frequency and a high intensity. -however, lower frequency ultrasound has a greater depth of penetration.

ABI ranges

<0.50 Severe arterial disease 0.74-0.50 Moderate arterial disease 0.94-0.75 Mild arterial disease 1.19-0.95 Normal >1.2 Falsely elevated, arterial disease, calcification, diabetes. increments of 25 until 50 or less

A physical therapist decides to perform a caudal glide hip distraction mobilization technique on a patient experiencing hip pain. Which of the following hit positions is the most appropriate to initially perform this technique? 20° of extension, 15° of abduction, and full internal rotation 20° of flexion, 15° of abduction, and slight internal rotation 30° of flexion, 30° of abduction, and slight external rotation Neutral flexion, neutral abduction, and slight external rotation

30° of flexion, 30° of abduction, and slight external rotation This is the loose packed position of the joints and place is the least amount of tension on the joint capsule and supporting ligaments. It is the position typically used for testing joint play and for initially applying distraction.

Physical therapist works with a nine-year-old child with cystic fibrosis. What age best approximates the median age of survival for a patient with cystic fibrosis? 15 years old 25 years old 35 years old 50 years old

35 years old Cystic fibrosis is an inherited disease of the exocrine glands that primarily affects the digestive and respiratory systems. -Great treatments include antibiotics, mucus telling agents, anti-inflammatory meds, Chest physical therapy, nutrition programs. -The median age of survival for a patient with cystic fibrosis is currently in the mid-30s.

A patient has a cerebellar dysfunction that affects both the trunk and the limbs. During examination, which of the following findings will MOST likely be observed? 1. Bradykinesia 2. Festinating gait 3. Impairments in muscle force generation 4. Impairments in the timing of muscle activation

4. Impairments in the timing of muscle activation Cerebellar dysfunction is characterized by delays in initiating and timing of movements. Ataxia is a general term used to describe abnormal coordination of movements. It is demonstrated by deficits in speed, amplitude of displacement, directional accuracy, and force of movement (pp. 565, 567). Ataxic movements are thought to occur due to impairments in the timing and duration of muscle activation.

monofilament grade supplies the MINIMUM amount of force that can be detected by a patient who has protective sensation but not fully normal sensation

5.07 monofilament supplies the least amount of force that can be sensed by patients with only protective sensation intact.

A physical therapist determines that a patient rehabilitating from ankle surgery has consistent difficulty with functional activities that emphasize the frontal plane. Which of the following would be the most difficult for the patient? Anterior lunge 6 inch lateral step down 6 inch posterior step down 8 inch posterior step down

6 inch lateral step down All of the other answers are in the sagittal plane

A physical therapist measures passive forearm pronation and concludes that the results are within normal limits. Which measurement would be classified as within normal limits? 60 degrees 80 100 120

80 degrees Normal form pronation is 0 to 80°. Forearm pronation is measured with the axis of the goniometer placed lateral to the owner styloid process, the proximal arm aligned parallel to the anterior midline of the humerus and the distal arm on the dorsal aspect of the forearm.

A patient rehabilitating from a lower extremity injury has been nonweightbearing for three weeks. A recent physician entry in the medical record indicates the patient is cleared for weight-bearing up to 25 pounds. What is the most appropriate device to use when instructing the patient on the new weight-bearing status? An inclinometer A tape measure An Anthropometer A scale

A scale -inclinometer is a gravity dependent goniometer for measuring joint position -tape measures measure distance -anthropometers gather data on the measurements and proportions of the human body. Commonly measuring long bone length or skin folds

Which type of diagnostic imaging would be used to assess cortical bone, especially looking for fracture?

A CT scan

The treatment plan for a patient who has hemiplegia is based on the theory of reinforcing normal movements are key points of control and avoiding all reflex movement patterns and associated reactions. This theory most closely resembles which of the following treatment approaches? Bobath Kabat Rood Brunnstrom

Bobath The bow bath approach for neurological rehabilitation is often termed neurodevelopmental treatment. The patient learns to control movements or functionality that promote normal movement patterns and integrate function. Emphasis on normal movements and rotational patterns are key components of this therapeutic model.

Which nerve roots innervate the abdominal muscles and back erector spinae?

T7-12 L1-5

contraindications in pregnancy

deep heating, e stim, traction SL exercises excessive hip abd or ext no valsalva No R side lying prolonged supine lying after 1st trimester

Contact precautions

gown and gloves when in contact with patient private room or with cohort patients MRSA, VISA, VRE, c diff, lice, scabies, impetigo, gram negative bacteria, diarrhea, hep-a, hep-b, dermatitis, rota virus MRS WEE

initial increased elevation affects and benefits to training at elevation

increased BP, HR, CO same SV eventually, SV and CO increases

initial increased elevation affects eventual benefit to elevation training

increases in BP, HR, CO. SV stays same CO and SV increases later on

Medial medullary syndrome

ipsilateral paralysis of the tongue due to the involvement of CN 12 and contralateral paralysis of the upper and lower extremities due to effect on the corticospinal tract

v tach

looks like t flutter but flutter they are closer together

Nutation vs pelvis tilt

nutation is sacral movement, which is opposite from ASIS and PSIS movement. looking at the pelvis from a side view: So APT (clockwise) goes with counternutation of sacrum (counterclockwise)

obstructive vs restrictive

ob: difficult to get air out (expiratory values decrease) res: difficult to get air in (all values decrease bc less air in lungs

Nerve innervation for Rhomboids, serratus anterior, deltoids and biceps

rhomboids: dorsal scapular C5 serratus anterior: long thoracic C5-C7 deltoids: axillary C5-C6 biceps: musculotaneous C5-C7

Physical therapist attempt to assess the dynamic balance of an elderly patient. Which screening troll would be the most helpful to test balance and gait? Functional independence measure Tennity performance oriented mobility assessment Fugal Meyer assessment Barthell index

tinetti performance oriented mobility assessment Genetti has two sections. The first section assesses balance and the second section assesses gate. -Hugo Meyer is an ordinal scale used to measure recovery post CVA -functional independence measure as a tool using rehabilitation hospitals to determine a patient's level of disability and burden of care -barthol index is designed to measure the amount of assistance needed to perform 10 different ADLs and mobility activities.

What are the upward rotators of the scapula?

upper trap, lower trap, serratus anterior (upset Thor smacks Loki)

post meniscus repair caution

-not perform knee flexion beyond 45 degrees in the first four weeks of rehabilitation -bike seat too low or forward will increase knee flexion

What are key characteristics of cataracts vs glaucoma?

Cataracs: Clouding of the lens, gradual loss of vision, central vision is lost before peripheral. Glaucoma: loss of peripheral vision first and then central vision.

Physical therapist discuss his positioning issues with the nursing staff prior to a scheduled treatment session. What bony prominence would be particularly susceptible to break down with the patient position in a supine position? Anterior shoulder Spinous process Patella Anterior superior iliac spine

Spinous processes -no need to explain this one

T score for osteoporosis and motions to avoid

T score of -2.5 or lower avoid trunk flexion and rotation activities

examples of UMN and LMN diseases

UMN: cervical myelopathy, TBI, MS, huntingtons, CP, stroke, SCI LMN: guillan-barre, polio, Graves, peripheral nerve injury, DMD, bells palsy, radiculopthy

rep ranges for different muscle needs

endurance: 13+ reps (3 sets) hypertrophy: 8-12 reps (4 sets) strength: under 8 reps (5 sets)

A patient diagnosed with peripheral vascular disease begins supervised treadmill exercise regimen intended to reduce claudication symptoms. After two minutes of ambulation, the patient reports the onset of claudication symptoms rated as a 3/10 on a pain scale. What is a physical therapist most appropriate response? Encourage the patient to continue until the symptoms become more painful Reduce the speed and grade of the treadmill until symptoms resolve Allow the patient to rest, resuming ambulation once symptoms have subsided discontinue use of the treadmill and select an alternative mode of exercise

Encourage the patient to continue until the symptoms become more painful -symptom provoking ambulation programs have been shown to effectively increase the intensity and duration of ambulation of patients with peripheral vascular disease are able to tolerate before the onset of limiting claudication symptoms. -patient engaging in exercise training to decrease claudication symptoms should be encouraged to continue ambulating until symptoms increased to a moderate level of severity. Rest breaks should be initiated before symptoms approach an intolerable intensity or at the patient's request. 3/10 on a pain scale is relatively mild, and in the absence of other symptoms provides ample evidence to continue ambulation

what type of optic lesions cause different visual disturbances?

Homonymous hemianopia: occipital lobe or optic tract Heteronymous (bilateral) hemianopia: optic chiasm complete blindness: optic nerve

A physical therapist works with a patient recovering from a surgical resection of a portion of the gastrointestinal tract. Which form of medication administration would likely be the most problematic for the patient based on the surgical procedure? Injection Intravenous Oral Transdermal

Oral since oral is an enteral form of medication administration, it involves the esophagus, stomach and small and large intestines, moving through the gastrointestinal system. Since the surgery involves the gastrointestinal tract, this form of medication would be avoided -injection, intravenous, and transdermal are all parenteral (non-oral) forms of medication administration

A physical therapist identifies a bluish discoloration of the skin and nail beds of a patient referred to physical therapy for pulmonary rehabilitation. What does this objective findings indicate? Hyperoxemia Hyperoxia Hypokalemia Hypoxemia

Hypoxemia -hyperoxemia refers to an increased oxygen concentration in the blood measured by arterial oxygen partial pressure PAO two values. A PaO2 greater than 120 mmHg. -hyperoxia is increased oxygen and tissues and organs -hypo kalemia severe potassium depletion and circulating blood. This is commonly manifested by episodes of muscular weakness or paralysis and postural hypotension. -hypoxemia refers to decreased oxygen concentration in the blood measured by a PaO2 less than 80 mmHg. Individuals whose blood is deficient in oxygen tend to have a bluish discoloration of the skin called cyanosis. Most noticeable in mucous membranes and nail beds

A patient uses a self administered assessment tool as a method to record daily progress. What type of reliability would be the most essential using this tool? Reliability of parallel forms Internal consistency Intratester Intertester

Intratester -reliability of parallel forms refers to consistency between results of two tests constructed in the same way from the same continent domain. There is only one form of the assessment tool in this example. -Internal consistency refers to the consistency of results across items within a test. The reliability is evaluated by estimating how well the different items within the test, which are supposed to reflect the same construct, do yield similar results. Not of concern in example. -intratester reliability refers to the extent to which scores on the tool obtained by the same tester are consistent. This form of reliability ensures that changes recorded can be attributed to progress in therapy and not to unreliable measurement. -intertester reliability refers to the extent of agreement of the scores recorded by two or more individuals. This address is the consistency of the implementation of a rating system. This example only has one individual.

causes of respiratory/metabolic acidosis/alkalosis

-Metabolic Alkalosis can be caused by GERD and emesis. Signs and symptoms include decreased respiration, numb digits, tetany, convulsions and cardiac arrhythmias. -Respiratory Acidosis can be caused by chest trauma, left congestive heart failure (pulmonary edema develops, carbon dioxide retention occurs), airway obstruction and COPD. Signs and symptoms include visual disturbances, confusion, dizziness, decreased deep tendon reflexes and ventricular fibrillation. -Metabolic acidosis can be caused by diabetic ketoacidosis, renal failure, shock, salicylate overdose and sepsis. Signs and symptoms include headache, mental dullness, kussmaul breathing, stupor and cardiac arrhythmia. -Respiratory Alkalosis can be caused by high altitude, pregnancy, fevers, hypoxia or increased tidal volume in vented patients. Signs and symptoms include lightheadedness, numb digits, tetany, convulsions and cardiac arrhythmias.

Standard wheelchair measurements

-the door width 32in. -Ramp width 36 in -Ramp slope 1:12 (1 inch rise has 12 inches length) Other guidelines for wheelchair accessibility include: Open floor plan •Wheelchair access to bathroom •Toilet seat height same as wheelchair seat height •Adequate clearance under sinks •Insulated pipes •Roll-in shower •Ramp landings every 30 ft (9.1 m) •No thresholds through doorways • Lever-type door handles.

A physician is assessing how well a baby is doing immediately after birth and how well the newborn is doing outside the womb. Which of the following findings indicate the BEST result? -Crying baby, HR<100 bpm, some muscle tone, grimacing present, pink body with blue extremities -Entire body is pink, active motion of extremities, grimacing present, HR <100, Irregular and slow respiration -Crying baby, HR< 100 bpm, pink body, flexed arms and legs, grimacing present and a cough -Crying Baby, HR >100 bpm, pink body with blue extremities, prompt response to stimulus, active movements of extremities

Crying Baby, HR >100 bpm, pink body with blue extremities, prompt response to stimulus, active movements of extremities Option A- 2+1+1+1+1=6/10 Option B-2+2+1+1+1=7/10 Option C: 2+1+2+1+2=8/10 Option D: 2+2+1+2+2=9/10

A physical therapist treats a 16-year-old soccer player in an outpatient clinic. The patient presents with diffuse lateral knee pain that increases with running and climbing stairs. Ober's test is positive. Which diagnosis is most likely based off of the findings? IT band syndrome Prepatellar bursitis Patellar tendinitis Posterior cruciate ligament tear

IT band syndrome -Obers test is used to test the flexibility of the IT band and a TFL with the patient in sidelying. A positive test indicates tight IT band or TFL. -IT band syndrome is caused by repetitive stress and it's normally found in Long distance Runner's Or anyone who participate in activities that require repetitive knee flexion and extension, such a soccer. -Results from friction of the IT band as it Slides over the proximal lateral femoral condyle.

pineal gland vs pituitary gland vs adrenal gland

PINEAL: secretes melatonin and regulates sleep-wake cycle PITUITARY: secretes things like HGH, TSH, ATCH, prolactin, LH and FSH. regulates metabolism, growth, sexual maturation, reproduction, blood pressure and many other vital physical functions and processes. ADRENAL: secretes cortisol, aldosterone (BP) and adrenaline. regulates metabolism, blood pressure and your body's response to stress.

Physical therapist examines the integrity of the L4 spinal level on a patient. What deep tendon reflexes are the therapist used to assess the L4 spinal level? Lateral hamstrings Medial hamstrings Patellar Achilles

Patellar -the patellar reflex is innervated at L3 - L4 spinal level -lateral hamstrings reflex is innervated at S1 - S2 -medial hamstring reflex is innervated at the L5 - S1 -Achilles reflex is innervated by the S1 - S2 spinal level

A patient complains of an insidious onset heel pain that is most painful when initially weight-bearing after period of inactivity. The patient demonstrates decreased dorsiflexion and tenderness over the medial calcaneal tubercle what is the patient most likely diagnosis? Plantar fasciitis Achilles tendinitis Tarsal tunnel syndrome calcaneal stress fracture

Plantar fasciitis They typically presents insidiously with pain originated at the hill. Symptoms are usually exaggerated with initial weight-bearing after period of rest and increases with activity. Decreased dorsiflexion, pain with palpation over the medial calcaneal tubercle and an antalgic gait are common findings

ECG findings that should reduce exercise intensity, stop exercise all together, and those that are emergent

REDUCE INTENSITY: 1 PVC, a flutter 2nd degree type 1 heart block STOP: 2 PVCs in a row, A fib, 2nd degree type 2 heart block EMERGENCY: more than 6 PVCs in a minute, 3 or more PVCs in a row, multifocal PVCs, V fib, myocardial infarction over 1mm elevation, myocardial ischemia over 2mm depression, 3rd degree heart block

A physical therapist works on diaphragmatic breathing with a patient who has a chronic pulmonary disease. The patient has significant weakness of the diaphragm and is hypertensive. Which of the following positions is the most appropriate to initiate treatment? Prone Supine Trendelenburg Semi Fowlers

Semi Fowlers -for patients have difficulty with breathing, you should consider positioning that will help to decrease the weight of the abdominal contents on the diaphragm to make breathing easier. The fact that the patient has hypertension should also be considered when determining which therapeutic positions are appropriate. -supine and prone both would lead to having increased weight of the abdominal contents on the diaphragm. -In Trendelenburg, the patient's head is lower than their feet. This position is used to facilitate drainage from the lower lobes of the lungs into increase blood pressure and hypotensive patients. The position would tend to increase the blood pressure of a patient that is already hypertensive. -semi Fowlers is when the patient is in supine with the head of the bed elevated to 45° and pillows under the knees for support and maintenance of a proper lumbar curve. This position would be recommended to reduce hypertension and facilitate movement of the diaphragm by using gravity to reduce the weight of the abdominal contents on the diaphragm

what is graves disease

The most common cause of hyperthyroidism. It is an autoimmune disease. Common findings include tachycardia, hyperreflexia, decreased muscle strength, weight loss, sensitivity to heat, diarrhea. All of those would be opposite for hypothyroidism

A high school athlete attends an in-service on flexibility training. What is the minimal flexibility training schedule the physical therapist should recommend? One day per week Three days per week Five days per week Seven days per week

Three days per week -flexibility training requires a minimum frequency of 2 to 3 times per week, however, ideally, stretching should occur 5 to 7 days per week.

Physical therapist palpates the bony structures of the wrist and hand. Which of the following structures would be identified in the proximal row of carpals? Capitate Hamate Triquetrum Trapezoid

Triquetrum So Long To Pinky, Here Comes The Thumb. Triquetrum has try in the name, think third carpal.

A patient who is two weeks status post anterior cruciate ligament reconstruction and is toe touch weight-bearing with bilateral axillary crutches reports a new complaint of posterior calf pain on the affected leg. Which additional clinical finding would most likely contribute to a decision to contact the referring physician? Weakness of the ankle dorsiflexors Warmth in the lower leg Reduction in active knee flexion of 5° Pain increase of 3 mm on a visual analogue scale

Warmth in the lower leg -A complaint of posterior calf pain accompanied by warmth in the lower leg may indicate a DVT. This would indicate contacting the referring physician. -a deep vein thrombosis results from the formation of a blood clot, which may become dislodged and is then termed an embolus. This is considered a serious medical condition since the embolus may obstruct a selected artery. -symptoms of a DVT include swelling of the lower extremity, pain, sensitivity over the area of the clot, and warmth in the involved area. -DVT is a potential serious complication following a surgical procedure.

Hydro gel versus calcium alginate dressings

Hydrogel dressings would be recommended for wounds with mild to moderate exudate Calcium absorb large amounts of exudate, up to 20 times our way, inform in jail. This maintains the moist wound environment while maintaining good permeability to oxygen.

things that increase buoyancy (upward force of water)

-COPD (inflated lungs) -osteoporosis (decreased bone density) -obese patients (fat has lower specific gravity) -being fully submerged > only to waist level

pursed lip breathing vs paced breathing

-Paced breathing is indicated when a person becomes dyspneic during an activity. In this, by breaking activities down into component parts and interspersing rest periods between each component, the total activity can be completed without dyspnea or undo fatigue. -Pursed lip breathing is used to decrease respiratory, reduce dyspnea and facilitate relaxation for patients with COPD.

Iontophoresis should be used with precaution for a patient who has which of the following conditions? 1. Peripheral neuropathy 2. Raynaud disease 3. Degenerative joint disease 4. Chronic edema

1. Peripheral neuropathy 1. Iontophoresis should be used with precaution in patients who have conditions involving decreased sensation 2. Raynaud disease is not a precaution or contraindication for iontophoresis 3. Degenerative joint disease may be associated with joint pain and inflammation, both of which could be indications for iontophoresis 4. Chronic edema may be an indication for iontophoresis

what skeletal condition is normal in newborn infant? 1. Pes cavus 2. Genu varum 3. Genu valgum 4. Talipes equinovarus

2. Genu varum pes cavus: high foot arch (like a big cave) talipes equinovarus: "club foot" foot points down and in. "equino" means PF

A physical therapist performs resisted isometric testing on a patient. The patient reports feeling pain during the test, however, strength is normal. Which of the following conclusions regarding this test is the most likely? A severe lesion such as a fracture A minor lesion of a muscle or tendon A complete rupture of a muscle or tendon Intermittent claudication may be present

A minor lesion of a muscle or tendon -Resisted isometric testing (MMT) attempt to identify the status of contractile tissue (muscles, tendons, attachments) answer the nervous tissues supplying the contractile tissue -a fracture would tend to have resisted movement that is classified as weak and painful -a minor lesion of a muscle or tendon would have movement that is strong and painful -complete rupture of a muscle or tendon attendant have movement that is weak and pain-free -intermittent claudication occurs as a result of insufficient blood supply and ischemia and active muscles. Resisted movement would exacerbate pain and cause cramping and muscles distal to the occluded vessel. Strength could be negatively impacted due to severity of pain

A physician instructs the patient to utilize an ED rotation brace for all athletic activities. Which of the following conditions would most warrant the use of this type of brace? Knee OA ACL reconstruction ACL insufficiency PCL reconstruction

ACL insufficiency -the rotation braces are most effective in patients with ligamentous instability, usually involving the ACL and PCL. -reconstruction of a ligament would not allow for a patient to be back in all athletic activities at this time. And the point of surgery is to reduce the functional instability that the rotation brace is used for.

3 tracts that make up the spinothalamic pathway

Anterior (ventral) spinothalamic tract: crude localized touch and pressure Lateral spinothalamic tract: pain and temperature Spinoreticular tract: diffused pain sensations

Difference between knee anterior drawer test and Lachmans

Anterior drawer is done with knee flexed to 90 degrees lachmans is done with knee flexed to 20-30 degrees

Physical therapist observes a patient complete hip abduction and adduction exercises in standing. Which axis of movement is utilized with these particular motions? Frontal Vertical Anterior posterior Longitudinal

Anterior posterior -motions of the body occur around three cardinal planes: Frontal (Coronal), sagittal (lateral), and transverse (axial) -those movements in the cardinal planes occur around three corresponding axes: anterior-posterior, medial-lateral, and vertical. -the movement being described is occurring in the frontal plane but it is swinging around the anterior- posterior axis.

A physical therapist walks with a patient in an inpatient cardiac rehabilitation unit. The patient describes sensation of lower extremity cramping and sensory changes after a distance of 50 feet. The pain subsides following a brief rest period. The therapist hypothesizes that the patient's symptoms are consistent with pain from claudication. Which of the following tests would be the most appropriate to confirm the hypothesis? Venous Doppler ultrasonography Arterial Doppler ultrasonography Clinical electromyography Electrocardiogram

Arterial Doppler ultrasonography -Claudication as leg pain that results from peripheral vascular disease and can originate from arterial or venous insufficiency. -Peripheral arterial disease results and ischemia to the tissues of the lower legs. Since the pain the patient is experiencing subsides with rest, it is likely caused by exercise induced muscle ischemia from arterial insufficiency. -If the patient's pain was caused by venous insufficiency it would likely occur several hours after exercise and relief would not be immediate.

A patient who has cerebral palsy exhibit slow, involuntary, continuous writhing movements of the upper and lower extremities. Which of the following motor impairments is most representative of this type of motor disturbance? Spasticity Ataxia Hypotonia Athetosis

Athetosis Cerebral palsy describes a group of non-progressive movement disorders that result from brain damage. Athetoid cerebral palsy involves damage to the basal ganglia. Athetosis refers to involuntary movements characterize a slow, irregular, and twisting. This type of motor disturbance makes it extremely difficult to maintain a static body position due to peripheral movements having no central stability.

Autonomic dysreflexia vs neurogenic shock vs spinal shock

Autonomic dysreflexia: acute elevation in BP along with bradycardia which usually occurs at T6 or above. headache and anxiety, constricted pupils, spasticity. vasoconstriction from sympathetic activation (dry pale skin) below lesion, parasympathetic response will occur above the level of injury which is responsible for sweating, piloerection, and flushing Neurogenic shock: SCI above T6. A patient with neurogenic shock will present with hypotension, bradycardia, cyanosis, warm-dry extremities, decreased cardiac output, peripheral vasodilation and venous pooling spinal shock: experience a loss of tone, areflexia, loss of sensation below the level of injury as well as marked disturbance of bowel and bladder disturbances

Cervical nerve innervations

C1-C2: SCM (accessory), upper trapezius (accessory), cervical extensors. C3-C4: partial diaphragm (phrenic), scalenes, levator scapulae. C5-C8: Diaphragm (phrenic), pectoralis major (lateral pectoral) and minor (medial pectoral), serratus anterior (long thoracic), rhomboids (dorsal sapular C5), latissimus dorsi (thoracodorsal). T1-T5: some intercostals, erector spinae. T6-T10: Intercostals and abdominals.

pelvis/low back/sacrum pain referrals

CAP Colon Appendix Pelvic viscera

Respiratory conditions

COPD: productive cough for 3 or more months every year for at least 3 years or more. COPD causes narrowing of the airways and hyperinflation of the lungs, which traps air inside causing an increase in the RV, thereby increasing the Total Lung Capacity. FEV1/FVC is also decreased due to narrowing and collapse of airways on expiration. will have decreased expiratory portions. and increased residual volumes/air in lungs bc difficult to get it out

A physical therapist evaluate the patient using a modified Ashworth scale and documents a rating of three. Which of the following conditions would most likely result in a patient exhibiting a rating of three on the scale? Cerebral palsy Down syndrome Duchenne muscular dystrophy Guillain-barre syndrome

Cerebral palsy -the modified Ashworth scale is widely used qualitative scale for the assessment of spasticity and measures the amount of resistance to passive stretch. -spasticity is a common clinical manifestation of an upper motor neuron lesion MAS ranges from zero (no increase in muscle tone) to four (affected limb rigid and flexion or extension). -rating of three is consistent with a considerable increase in muscle tone and difficulty with passive movement. -Down syndrome would have hypotonia rather than spasticity -DMD is a lower motor neuron disease and spasticity is not part of this clinical presentation Guillain-barre is also a lower motor neuron disease and spasticity is not part of this clinical presentation

when would you see + murphys sign and what are the symptoms of this diagnosis

Cholecystitis refers to an inflammation of the gallbladder. Symptoms include right upper abdominal pain which can refer to right upper back, nausea, vomiting, and occasionally fever. increases with eating fatty meals. Murphy's sign is seen to be positive with painful palpation under the ribs on R side.

A 65 year old patient with diabetic neuropathy presents to the clinic with a complaint of frequent falls especially at night. Which of the following conditions of the Sensory Organization Test is this patient MOST LIKELY to score poorly? Conditions 5 and 6 Conditions 1 and 3 Conditions 2 and 4 Conditions 4 and 6

Conditions 5 and 6 Due to peripheral neuropathy, this patient is more dependent on their vision and vestibular systems. Patients depending on vision become unstable in conditions 2, 3, 5 & 6 where we either close the eyes, or have a conflict between vision and the vestibular system. With conditions 1 and 4, the patient will have the opportunity to rely on the visual system and maintain balance.

other tract pathways

Corticospinal tract: carry movement-related information from the cerebral cortex to the spinal cord. Dorsal column-medial lemniscus pathway: is a sensory pathway of the central nervous system. It conveys sensation of fine touch, vibration, pressure, two-point discrimination and proprioception (position) from the skin and joints.

A physical therapist completes a respiratory assessment on a patient in an acute care hospital. The examination reveals a decrease in the intensity of lung sounds during auscultation. Which of the following scenarios would most likely result in this finding? Decrease in lung tissue density due to emphysema Increase in lung tissue density due to emphysema Decrease in lung tissue density due to pneumonia Increase in lung tissue density due to pneumonia

Decrease in lung tissue density due to emphysema -an increase in lung tissue density causes increased sound transmission, whereas a decrease in lung tissue density causes decreased sound transmission. -emphysema would cause a decrease in lung tissue density. Pneumonia causes consolidation, which is characterized by an area of lung tissue that is filled with liquid instead of air. This would cause an increase in long tissue density and an increase in lung sounds.

You should ride the stationary bike at 50 revolutions per minute following the surgery. The physical therapist would like to provide some more of a stretch of the patient's knee while cycling to facilitate flexion range of motion. Which of the following modifications to the cycling parameters would be the most appropriate way to achieve the physical therapist's objective? Decrease the seat height and increase the solutions per minute Increase the seat height and increase the revolutions per minute Decrease the seat height and decrease the revolutions per minute Increase the seat height and decrease the revolutions per minute.

Decrease the seat height and decrease the revolutions per minute -Decreasing a seat height what caused the knee to come into greater flexion. And decreasing the revolutions per minute would allow the need to maintain more time in the flexed position, causing greater stretch.

School therapist observes the gate of a patient with weakness in her lower extremities following a lengthy inpatient hospitalization. Which gate description would be most likely? Decreased step length unilaterally Decreased step length bilaterally Increased step length unilaterally Increased step length bilaterally

Decreased step length bilaterally -Decreased step length bilaterally produces a more conservative, stable gate and is often seen in patients with generalized weakness falling periods of prolonged immobility

Delirium versus dementia

Delirium is a fluctuating attention state. They demonstrate a fluctuating course with symptoms of confusion that alternate with lucid intervals. Sleep/wake cycles are disrupted and confusion is typically worse at night Dementia can have symptoms such as hallucinations throughout the day, persistent personality changes, depressed level of arousal etc.

A patient being treated for shoulder impingement complains of muscle soreness in the triceps muscles for two days following a physical therapy session. Which of the following exercises would most likely have resulted in the stated complaint? Eccentric bench press with dumbbells Concentric bench press with dumbbells Concentric pectoral fly with resistance bands Eccentric pectoral fly with resistance bands

Eccentric bench press with dumbbells -benchpress would be the exercise here working the triceps. -Eccentric work is the main cause for delayed onset muscle soreness due to it being in the more difficult phase of exercise compared to concentric.

A physical therapist concludes that is necessary to elongate the long head of the triceps brachii as part of the passive range of motion assessment of the elbow. Which positioning in the arm would assist the therapist to meet the stated objective? Elbow extension and shoulder extension Elbow extension and shoulder flexion Elbow flexion and shoulder extension Elbow flexion and shoulder flexion

Elbow flexion and shoulder flexion -This position maximally elongates the triceps. Due to it being a two joint muscle

To palpate the insertion of the supraspinatus tendon, a physical therapist should place a patient's arm in which position?

Extending and medially (internally) rotating the shoulder

A three-year-old child throws frequent timber tantrums, usually contrive to gain attention. The physical therapist, recognizing the child subjective, refuses to acknowledge the action. What best term describes a type of behavior therapy? Aversive conditioning Extinction Operant conditioning Rational emotive imagery

Extinction By refusing to acknowledge the child's tantrums, the physical therapist avoids reinforcing the behavior. As a result, the tantrums may decrease in frequency and eventually disappear. This withholding of reinforcement for previously reinforce behavior well hope to decrease the future probability behavior and is called extinction. -aversive conditioning is behavioral therapy technique which reduces the appeal of behavior by associating behavior with physical or psychological discomfort. The individual is exposed to an unpleasant stimulus while engaging in the targeted behavior. -operant conditioning is learning that takes place when the learner recognizes the connection between behavior and its consequences. -rational emotive imagery is a form of intense mental practice for learning new emotional and physical habits. The behavioral techniques focuses on uncovering irrational beliefs which may lead to unhealthy negative emotions and replacing them with more productive rational alternatives.

Physical therapist uses a special test administered in prone to determine that a patient possesses increased femoral anteversion. Which structure would most likely be palpated during this testing procedure? ASIS Greater trochanter Innominate Pubic tubercle

Greater trochanter -the testing question is called the Craigs test and the patient is in prone with the knee flexed to 90°. The degree of femoral anteversion corresponds to the angle formed by the lower leg with the perpendicular access of the table. Normal anteversion for an adult is 8-15° femoral anteversion corresponds with hip IR

Which one of the following sympathetic cardiovascular changes occurs in a patient with long-standing type 1 diabetes mellitus? HR becomes fixed Higher resting HR Lower resting HR Exercise increases HR

HR becomes fixed Long term complication of Type 1 diabetes mellitus is cardiac denervation syndrome which results in a fixed HR that is unresponsive to exercise, stress or sleep.

What is auto splinting?

In pleural pain, pain is alleviated by lying on the affected side (in this case- right side), which diminishes the movement of that side of the chest. This is called auto-splinting.

A patient returns to physical therapy after completing medical testing using Doppler ultrasonography. Which medical condition would most likely necessitate this type of diagnostic imaging? Peripheral neuropathy Intermittent claudication Cardiac arrhythmias Bursitis

Intermittent claudication Doppler ultrasonography is a noninvasive instrument used to assess arterial blood flow. -intermittent claudication is characterized by painful cramping or aching in the lower extremities during physical activity. The condition is associated with impaired blood flow in the lower extremities and thus decreased perfusion to the lower extremity muscles.

What is erb's paralysis and what nerve roots are affected?

It is also known as a brachial plexus injury. It involves the upper trunk C5 to C6 nerves -commonly involves the super scapular, musculocutaneous, and axillary nerves. -the limb is held in a position of adduction, forearm pronation, and wrist and finger flexion (waiters tip position)

What is the Acapella fluid secretion technique?

It is an airway clearance device to help move mucus from the airways. You can position either in sitting upright or in a postural drainage position. It is a mouthpiece and what you exhale as much as possible through the mouthpiece and it vibrates to help move mucus

A physical therapist notes a decrease in passive range of motion in a patient's affected shoulder as compared to their unaffected shoulder. Which of the following would provide the physical therapist with the most valuable information to determine the cause of this discrepancy? Manual muscle testing Joint mobility Sensory testing Active range of motion

Joint mobility If the patient is having decreased passive range of motion, then they're active range of motion and manual muscle testing won't matter since it is not an issue with contractile tissue. The issue could be due to hypomobility of the joint

A patient complains of frequent episodes of tingling on the medial aspect of the calf and ankle especially on forward flexion. On evaluation, the knee jerk is found to be weak along with weakness of tibialis anterior and extensor hallucis muscles and a limited straight leg raise with negative prone knee bend. A disc herniation at which of the following levels would cause these symptoms? L2-L3 L3-L4 L4-L5 L5-S1

L3-L4 L3 would have sluggish knee jerk, a positive prone knee bend and pain on full flexion. Paresthesias would be present on inner knee and anterior lower leg L4 lesions would be associated with weakness of tibialis anterior, extensor hallucis along with paresthesias in the medial aspect of calf and ankle, weak knee jerk and a limited straight leg raise. L5 lesions would be associated with weakness of extensor hallucis, peroneals, gluteus medius, ankle dorsiflexors, hamstrings along with paresthesias on the lateral aspect of leg, medial three toes with decreased ankle jerk and painful crossed straight leg raise. S1 lesions would cause paresthesias along the lateral two toes, lateral foot, lateral leg to knee, plantar aspect of foot along with weakness of the calf and hamstrings, wasting of gluteals, peroneals, plantar flexors and limited straight leg raise. L4 nerve root is affected in this case. At the lumbar level between L3-L4: Nerve root exiting- L3. Nerve affected in stenosis- L3 But with herniation, it is the L4 nerve root that is involved. Because of the course of the nerve root as it exits, the L4 disc (between L4 and L5) only rarely compresses the L4 nerve root; it is more likely to compress the L5 nerve root. (Image attached for better understanding). In this case the symptoms aggravated on forward bending- indicating it was a disc pathology. So for L4 to be affected, the herniation should have taken place at L3-L4 level.

Which joints are primarily affected in late or chronic Lyme's disease?

Large joints of the body, especially the knee A bull's-eye rash is associated with lyme's disease.

In order to unlock the knee from terminal extension in a closed kinetic chain, which of the following must occur?

Lateral (external) rotation of the femur on the tibia

What Knee valgus to the patella?

Laterally displaced the patella.

Which of the following clinical features is a CONTRAINDICATION to intermittent compression? 1. Past history of deep vein thrombosis 2. Impaired sensation 3. Local infection 4. Peripheral neuropathy

Local infection 1. Recent or acute deep vein thrombosis is a contraindication, but a past history or having a predisposition to deep vein thrombosis is not. 2. Impaired sensation is a precaution. 3. Infection may spread as a result of compression, and, therefore, infection is a clear contraindication. 4. Peripheral neuropathy is not a contraindication; it is a precaution.

A physical therapist is treating a patient with an acute ankle sprain. The therapist plans to utilize elastic bandaging to reduce the patient's ankle edema. Which of the following bandages would be the most appropriate to use with this patient? Short stretch bandaging to utilize a high working pressure Short stretch bandaging to utilize a high resting pressure Long stretch bandaging to utilize a high working pressure Long stretch bandaging to utilize a high resting pressure

Long stretch bandaging to utilize a high resting pressure -ace wraps are a type of long stretch bandaging that are desirable due to high extensibility, high resting pressure, and low working pressure. They use compression while the patient is resting to move fluid back toward the heart. -Short stretch bandages have a high working pressure and a low resting pressure. These are normally used to manage edema due to venous disease and lymphedema and to assist with the muscle pumping action of the calf muscles while the patient is active.

Physical therapist working in an acute care hospital treats a patient with transmission based precautions. The therapist treats the patient in their room and is able to leave the door open during treatment. The therapist is required to wear a mask only once they are within 3 feet of the patient. This scenario would be most appropriate when treating a patient diagnosed with what condition? MRSA Mumps Impetigo Measles

Mumps -this patient likely has a pathogen associated with droplet transmission based precautions since the door can be left open during treatment and the therapist only needs to wear a mask when within 3 feet of the patient. -MRSA is associated with contact precautions through direct or indirect contact. Direct is skin to skin while indirect involves a contaminated intermediate object. Gloves and a gown are required for close contact -mom says associated with droplet precautions. Droplet transmission of infectious agents could occur through contact to the mucous membranes of the mouth and nose, contact with the conjunctivae, and through coughing, sneezing, talking or suctioning. Transmission requires close contact as the infectious agents do not suspend in the air and travel only 3 feet or less, therefore a mask should be worn when working within 3 feet of the patient. -Impetigo is associated with contact precautions. -measles is associated with airborne precautions. This will require a private room with the door closed and monitored negative air pressure.

What is a Lisfranc injury?

One or more of metatarsal bases displaced from tarsus. Usually occurs by crush injury. Like when an athlete plants foot and another player lands on their mid foot.

A physical therapist discusses common cognitive and behavioral changes associated with stroke with family members of a patient with right hemisphere damage and resultant left hemiplegia. Which term does not accurately describe the typical patient presentation? Poor judgment Impulsive Quick Overly cautious

Overly cautious -Usually, patients with left hemisphere damage often are described as cautious, anxious, and disorganized. They often have difficulties communicating and in processing information and sequential manner. -Oppositely, patients right hemisphere damage often have difficulty with reasoning and judgment, difficulty and grasping the whole idea of a task, are often impulsive and tend to overestimate their abilities, and they are often quick when performing mobility or tasks.

What can you test via the anterolateral spinothalamic pathways?

Pain and temperature

What is Osgood-Schlatter disease?

Painful condition resulting from inflammation of the tibial tubercle at the insertion of the patellar tendon. The disorders typically seen in adolescent athletes during periods of rapid growth. Quadriceps strengthening exercises which causes repetitive stress to the inflamed musculoskeletal tissues.

While observing a patient ambulating, a physical therapist notes that the patient's gate has the following characteristics: narrow base of support, short bilateral step length, and decreased trunk rotation. Which medical diagnosis would most likely be associated with a describe gait pattern? CVA Parkinson's Post polio syndrome Multiple sclerosis

Parkinson's -A patient that has experienced a CVA would commonly be found with hemi plegia or hemiparesis dash showing weakness and things like foot drop decreased ability at the ankle knee and hip -a patient with Parkinson's disease has decreased stride length and velocity. As a disease progresses, the patient appears to be attempting to catch up with their center of gravity is the step length become smaller this is termed festination. -a patient with post polio syndrome would have asymmetrical gait pattern secondary to weakness, fatigue, and pain -a patient with multiple sclerosis would have impaired trunk control and balance. They would often be circumduction to assist with foot clearance and ataxia due to weakness and tonal influence

An acute care physical therapist is reading the reports of a patient with chronic kidney disease. The patient appears to be confused and lethargic. His body is warm to touch and he is in bradycardia. His lab findings are as follows: pH: 7.20; HCO3: 21mEq/L; PaCO2: 34 mm Hg. Which of the following is the MOST APPROPRIATE diagnosis for this patient? ncompensated metabolic alkalosis Compensated metabolic acidosis Partially compensated metabolic acidosis Uncompensated metabolic acidosis

Partially compensated metabolic acidosis Chronic kidney disease with renal failure results in acidosis because the failing kidney not only is unable to rid the body of excess acids but also cannot produce necessary bicarbonate. Normal values for pH = 7.35-7.45, PaCO2 = 35-45 mm Hg; and HCO3- = 22-26 mEq/L.

What would you see in adrenal insufficiency

Patience would be hyponatremic, Hyperkalemic, hypo glycemic, and may have acidosis. Adrenal glands produce hormones that help regulate your metabolism, immune system, blood pressure, response to stress and other essential functions.

cardiac rehab protocol

Phase 2: 4-9 METs 30-60 mins, 3-4 times/week. 5-10 mins warm up and cool down Discharge: reaching 9 METs

A patient with cardiac arrhythmias has been prescribed beta blockers. What is the most effective way to monitor the patient's activity tolerance? RPE Visual analogue scale Palpation of radial artery Assessment of telemetry strip

RPE Beta blockers are prescribed to address cardiac ischemia, hypertension, and cardiac arrhythmias. Beta blockers decrease myocardial oxygen demand through decreasing sympathetic input to the heart tissue as a result heart rate and contractility decreases (mute the heart rate response to exercise). RPE, or the Borg scale, subjectively access as exercise intensity and is the preferred method for a patient taking beta blockers.

If physical therapist is treating a patient with adhesive capsulitis in an outpatient setting. The patient has a history of gastroesophageal reflux disease and is currently treated with Prevacid. Which position would potentially be the most problematic for the patient when performing upper extremity exercises? Recumbent position on a mat table in right side lying Standing on level surfaces Setting with hips and knees flexed to 90° Standing without support on a foam surface

Recumbent position on a Mat table and right sidelying -GERD is the result of an incompetent lower Soffa Gille sphincter that allows reflux of gastric contents. This backwards movement of stomach acid and contents can cause esophageal tissue injury overtime. -for a patient with GER de you will need to avoid certain exercises, avoid exercising in a recumbent position, avoid right side line, and avoid tight clothing. -When lying on your right side, your stomach is actually above your esophagus, creating a leaky faucet spouting stomach acid into the delicate lining of your esophagus. This is especially true when your stomach is full. -Sleeping on the back increases how often acid reflux happens at night. When you sleep flat on your back and acid escapes from your stomach, it can flow freely into your esophagus and remain there

A patient that required a medical ventilator for two weeks following a near drowning incident is clear to gradually decrease use of the device. Which measured cardio pulmonary value would indicate a sign of distress during the weaning process? Respiratory rate of 38 breaths per minute Tidal volume of 350 mL Pulse oximetry reading at 91% Heart rate change of 10 bpm over baseline

Respiratory rate of 38 breaths per minute -the normal respiratory rate is 12 to 16 breaths per minute. A significantly elevated respiratory rate of 38 breaths per minute is a cardinal sign of distress -350 mL would not be a distress tidal volume. The one less than 325 mL would be (average is ~500mL) -pulse ox less than 90 would be distressed Heart rate change of 20 bpm or more would be considered a sign of distress

What is a STEMI Heart attack?

ST elevated myocardial infarction. On an ECG strip this would be shown by an elevated ST wave of More than 1 mm.

ABG partially vs completely compensated vs uncompensated

Something would be completely compensated if the pH balance were back within normal levels with both the bi carbonate and partial pressure of carbon dioxide being slightly off. Something is partially compensated if the bi carbonate and partial pressure of carbon dioxide are slightly off and the pH is also slightly off. Something is uncompensated is one item and the pH are off and the other item is within normal limits.

Which of the following interventions is MOST appropriate for an individual who has multiple sclerosis? 1. Submaximal exercise performed in the morning 2. Swimming exercises performed in a heated indoor pool 3. Resistance exercise alternating each day between upper and lower body workouts 4. Maximal exercise to the point of fatigue with several days of rest between training sessions

Submaximal exercise performed in the morning MS patients are susceptible to temp changes sessions should circuit train between UE and LE in one session. All upper in one session will increase fatigue. don't want to fatigue

Physical therapist instructed patient in an exercise designed to increase pelvic floor awareness and strength. The exercise requires the patient to tighten the pelvic floor isometrically as if attempting to stop the flow of urine. What is the most appropriate initial position for this exercise? Supine Sitting Tall kneeling Standing

Supine -the exercise described is called the Kegel exercise -the initial position for the pelvic floor exercise should remove or minimize the influence of gravity -kegel exercises are often used as a part of treatment program for incontinence.

Which of the following humeral fractures must be examined MOST immediately for neurovascular status?

Supracondylar

Symptoms of unilateral vestibular hypofunction

Symptoms of UVH - resting (spontaneous) nystagmus, oscillopsia, dysequilibrium, postural instability.

Call therapist monitors the blood pressure of a patient during increasing levels of physical exertion. Assuming a normal physiologic response, which of the following best describes the patient's blood pressure response to dynamic exercise? Systolic pressure increases, diastolic increases Systolic pressure remains the same, diastolic increases Systolic pressure and diastolic pressure remain the same Systolic pressure increases, diastolic pressure remains the same

Systolic pressure increases, diastolic pressure remains the same -The normal response to dynamic exercise is a progressive increase in systolic blood pressure and no change in diastolic pressure. -normal increase in systolic blood pressure is approximately 5 to 10 mmHg per metabolic equivalent. -an increase in diastolic blood pressure of more than 10 mmHg may be indicative of exertional ischemia

pressure ulcer scales vs diabetic ulcer scales

The Wagner Ulcer Grade Classification system is a tool designed for examination of the diabetic foot -Grade one ulcer is superficial and not involving subcutaneous tissue -Grade 2 ulcer is a deep ulcer with penetration through the subcutaneous tissue potentially exposing bone, tendon, ligament or joint capsule. -Grade 3 ulcer is a deep ulcer with osteitis, abscess or osteomyelitis. It would have infection. -Grade 4 ulcer involves gangrene of the affected digit. Bates-Jensen Wound Assessment Tool Gosnell, Braden, or Norton Ulcer Assessment Scale Pressure Ulcer Scale for Healing all pressure ulcer scales -Stage 1 ulcers have not yet broken through the skin. -Stage 2 ulcers have a break in the top two layers of skin. -Stage 3 ulcers affect the top two layers of skin, as well as fatty tissue. -Stage 4 ulcers are deep wounds that may impact muscle, tendons, ligaments, and bone. -Unstageable cant be seen due to necrosis or slough

describe hyper parathyroidism vs hypoparathyroidism

The most common cause of hyperparathyroidism is a benign tumor of one or more of the parathyroid glands. The pneumonic "moans, groans, Stones and bones" covers many of the signs and symptoms associated with the disease. -moans: mental problems such as drowsiness, depression, confusion, and poor memory -groans: arthralgia, myalgia, gout and abdominal pain -stones: kidney stones or renal calculi often result from hypercalcemia -bones: bones are often affected in hyperparathyroidism like decalcification (osteopeinia) or pathologic fractures. also increased reflexes and muscle atrophy hypoparathyroid: hypocalcemia and cardiac arrhythmias.

What is an optimal training range of a patient's max inspiratory pressure to increase ventilatory muscle strength?

The optimal training range for increasing ventilatory muscle strength is 30% to 75% of the patients max inspiratory pressure.

Signs and symptoms of femoroacetabular impingement

This is a deformity of the for moral head or neck and the acetabular rim. Patients typically complain of anterior groin or hip pain reproduced by FADIR & FABER tests, hip internal rotation less than 20°, mechanical symptoms such as snapping or clicking, and radiographic findings of a cam or Pincer deformity. Symptoms would be aggravated by deep squats in sport activities.

lymph from different areas is drained to which ducts?

Thoracic duct: L and R legs, L side of upper body Lymphatic duct: R side of upper body Lower extremities > Lumbar lymphatic trunks > Cisterna chyli > Thoracic duct > Left subclavian vein (bc legs go to thoracic, which is on the L side)

closed packed positions of elbow

Ulnohumeral joint: extension with supination; radiohumeral joint: 90 degrees of flexion with 5 degrees of supination; proximal (superior) radioulnar joint: 5 degrees of supination

Physical therapist determines the examination that a patient exhibit poor (2/5) strength of the adductor pollicis and a positive Froment's sign. The therapist is most expect a policy involving which of the following nerves? Median Anterior interosseous Ulnar Posterior interosseous

Ulnar -a positive Froment's sign is the patient flexing the distal phalanx of the thumb when trying to keep the therapist from pulling a piece of paper away from between their thumb and index finger. This is due to weakness or paralysis of the adductor pollicis muscle. And this muscle is innervated by the deep motor branch of the ulnar nerve. -median nerve lesions may present with weak pronation of the forearm, weak wrist flexion, paresthesia of the first through half of the fourth fingers, and atrophy of the thenar muscles. -anterior interosseous nerve is a motor branch of the median nerve. It innervates the muscles in the deep layer of the anterior forearm including flexor pollicis longus, the index and long fingers of the flexor digitorum profundus, and the pronator quadratus. The classic sign of anterior interosseous syndrome is in ability to perform the OK sign. -the posterior interosseous nerve is a deep motor branch of the radial nerve innervates supinator, extensor carpi radialis brevis, extensor carpi ulnaris, extensor digitorum, extensor indicis, extensor digiti minimi, abductor pollicis longus, extensor pollicis longus, and extensor pollicis brevis. Palsy of this nerve typically affects the ability to extend the fingers and thumb. -the ulnar nerve innervates the flexor carpi ulnaris and the medial half of the flexor digitorum profundus. The deep motor branch of the ulnar nerve innervates the hypo thenar muscles, the interossei muscles, the two medial lumbricals, the adductor pollicis muscle, and the deep part of the flexor pollicis brevis. Weakness of the adductor pollicis and a positive Froment sign are typical signs of ulnar nerve palsy

What is the obturator sign and when is it used?

Used for suspected acute appendicitis. It is internal rotation of the hip with the hip and knee flexed.

Pivot shift test

Used to identify anterolateral rotary instability. Pivot shift test provides additional information about the lateral stabilizers of the knee. It determines the presence of a rotary component to the anterior knee instability.

A physical therapist is treating a patient with right hemiplegia and standing balance activities. The therapist works on reaching activities on level surfaces, on uneven surfaces, and by picking items up off the ground. Which type of practice is most consistent with a describe interventions? Blocked Distributed Constant Variable

Variable -Variable practice refers to the practice of a given task under different conditions -Blocked practice is consistent practice of a single task, performing the same single task repeatedly -Distributed practice is where a patient's rest time between trials is equal to or greater than the practice time itself -Constant practice is practicing a given task under a uniform condition, like performing standing balance activities in the same environment with the same equipment each trial.

Where would you find arterial and venous ulcers?

Venous ulcers are more commonly found on the distal lower leg and medial malleolus. Arterial ulcers are more commonly found on the distal lower leg like the toes and foot, the lateral malleolus, or the anterior tibial area

What is pleural effusion?

accumulation of fluid in the pleural space (located between the parietal and visceral pleura. Not in the airways.) So no secretion techniques are needed.

Motor pathologies of cerebellar impairment

asthenia, dysarthria, dysmetria, dysdiadochokinesia, dyssynergia, asynergia, ataxia, hypotonia, nystagmus, rebound phenomenon, tremor, longer time to initiate movements, harder to stop movements

voice sounds

bronchophony: increased vocal resonance. greater clarity and loudness "99" egophony: form of bronchophony. long E changes to long, nasaly A whispered pectoriloquy: increased loundness of whispers

Sxs of Pancoast tumor

can cause compression paravertebral sympathetic nerves leading to Horner's syndrome (drooping eyelid, lack of sweating, pupil constriction), compression of recurrent laryngeal nerve causing hoarseness of voice, compression of brachial plexus causing atrophy and weakness of muscles of arm and hand

presentation of deep tissue injury

center of the ulcer is a localized area of deep purple tissue surrounded by non-blanchable redness indicating a deep tissue injury. In deep tissue injury, wound may present as a dark purple or blue hue with erythema or a blood-filled blister caused by damage of underlying soft tissue by pressure or a shear force.

cord syndromes

central cord: from hyperextension. UE > LE. motor > sensory. anterior: flexion injury - diving into pool. complete loss of movement, and pain and temperature loss, but it preserves light touch sensations posterior: causes loss of light touch sensation, but it preserves movement, and pain and temperature sensation

Medial inferior pontine syndrome

contralateral hemiparesis of face and extremities and ipsilateral nystagmus.

Lyme disease

could follow tick bite Stage 1, the early, localized stage: fatigue, chills, fever, headache, lethargy, myalgias, or arthralgias Stage 2, disseminated infection, occurs within weeks to months after the tick bite: multiple, smaller erythema migraines, similar to the original rash. aseptic meningitis with mild headache, stiff neck, and difficulty with mentation; cranial neuropathies, particularly Bell palsy (involvement of cranial nerve VII); and radiculopathies. Stage 3, late persistent infection, may become apparent months to years after the initial infection. chronic monoarticular (one joint) or oligoarticular (affecting only a few joints) arthritis. This type of arthritis is associated with marked pain and swelling, especially in the larger joints, such as the knees.

are obese patients more susceptible to hyperhydration or dehydration?

dehydration They have increased heat intolerance as that they are less able to adapt to temperature changes, risk of hyperthermia and heat exhaustion.

which part of the GI tract directly touches the stomach

duodenum

What are some risk factors for osteoporosis?

female white/asian increase age low body weight smoking inactivity/sedentary certain medications alcohol calcium/vitamin D deficiencies CO morbidities Estrogen deficiency Excessive caffeine intake (increases the amount of calcium lost through urine) hyperthyroidism hyperparathyroidism

end feels

firm (stretch) ex: ankle dorsiflexion. hard (bone to bone) ex: elbow extension. soft (soft tissue approximation) ex: elbow flexion, knee flexion empty: motion stopped by patient from pain boggy: edematous abnormal hard: loose articular

the effects of aging on the respiratory system?

increase in residual volume & functional residual capacity, increase in airway resistance decrease in diffusion capacity.

tight hip abductors would cause the pelvis on that side to lower or rise?

lower

what is preferred? higher vs lower duty cycle larger vs smaller max voluntary isometric contraction (MVIC)

lower duty cycle larger MVIC

should you exercise a patient prior to dialysis?

no. avoid this and keep METs to 3-5 and use RPE to monitor

optic lesions and concurrent dysfunctions

optic nerve (past chiasm): complete loss of vision in ipsilateral eye optic tract (2 pieces before they split to separate (same) sides at chiasm): Homonymous Hemianopia optic chiasm: loss of heteronymous hemianopsia occipital cortex: macular sparing (spares center field of vision of each eye)

What is myositis ossificans and when could it occur?

painful condition of abnormal calcification within a muscle belly. It can occur by aggressively stretching a muscle following direct trauma that results information of a hematoma.

What is fremitus?

palpable vibration on the chest wall when talking increased with consolidation bc sounds are more easily transmitted. just like breath sounds are increased with consolidation.

Best glide for adhesive capsulitis?

posteroinferior

neuro facilitatory treatment interventions for flaccid muscles to help stimulate them to contact.

quick stroking, quick icing, rapid vestibular stimulation

spondylolisthesis vs retrolisthesis

spondylolisthesis: there is a forward displacement of one vertebra over another. retrolisthesis: the upper vertebral body is displaced backward relative to the vertebrae below

What is rhabdomyolysis?

the breakdown of muscle tissue releases muscle fiber contents into the blood. -this may occur after over exertion or fall and crush injuries and burns. Those with liver impairment from alcohol abuse or prolonged use of some drugs such as statins or at risk for developing rhabdo. -these substances can cause kidney damage -lab testing will show creatine kinase levels that are more than 10 times the upper limit of normal. -leads to muscle aches, cramps, weakness and soreness. -Dark color of urine is due to liver failure.

which nerve is compressed in anterior compartment syndrome of the LE?

the common peroneal nerve, Motor loss includes an inability to dorsiflex the foot (drop foot), which results in a high steppage gait. Sensory loss is a small triangular area between the first and second toes.

A fracture involving listers tubercle would affect what thumb motion? and which tendon wraps around the tubercle?

thumb interphalangeal extension - EPL tendon

Physical therapist assesses the saccadic movements of a patient with a mild traumatic brain injury and notes that the patient has difficulty shifting their gaze laterally, though the other movements occur normally. Which cranial nerve is most likely impaired? ii iii iv vi

vi -Cranial nerve six (efferent) is the abducens nerve which innervates the lateral rectus muscle of the eyeball. It abducts the eye laterally. -Cranial nerve four (efferent) is the trochlear nerve innervates the superior oblique muscle the eyeball and move the eye down and in -Cranial nerve three (efferent) is the oculomotor nerve innervates multiple muscles of the eyeball. It allows the eye to move up, down, medially, and lift the eyelid -Cranial nerve two (afferent) is the optic nerve which gives the sensation of sight. -Afferent Arrives (sensory), Efferent Exits (motor)

around what week does a grade 2 ligament injury begin to organize?

week 3 - though not fully organized yet also deficient in tensile strength

when would a walking program be stopped on the claudication scale?

when reaching a 2

classic symptoms of multiple sclerosis

women > men nystagmus and diplopia trigeminal neuralgia Lhermitte's sign aggravation of symptoms with heat

A physical therapist contemplates the necessary amount of force to overcome friction when performing mechanical lumbar traction using a non-split traction table. What percentage of a patient's body weight would be most representative of this value? 5% 15% 25% 50%

25% -a significant amount of overall traction force is necessary to overcome friction between the patient's body and the surface of the traction table. -this percentage is found by multiplying the percentage of body weight below L3 (50%) and the coefficient of friction (.5 - this is the coefficient of friction of the human body on a mattress) -it's a split traction table were used it would significantly decrease the effects of friction

Rifampicin

Anti-TB medication. causes red-orange tears and urine Isoniazid, Rifampicin, Ethambutol and pyrazinamide (PZA) are first line anti-TB drugs

Cranial nerve reflexes

BLINK/CORNEAL: trigeminal 5 (sensory) and facial 7 (motor) GAG: glossopharyngeal 9 (sensory) and vagus 10 (motor) PUPILLARY: optic 2 (sensory) and oculomotor 3 (motor - if one eye constricts and one doesnt)

A physical therapist prepares to apply sterile dressing to a wound after debridement. The therapist begins the process by drawing the wound using a towel. The therapist supplies medication to the wound using a gauze pad and then applies a series of dressings that are secured using a bandage. The application of which step would not warrant the use of a sterile technique? Bandage Dressings Medication Towel

Bandage The bandage is the only part of this application that does not come in direct contact with the wound, that it does not require sterile technique.

Physical therapist treat a patient who has an entrance wound on the top of their head and an excellent wound at the anterior right thigh secondary to coming into contact with a live powerline while at work. Which of the following complications with a therapist to be most alert for given the type of injury this patient sustained? Hypervolemia Cardiac arrhythmias Pulmonary fibrosis Heterotropic ossification

Cardiac arrhythmias This injury was caused by an electrical burn. Complications can include cardiac arrhythmias, respiratory arrest, renal failure, neurological damage, and fractures.

During a gate analysis on a patient rehabilitating from lower extremity injury, the physical therapist measures the number of steps taken by the patient in a 30 second period. The therapist is measured which of the following gate parameters? Acceleration Cadence Velocity Speed

Cadence -Cadence is the number of steps taken by a person per unit of time. Walking with increased cadence will decrease the duration of double support time. Cadence of 110 steps per minute is typical in a male and 116 steps per minute is typical in a female. -Acceleration is the rate of change of velocity with respect to time -Velocity is the rate of linear forward motion of the body which is measured in centimeters per second, meters per second or miles per hour. Walking velocity equals distance walked divided by time. -Speed is usually classified as slow free (normal walking speed) or fast.

What are two ways to decrease heart rate without the use of beta blockers?

Carotid massage and the Valsalva maneuver

A patient in the physical therapy jam suddenly begins to cough, which is shortly followed by wheezing. The physical therapist, recognizing the possibility of an airway obstruction, should take which of the following actions? Attempts to ventilate Administer abdominal thrusts Perform a quick finger sweep of the mouth Continue to observe the patient, but do not interfere

Continue to observe the patient, but do not interfere -coughing and wheezing indicate that the airway is not completely obstructed. As a result, the physical therapy should continue to monitor the patient, however, should not formally intervene. Usually a patient that is coughing or independently dislodge the object causing the obstruction.

Which of the following muscular responses would be expected during normal micturition?

Contraction of the detrusor and relaxation of the pelvic floor

An 82-year-old patient reports fatigue when ambulating to and from their home mailbox. Which of the following factors would most likely contribute to the patient reported difficulty? Increased diastolic blood pressure Decreased cardiac output Decreased cardiac afterload Increased arterial patency

Decreased cardiac output -cardiovascular changes that occur with aging include increased cardiac afterload, reduced cardiac output, and increased systolic blood pressure. -An increase in diastolic blood pressure is not common with aging, nor would it increase the patient's level of fatigue -decreased cardiac output it means decreased perfusion of the peripheral tissues causing exertional fatigue -cardiac afterload is the force against which of the heart muscle contract and move blood into the systemic circulation (increased afterload would cause increased fatigue). Afterload normally increases with age. -arterial patency decreases with aging into plaque deposits and diminished elasticity of the arterial walls.

which burn is most likely to cause keloid scar

Deep partial-thickness -Epidermal burns will not produce scars. just red skin. -Superficial partial-thickness produces minimal scarring. blisters present. -deep partial thickness will have lots of scarring. no blisters with slow capillary refill (has blanching). -Full-thickness scarring secondary to escharotomy and grafting. White with no blanching (bc its not red) -Subdermal scarring secondary to escharotomy and grafting. down to fat layer, no nerve endings left

duodenal vs gastric ulcers

Duodenal is painful with empty tummy Gastric is painful with full tummy

When would be a good time to use cryotherapy?

During an acute inflammatory condition. It should be applied as soon as possible in multiple times a day during the acute cycle. No longer than 20 minutes.

Internal versus external obliques

External obliques are your pocket muscles internal obliques run the opposite way

A physical therapist assesses a patient with a traumatic brain injury using the Glasgow coma scale. If the therapist documents the eye-opening score as a three (E3), which of the following responses was observed during this portion of the assessment? Spontaneous eye-opening Eyes open in response to speech Eyes open in response to pain Eyes do not open (nil response)

Eyes open in response to speech The GCS is a neurological assessment tool used initially after TBI to determine the patient's level of consciousness and the severity of the injury. -it is scored on verbal response (1-5), movement response (1-6), and ability to open their eyes (1-4). -for the eye portion, eyes not opening with score a one. Opening eyes only to pain would be a two. Opening eyes in response to speech would be a three. And a patient who can spontaneously open their eyes would be given a four.

A physical therapist completes a developmental assessment on an infant. Which of the following pediatric reflexes should the therapist expect to be integrated at the youngest age? Plantar grasp Moro Landau Galant

Galant -The plantar grasp would be integrated by nine months to ensure that there's no hindrance towards walking -the Moro reflects would be integrated by five months of age (arms extend and abduct) -Landau reflects isn't integrated until the second year (superman in prone) -galant would be integrated by two months (laterally flex to side stimulated)

Physical therapist is considering the use of electrical stimulation on a chronic wound. Which of the following sets of parameters is the most appropriate? Monophasic, direct current Biphasic, direct current Monophasic, alternating current Biphasic, alternating current

Monophasic, direct current -the electrical stimulation appropriate for a chronic wound would be high-voltage post current. This uses monophasic direct current to stimulate angiogenesis and epithelial migration, decrease bacterial activity and wound pain, and increased oxygen perfusion and tensile strength. -monophasic current is a type of pulsatile current and produces a polarity effect in tissues. -Direct current is characterized by constant flow of electrons from the anode (positive electrode) to the cathode (negative electrode). -Biphasic pulsed current has two phases, one which is positive and one which is negative. -An alternating current is categorized by polarity that continuously changes from positive to negative with a change in direction of current flow.

A patient is unable to take in an adequate supply of nutrients by mouth. As a result of patient's physician orders the implementation of tube feeding. What type of tool is most commonly used for short term feeding? Endo brachial Nasogastric Endotracheal Thracheostomy

Nasogastric -an Endo brachial tube is a flexible catheter for bronco spirometry and to control secretions and the remainder of the tracheobronchial tree under anesthesia -an NG tube is a plastic tube inserted through the nostril that extends the stomach. Use for liquid feeding, medication administration or to remove gas from the stomach for short term feeding. A gastric tube is inserted directly into the stomach for long-term feeding. -an endotracheal tube is an airway catheter inserted into the trachea for endotracheal intubation -a tracheostomy is when an opening is made in the trachea in order to insert a catheter or tube to facilitate breathing.

Venous filling times

Normal filling time is 15 seconds. Greater than 15 seconds indicates arterial disease Less than 15 indicate venous disease.

What type of exercise should be avoided for the first 6 to 8 post operative weeks after a medium meniscal tear?

Open chain resisted knee flexion The attachments of the medial meniscus include the semimembranous tendon, MCL and fibrous capsule, and the medial meniscal patellar ligament. During open chain resisted knee flexion, the semimembranous tendon will pull on the posterior aspect of the medial meniscus and in doing so may tear the surgical repair.

Stages of pressure Ulcers (1-4)

Stage 1. - persistent area of skin redness (without break in skin) that does not disappear when pressure is relieved Stage 2. - partial thickness loss of skin layer = abrasion, blister, shallow crater presented Stage 3. - full thickness of skin, lost, exposing the subcutaneous tissues = looks like a deep crater Stage 4. - full thickness of skin and subcutaneous tissue lost, exposing muscle, bone = present as deep crater, may include necrotic tissue, exudate, infection

psoriatic and rheumatoid arthritis complications

Psoriatic: psoriatic skin and nail changes, conjunctivitis and iritis rheumatoid: vasculitis

NY Heart Association Classification (4 classes)

-Class one patients will have no limitations in their physical activities. Ordinary physical activity will not cause undue fatigue, palpitations, dyspnea or anginal pain. -Class two patients will have slight limitations in their physical activities and are able to complete less than ordinary physical activities. Ordinary physical activities may result in fatigue, palpitations, dyspnea or anginal pain. -Class three patients will have marked limitations in their physical activities. They are comfortable at rest and now less than ordinary physical activity causes fatigue, palpitations, dyspnea or anginal pain. -Class four patients are unable to carry on any physical activity without discomfort. Symptoms of cardiac insufficiency or anginal syndrome may be present even at rest.

Lymphedema staging (0-3)

-Stage 0 (preclinical stage), patient begins to feel heaviness in limbs. -Stage 1 is reversible lymphedema and elevation of the arm will reduce swelling. -Stage 2 as the patient doesn't experience any changes of swelling during arm elevation and initial signs of fibrotic changes are present during this stage. -Stage 3 is lymphostatic elephantiasis, hardening of dermal tissues, papilloma's of the skin will be noticed.

deep vs superficial peroneal nerve

-Superficial fibular (peroneal) nerve supplies the muscles of the lateral compartment of the leg and provides sensation to the anterolateral aspect of the leg. ((Deformity: equino varus (clubfoot) -supination and PF)) -Deep fibular (peroneal) nerve mainly supplies the muscles of the anterior compartment of the leg and the dorsum of the foot (Tibialis anterior, Extensor digitorum brevis, Extensor digitorum longus, Extensor hallucis longus, Peroneus tertius). Sensory around the webspace between first and second toes

q angle

-formed between quad muscle and patellar tendon -increases with femoral anteversion (genu valgum) -decreases with retroversion normal in female is 13-18 degrees

Tests in laslett's cluster?

-gaenslens (supine - one hip into ext, one flex) -sacral thrust (pt prone - push on sacrum) -compression (sidelying and push on hip) -distraction (push on both ASIS) -thigh thrust (pt supine with hip flexed to 90)

swayback

-hips and knees extended -PPT -elongated neck flexors -favored leg - longer

exercise changes with pregnancy

-lower BP in first few months, then increase - increased HR -Respiration rate will not increase proportionally with moderate and severe exercise -Hematocrit levels will be lower but increase during vigorous exercise -Stroke volume and cardiac output will increase with steady state exercise - Increased O2 consumption -Cardiac reserve is decreased during exercise -decreased total lung capacity (restrictive) -decreased vital capacity -increased tidal volume

A patient with suspected pulmonary embolism is referred for additional diagnostic testing. Which of the following patients would be the greatest risk for developing his medical condition? 35YO female on bed rest, obese, on oral contraceptives 45YO male two weeks post tibial fracture requiring internal fixation 70YO female three weeks post closed reduction of a colles' fracture 50YO male three weeks post malleolus fracture in walking boot

35 year-old female on bed rest, obese, on oral contraceptives -any condition that slows or changes the flow of blood within a vein can increase the risk for thrombus formation and ultimately a pulmonary embolism. These risk factors include prolonged immobility, recent surgery, pregnancy, obesity, cigarette smoking, medications like birth control and hormone replacement, blood conditions characterized by thick blood, blood clotting disorders, advanced age, cancer, and certain autoimmune disorders. -the patient in the example is immobile, obese, and on birth control.

A patient in the hospital following surgery sets the target indicator on an incentive spirometer. Which individual would most likely have the highest target value when using the device? 35 year-old female, 66 inches tall, post transmyocardial revascularization 45-year-old male, 72 inches tall, post total knee arthroplasty 55-year-old female, 64 inches tall, post cardiac valve replacement 55 year-old male, 70 inches tall, post CABG

45-year-old male, 72 inches tall, post TKA -incentive spirometry provides visual feedback to encourage the patient to take long, slow, deep inhalations. -males typically have larger lungs and females, increase high correlate positively to increase target incentive capacity, and age has a negative impact on inspiratory capacity due to loss of elasticities in the lung tissue -although the surgical procedure is typically a less relevant factor, the patients undergoing cardiac surgery would play a role due to sternal precautions and expected pain with lung expansion

A physician provides a group of physical therapist with an overview of diagnostic imaging techniques commonly used in clinical practice. Which imaging technique would be considered non-invasive? Anthrography Myelography Discography Computed tomography (CT)

A CT scan -Anthropgraphy is an invasive test utilizing a contrast medium to provide visualization of joint structures through radiographs. Soft tissue disruption can be identified by leakage from the joint cavity and capsule. -myelography is an invasive test that combines fluoroscopy in radiography to evaluate the spinal subarachnoid space. A utilizes a contrast medium that is injected into the epidural space by spinal puncture. It is used to identify bone displacement, disc herniation, spinal cord compression or tumors. -Discography is an invasive test that involves injecting a radiopaque dye into the nucleus pulposes of an intravertebral disc using a radiographic guidance. This can be used to identify disruptions of the nucleus pulposes or the annulus fibrosus. CT scans are non-invasive and use cross-sectional images better than enhanced on a computer to allow the images to have significantly better contrast resolution when compared to conventional x-rays.

Meniere's disease

Abnormal condition within the labyrinth of the inner ear that can lead to a progressive loss of hearing. The symptoms are dizziness or vertigo, hearing loss, and tinnitus (ringing in the ears).

What is hip dysplasia?

Abnormality in the size, shape, orientation, or organization of the memorial head and/or acetabilen that can result in hip sub relaxation or dislocation. More common in females than males.

Physical therapist attempt to determine a patient's general willingness to use an affected body part. What objective information would be the most useful? Passive movement Sensory testing bony palpation Active movement

Active movement Active movement is the only option which shows the patient actually willingly moving their body part

Active vs passive insufficiency

Active: muscle is shortened over both joint (triceps during shoulder and elbow extension) Passive: muscle is stretched over both joints (hamstrings tight in hip flexion and knee extension)

Addison's disease vs cushings disease

Addisons (primary adrenal insufficiency): occurs when the adrenal glands do not produce enough of the hormones cortisol or aldosterone dark pigmentation of the skin low blood pressure, fatigue which improves with rest, hypoglycemia. weight loss. abdominal pain, abnormal menstrual periods, dehydration, depression, diarrhea, dizziness, loss of appetite, muscle weakness, nausea, patches of dark skin around scars/skin fold and joints, sensitivity to cold, unexplained weight loss, worsening fatigue, hyponatremia Cushings: excess of free circulating cortisol from the adrenal cortex. significant weight gain in the abdomen and in the face. bruising and stretch marks. high levels of cortisol, elevated blood glucose and high levels of ACTH from pituitary, amenorrhea, high blood pressure, osteoporosis, and susceptibility to bruising.

A patient suffers a laceration injury to the forearm while exercising in the therapy gym. The physical therapist is having trouble stopping the flow of blood from the wound. Which of the following actions would be the last resort when treating the laceration? Apply pressure directly over the wound Apply a tourniquet proximal to the side of injury Elevate the arm above the level of the heart Apply pressure to the brachial artery

Apply a tourniquet proximal to the site of injury -first steps should be to elevate the extremity and apply pressure directly over the wound after washing their hands, applying gloves, and using a clean towel. -if that isn't helping, and the next step would be to apply pressure to the brachial artery, this is an ideal because it doesn't just stop blood circulation to the affected extremity, and also stop circulation within the arterial distribution. -the absolute last step would be using a tourniquet because it could create tissue damage that is unnecessary. It is only used in cases of severe bleeding that are life-threatening.

A patient begins to cry in the middle of a treatment session. The physical therapist attempts to come for the patient, however, eventually has to discontinue treatment. What section of the soap note would be the most appropriate to document the incident? Subjective Objective Assessment Plan

Assessment -Adding this into the assessment section serves to justify the decision to terminate treatment. -The assessment is a section where the physical therapist can express their opinion. Short and long-term goals are often expressed here as well as changes in the treatment program. -the plan section includes ideas for future physical therapy sessions. Frequency and expected duration of services are also incorporated here. -the subjective section refers to information the patient communicates directly to the physical therapist. Like statements, social history, medical history or patient complaints. -the objective section refers to information the physical therapist observes. Including range of motion, muscle strength, functional abilities.

Hypo vs hyper thyroidism

Both: proximal muscle weakness, fatigue, hair loss. Hypo (hashimotos): constipation, sensitive to cold, decreased HR, increased BP, increased blood glucose, weight gain, dry skin, hair loss, depression, joint stiffness. carpal tunnel, bradykinesia. Hyper (graves disease): diarrhea, sensitive to heat, increased HR, increased RR, weight loss, hyperpigmentation, low BP. can cause osteoporosis periarthritis of shoulder and tendinitis of wrist, bulging eyeballs

A physical therapist is assessing a patient with a sudden onset of dizziness two days ago. Upon performing the Roll Test, the patient had geotropic nystagmus, stronger on the right side as compared to the left. Which of the following is the MOST APPROPRIATE intervention for this patient? Semont maneuver for canalithiasis with side lying on the L Semont maneuver for cupulolithiasis with side lying on the R Bar-B-que roll for cupulolithiasis maneuver starting on the R Bar-B-que roll maneuver for canalithiasis starting on the R

Bar-B-que roll maneuver for canalithiasis starting on the R Patient presents with horizontal canal BPPV (Canalithiasis) on the R side since the geotropic nystagmus is stronger on the R. The most appropriate intervention for canalithiasis is the bar-b-que roll with the head turned to the involved side first.

what are signs consistent with shock?

Becoming agitated, nauseous, and appearing pale and gray. Usually also an increased pulse and respiratory rate with decreased blood pressure.

A patient sustains a deep laceration on the anterior surface of the forearm. The physical therapist attempt to stop the bleeding by applying direct pressure over the wound, but is unsuccessful. The most appropriate action is for the therapist to attempt to apply compression over the pressure point of which of the following arteries? Brachial Femoral Radial Ulnar

Brachial -when direct pressure and elevation fail to stop severe bleeding from an open wound, attempt to use the pressure point of a major artery. This technique should only be used when absolutely necessary due to it stopping the circulation to not just the injured extremity, but within the arterial distribution. -It is appropriate to compress the artery more proximal to the wound. The femoral artery is in the leg, and the ulnar and radial arteries are distal or even with the laceration.

aquatic effects on CV and respiratory

CV: think relaxing by a pool decreased HR and BP inrceased SV and CO RESP: decreased vital capacity bc hard to breathe in against water forces increased work of breathing

Compare complex regional pain syndrome types one and two

Complex regional pain syndrome is unexplained in hypersensitive pain, temperature changes, skin changes, and swelling of the affected area. Type one has no known nerve damage whereas type two there's no nerve injury such as a crush injury to a peripheral nerve.

Thoracic outlet syndrome

Compression syndrome of upper limb neurovascular bundle at the level of scalene muscles and first rib. 70% with arterial injury have a cervical rib. Can occur from hypertrophied scalene muscles due to their potential for elevating the first rib thereby compressing the neurovascular bundle.

A physical therapist prepares to conduct a gate assessment on a patient recently referred to physical therapy. Which variable would be most influential when attempting to estimate the patient stride length? Coordination Height Strength Weight

Height -stride length is the distance measured between heel strike and the following heel strike of the same limb. -step length is the distance measured between heel strike and subsequent heel strike of the opposite limb. -a patient's height is the primary determinant when estimating stride length since height is positively correlated with limb length and a longer limb will result in a longer stride

When selecting an assistive device for a patient with a lower extremity injury, which of the following patient characteristic should be the least critical for the physical therapist to consider? Cognitive ability Height and weight Upper and lower extremity strength Level of coordination

Height and weight -an assistive device should be selected based on a patient's weight-bearing status as well as their current abilities and limitations. -the majority of assistive devices are appropriate for patients of earring weight and can be readily adjusted to accommodate different heights.

The medical record indicates that a patient being treated for a low back strain is taking Lasix for the management of heart failure. During ambulation activities, the patient complains of lightheadedness fatigue and weakness. The therapist measures the patient's vital signs and identifies an irregular pulse. What is the most likely explanation? Dehydration Angina Hyper natremia Renal failure

Dehydration Lasix is a diuretic that is prescribed to decrease circulating blood volume, thereby decreasing preload. Used in heart failure. -diuretic agents act increased excretion of sodium, potassium and water from the body is urine. As a result dehydration is often a persistent side effect. -diuretics would cause hyponatremia not hypernatremia

Which endocrine diseases can you potentially find adhesive capsulitis?

Diabetes mellitus, hypothyroidism and hyperthyroidism NO night pain

sxs of UVH

Dizziness or vertigo, poor balance, especially with head turns, trouble walking, especially outdoors, in dark rooms, or in crowded places, blurred vision, especially when turning your head quickly. Nausea and vomiting in acute or severe cases. head tilt to one side

Signs of respiratory distress

Dyspnea, shortness of breath, or cramping in the calf muscles

A physical therapist observes that a patient with chronic venous insufficiency is wearing stockings with 20 mmHg of pressure despite being prescribed stockings with 40 mmHg of pressure. Which clinical finding would be most anticipated? Edema Loss of sensation in the feet Angina pectoris Hyporeflexia

Edema Chronic venous insufficiency typically affects the distal lower extremities and is characterized by Venous incompetence and resultant venous hypertension. It increases fluid volume in interstitial spaces and eventually overloads the lymphatic system and results in edema. Compression stockings are commonly used and are normally 30 to 40 mmHg of pressure. -loss of sensation would more likely be observed in patients with diabetes or if they are wearing a garment that is too tight -angina pectoris is a symptom of myocardial infarction not chronic venous insufficiency -hyporeflexia could be a lower motor neuron disorder. Reflexes would not be significantly impacted based on change in stocking pressure

A physical therapist completes a posture screening and muscle length test of the hip flexors on a patient. The therapist determines that the patient has extremely tight hip flexors bilaterally. What common structural deformity is most often associated with tight hip flexors? Scoliosis Exaggerated kyphosis Exaggerative lordosis Spondylolysis

Exaggerative lordosis -shortness of the hip flexors is often observed in standing as lumbar lordosis or in a special test such as the Thomas test. -exaggerative lordosis would have resultant anterior pelvic tilt and causing the hips to become more flexed

low back flexion and extension biases

Flexion: Flexion: Spondylosis, lysis, listhesis, stenosis Extension: OA, ankylosing spond, herniated disc, osteoporosis

A physical therapist exam is a child with spastic quadriplegia cerebral palsy in the school setting. Based on this diagnosis, which muscle is most likely to be shortened? Flexor digitorum profundus Triceps Gluteus minimus Tibialis anterior

Flexor digitorum profundus -cerebral palsy is a non-progressive disorder of the central nervous system resulting in impairments in posture and volitional movement. -spastic quadriplegia describes an upper motor neuron lesion in the motor cortex of the cerebrum involving the trunk and all four extremities. -with this diagnosis, the elbow flexors tend to demonstrate increased spasticity, as well as the wrist and finger flexors, hip adductors, and ankle plantar flexors.

Electrode placement for muscle spasm vs. weak muscles and for generalized work vs. a specific muscle.

For spastic muscles we start with low sensitivity and gradually increase it while with flaccid or weak muscles we start with high sensitivity. Wide space electrodes recruits more muscle activity while closely spaced is for specific muscle.

pad spacing in EMG biofeedback

For strengthening a weak muscle always start with widely spaced electrodes and high sensitivity to recruit more motor units. As the muscle gets stronger, the electrodes are moved closer and sensitivity decreased. For decreasing muscle spasm, the electrodes are placed closer and lower sensitivity is used. And as the muscles relaxes, we can increase the spacing and sensitivity.

Physical therapist examines the wound of a patient with diabetes mellitus. The wound is located on the plantar surface of the foot under the head of the first metatarsal and appears non-infected. When asked to flex the great toe the therapist observes a tendon gliding in the wound bed which grade of the Wagner ulcer grade classification scale best describes this wound? Grade one Grade 2 Grade 3 Grade 4

Grade 2 -the Wagner ulcer grade classification system is a scale to categorize D vascular ulcers grades zero through five based on wound depth and the presence of infection. -Grade one ulcer is superficial and not involving subcutaneous tissue -Grade 2 ulcer is a deep ulcer with penetration through the subcutaneous tissue potentially exposing bone, tendon, ligament or joint capsule. -Grade 3 ulcer is a deep ulcer with osteitis, abscess or osteomyelitis. It would have infection. -Grade for ulcer involves gangrene of the affected digit.

A physical therapist observes a wound that is noticeably red, slightly shiny, and mildly moist. What is the therapist most likely observing? Slough Granulation tissue Necrotic tissue Exposed tendon

Granulation tissue -slough is tan or yellow, soft, and loosely attached to the base of the wound -granulation tissue is pink or red, shiny, and moist -necrotic tissue is dead tissue. Like slough or eschar. eschar is characterized as black or brown, hard, and firmly attached to the base and edges of the wound -and expose tendon would be shiny or moist, but it would not be red

A patient in an acute care hospital has a catheter inserted into the internal jugular vein. The catheter travels through the superior vena cava into the right atrium. The device permits removal of blood samples, administration of medication, and monitoring of central venous pressure. What is the most appropriate term for this device? Arterial line Intravenous line Hickman catheter Swan-Ganz catheter

Hickman catheter -and arterial line consists of a catheter that is inserted into an artery and attached an electronic monitoring system. It is used to measure blood pressure or to obtain blood samples. -an intravenous system consists of a sterile fluid source, a pump, clamp, and a catheter to insert into a vein. This can be used to infuse fluids, electrolytes, nutrients, and medication. They are most commonly inserted into superficial veins such as the facility, cephalic, or anti-cubital veins. They are also used to permit nutrient introduction when the G.I. tract is not able to digest and absorb food. -Hickman catheter (indwelling right atrial catheter) inserted into the right atrium of the heart. It permits removal of blood samples, administration of medication, and monitoring of central venous pressure. -a Swan-Ganz catheter is a soft, flexible catheter that is inserted through a vein and eventually into the pulmonary artery. It is used to provide continuous measurements of pulmonary artery pressure.

common TMJ pathologies

Hypomobility: decreased mouth opening but no pain Disc displacement with reduction: has clicking sound Synovitis: decreased opening but there is no deviation Capsulitis: decreased opening but there is a deviation

What is arthrogryposis multiplex congenita?

It is not a specific diagnosis, but a symptom that can be associated with other medical conditions. It characterized by limited joint motion from contracture resulting in deformity present at birth

What is meconium ileus?

Meconium ileus is a bowel obstruction that occurs when the meconium in your child's intestine is even thicker and stickier than normal meconium, creating a blockage in a part of the small intestine called the ileum. Most infants with meconium ileus have a disease called cystic fibrosis early stage of CF

What is Meneire's Syndrome?

Meniere's disease is a recurrent and usually progressive vestibular disease. It is associated with tinnitus, deafness, sensation of fullness in the ear and vertigo

A patient recovering from pneumonia is prescribed to pharmacological agent administered with a nebulizer which pharmacological agent would most likely be utilized? Calcium channel blocker agents Mucolytic agents Nitrate agents Sedative hypnotic agents

Mucolytic agents -a nebulizer is a device commonly used to administer medication in the form of a mist that is inhaled into the lungs. -patients recovering from pneumonia often present with viscous mucus secretions inhibit proper lung functioning -calcium channel blocker agents decrease the entry of calcium into smooth muscle cells resulting in diminished myocardial contraction, vasodilation, and decreased oxygen demand of the heart. They are used to treat hypertension, angina pectoris, arrhythmias, and congestive heart failure. Administered orally. -mucolytic agent decreased viscosity of mucus secretions by altering their composition and consistency. They are used to treat pneumonia, emphysema, chronic bronchitis, and cystic fibrosis. Administered via nebulizer. -nitrate agents decrease ischemia through smooth muscle relaxation and dilation of peripheral vessels. They're used to treat angina pectoris. Administered sublingually. -sedative agents produce a calming affect and relaxation, while hypnotic agent induced sleep. Used to treat anxiety and insomnia and for preoperative sedation. Administered orally.

Physical therapist treats a patient using a vacuum assisted closure device. Which of the following best describes the mechanism used by this type of device to promote wound healing? Positive pressure in the wound bed Negative pressure in the wound bed Atmospheric pressure in the wound bed Alternating pressure in the wound bed

Negative pressure in the wound bed -vacuum assisted closure devices are most commonly referred to as negative wound pressure therapy. The specialized dressing and battery powered vacuum device creates a negative pressure environment. This aids and the management of wound exudate and interstitial edema and facilitates growth of granulation tissue. -positive pressure would have the tendency to cause ischemia and discomfort that would impede healing Atmospheric pressure is just normal pressure exerted on a wound by surrounding air Alternating pressure is often utilized in specialized mattresses and overlays to prevent skin breakdown and individuals at risk for pressure injuries.

blood pressure guidelines

Normal: less than 120/80 Elevated: systolic between 120-129 and diastolic less than 80 Stage 1: systolic between 130-139 and diastolic between 80-89 Stage 2: Systolic at least 140 or diastolic at least 90 mm hg Hypertensive crisis: systolic over 180 and/or diastolic over 120

mid scap/upper mid back pain referrals

PEGS Pancreas Esophagus Gallbladder Stomach

A patient is referred to physical therapy for right shoulder pain. Which of the following findings suggests that physical therapy intervention may not be appropriate? 1. Pain that subsides with right sidelying 2. Tenderness to palpation at the origin of the biceps tendon 3. Trigger points at the right rib 2-3 intercostal space 4. Pain with resisted shoulder lateral (external) rotation

Pain that subsides with right sidelying Right sidelying often increases musculoskeletal pain but may decrease pleural friction/irritation and thus may decrease visceral pain (Goodman). Trigger points are the most common musculoskeletal cause of chest pain

During an exercise session of physical therapist monitors exercise intensity with a patient post cardiac transplantation. Which method is the most appropriate for the therapist to use? Metabolic equivalents Perceived exertion scale Pulmonary function test Target heart rate range

Perceived exertion rate -A scale such as the Borg RPE scale is a subjective means of reading the intensity of exercise. This is particularly useful following cardiac transplantation since the transmitted heart fails to respond normally the sympathetic nervous system stimulation results in an abnormal heart rate response to exercise -due to the above, target heart rate would not be an appropriate use -pulmonary function tests do not measure exercise intensity yeah -metabolic equivalents are not an objective means of monitoring exercise intensity, although they may be useful to select appropriate exercise activities.

A physical therapist collects data as part of a preseason athletic screening program designed to identify individuals susceptible to heat illness. which of the following measures would be the MOST valuable to collect? Height Weight Percent body fat Vital capacity

Percent body fat Patients with higher body fat percentage are more susceptible to heat illness since the larger the person is, the more difficult it will be to dissipate excess heat.

Physical therapist treat a moderately obese patient in an acute care hospital. The chart review revealed atherosclerosis, diabetes mellitus, and hypertension. The patient presents with an ankle brachial index value of .55. The patient complains of pain with walking that is relieved with rest. Which of the following medical conditions to be the most consistent with the patient's clinical presentation? Peripheral vascular disease Coronary artery disease Congestive heart failure Cor pulmonale

Peripheral vascular disease -the primary factor for developing peripheral vascular disease is atherosclerosis. Other etiologies and risk factors include phlebitis, autoimmune disease, diabetes mellitus, smoking, hypertension, positive family history, and obesity. -normal ABI is considered one to 1.4. -greater than 1.4 indicates calcification and vessel hardening -value of .55 would indicate moderate arterial disease and claudication -coronary artery disease refers to the narrowing or blockage due to fatty buildup of cholesterol within the artery walls reducing the overall blood flow to the cardiac muscles. Symptoms can include nausea, vomiting, heartburn, shortness of breath, chest pain, and profuse sweating. -congestive heart failure is a progressive condition where the heart cannot maintain a normal cardiac output to meet body demands for blood and oxygen. The ventricles weekend and dilate to the point that the heart cannot pump efficiently. This could include arrhythmia, pulmonary embolism, hypertension, valvular heart disease, myocarditis, unstable angina, renal failure, and severe anemia. -cor pulmonale, or pulmonary heart disease, is hypertrophy of the right ventricle caused by altered structure or function of the lung. Cardinal symptoms progressive shortness of breath, Especially with exertion. Other symptoms include fatigue, atypical chest pain, swelling of the lower extremities, dizziness, and syncope.

A patient begins presenting with tonic clonic movements involving off works remedies. What is a therapist best immediate course of action?

Position the patient inside line with the mouth pointing towards the ground. This will ensure an open airway and prevent aspiration. You should also loosen restrictive clothing and remove potentially harmful objects nearby

Normal Hematology values

RBCs: 5-6 x 10^6/uL WBCs: 4-11,000 (no exercise if <5000 with fever) Hemoglobin (Hgb): 12-18g/dL (no exercise <8) Hematocrit (Hct): 37-52 % (no exercise <25%) Platelet: 150,000-450,000 (no exercise <10,000) Prothrombin: 11-15 seconds INR healthy: .9-1.1 INR warfarin: 2-3.5 HbA1c: below 5.7% (over 6.5% is diabetic) ESR: Men under 50 years old: less than 15 mm/hr. Men over 50 years old: less than 20 mm/hr. Women under 50 years old: less than 20 mm/hr. Women over 50 years old: less than 30 mm/hr. C-reactive protein (CRP): Less than 10 mg/L

quadrant pain referral patterns of abdomen

RUQ: gallbladder, pericardium, liver, head of pancreas, peptic ulcers LUQ: diaphragm, tail of pancreas, spleen, heart RLQ: appendix, crohn's (skip lesions) LLQ: IBS, ulcerative colitis (continuous lesions and bloody stool), diverticulitis

What are the Ottawa ankle rules?

Radiographs unnecessary if the following conditions are met: 1. Patient able to walk 4 steps at time of injury or evaluation 2. No bony tenderness over distal 6cm of either malleolus

What are the reliability value ranges?

Reliability coefficient above 0.75 demonstrate good reliability. Confessions from 0.50-0.75 demonstrate moderate reliability and confessions below 0.50 represent poor reliability.

It's all the scene in physical therapy after her mother expressed concern that she had a leg length discrepancy. Examination and standing reveals right shoulder elevation compared to the last, unequal spacing between the upper extremities in the trunk, and the head positioned away from midline. Which condition is most consistent with the describes clinical presentation? Spondylolysis Kyphosis Spondylolisthesis Scoliosis

Scoliosis -everything listed is correspondent with scoliosis -spondylolysis refers to severe degeneration or fracture of a vertebrae. Common in gymnastics, football, and weightlifting. Postural deformities are not typically noted with spondylolysis -Kyphosis is excessive curvature of the spine typically identified in the thoracic spine. -Spondylolisthesis is when the fractured pars interarticularis separates, allowing the injured vertebrae to shift or slip forward on the vertebrae directly below it.

A physical therapist perform stretching activities to address a knee flexion contracture on a patient with complete paraplegia. How would this contracture be classified within the medical record? Primary impairment Secondary impairment Primary prevention Secondary prevention

Secondary impairment -In this case the fact that the patient has complete paraplegia would be the primary impairment. -The fact that the flexion contracture is coming from the inability to actively move the musculature because of the complete paraplegia means that the flexion contracture is a secondary impairment of the paraplegia

A therapist is observing a patient with a transtibial prosthesis and notices "drop off" during the late stance phase of gait. Which of the following prosthetic/anatomical causes would be the LEAST LIKELY cause of this gait deviation? Keel too short Socket too far posterior Insufficient plantarflexion Knee flexion contracture

Socket too far posterior An early knee flexion during the late stance phase of gait is also known as "drop off". It occurs due to the following causes: Prosthetic causes: High shoe heel, Insufficient plantar flexion, Keel too short, Dorsiflexion stop too soft, Socket too far anterior, Socket excessively flexed, Cuff tabs too posterior. Anatomical causes: Knee flexion contracture. A socket placed too far posterior would cause delayed knee flexion.

What is the typical presentation for juvenile rheumatoid arthritis?

Swollen, stiff, painful joints usually worse in the morning. Fatigue, fever, swollen lymph nodes, and poor weight gain or slow growth. If this is suspected the therapist should refer the patient to a physician.

if the vagus nerve is damaged, which way will the uvula deviate?

uvula deviates AWAY FROM (contralateral) the affected side.

Visual vs vestibular vs proprioception systems

visual: uses eye sight Vestibular: sensation comes from movements that involve twisting, spinning, rocking, turning upside down, or moving fast. Required for maintaining balance during standing. Proprioception: is our internal knowledge of where our body parts are.

MCP and IP joint dysfunctions

Trigger finger: rupture of the extensors at the distal phalanx. no MCP flexion. Boutonnieres: Rupture of central tendon. extension of MCP and DIP and flexion of PIP Swan neck: flexion of MCP, flexion of the distal interphalangeal and extension of the PIP. A patient with a swan neck deformity would experience a contracture of the intrinsic muscles (MCP flex) along with dorsal subluxation of lateral extensor tendons (ext of PIP and flex of DIP) Dupuytrens: banding at the palm as well as digit flexion contractures. looks like skin crossing MCP joint is contracted.

Physical therapist directs a physical therapist assistant to measure the seat depth for a patient as part of the assessment for the appropriate fit of a wheelchair. How many inches should the therapist subtract from the standard measurement taken for wheelchair depth? One Two Four Six

Two -see depth is determined by measuring from the patient's posterior buttock, along the lateral side to the popliteal fold, then subtracting approximately 2 inches to avoid pressure from the front edge of the seat against the popliteal space. -normal seat depth in an adult size wheelchair 16 inches

You are assessing a patient with a CVA who presents with right sided body weakness, inability to understand commands, weakness of the R half of the face and homonymous hemianopsia. Which of the following will MOST LIKELY be absent in this patient? Topographic disorientation Lack of fluency while talking Loss of sensation on the right side of body Limb-kinetic apraxia

Topographic disorientation The symptoms of weakness of R body and face, inability to understand commands (also known as Wernicke's Aphasia), and homonymous hemianopsia are suggestive of an L MCA infarct. With an L MCA infarct, Broca's aphasia, limb-kinetic apraxia and loss of sensation of the right UE and face are commonly seen. Topographic disorientation is usually seen with lesions involving non-dominant primary visual areas.

Patient receive surgical discharge instructions that include the use of an incentive spirometer and lower extremity compression stockings. Which of the following surgical procedures would be the most likely to necessitate the use of this type of medical equipment? Achilles tendon repair Lateral ankle ligament reconstruction Knee meniscectomy Total knee arthroplasty

Total knee arthroplasty -incentive spirometry is used to encourage the patient to take long, slow, deep inhalations. It may speed recovery and reduce the incidence of postoperative complication such as pneumonia. -Off-the-shelf compression garments or antiembolism stockings are used to prevent deep vein thrombosis and patience on bed rest and provide support to lower extremity venous circulation. Achilles tendon repair is typically an outpatient surgical procedure with a decrease risk of re-injury Lateral ankle ligament is the same as above -Knee meniscectomy is the removal of all or part of a torn meniscus and the patient is usually weight-bearing as tolerated immediately after surgery. Describe the patient would use the stockings an incentive spirometer after a TKA due to the magnitude of the surgery.

Physical therapist applies addressing to an area of skin on a patient's heel. The therapist decided to use the dressing as a prophylactic measure to reduce the risk of skin breakdown in an area that was determined to be particularly susceptible. Which of the following dressings with the therapist have most likely used? Calcium alginate Hydrocolloid Hydrogel Transparent film

Transparent film -transparent film dressings consist of thin membranes coated with a layer of acrylic adhesive. Since the film is transparent and allows for frequent assessment of the wound and offer some level of protection. They are oxygen permeable but not permeable to micro organisms and moisture. -calcium alginate dressings or Highly absorptive and typically utilize with wounds that produce moderate to heavy exudate. A wound with minimal exudate is unlikely to saturate the alginate to the extent necessary for it to form a beneficial hydrophilic gel. -hydrocolloid dressings consist of gel forming polymers backed by a strong film or film adhesive. The dressings absorb exudate by swelling into a gel like mass and vary in permeability, thickness, and transparency. Hydrocolloid dressing is not used for superficial wound, more often used on partial and full thickness wounds. -hydrogel dressings are moisture retentive primary dressings that provide a moist environment for wound healing. They are not normally used as a prophylactic measure and is instead used to prevent the wound from dehydrating and impeding the healing process.

Pathophysiological causes of type one versus type two diabetes

Type one diabetes results from loss of pancreatic beta cell function and an absolute insulin deficiency. Type two diabetes results from impaired ability of the tissues to use insulin (insulin resistance) accompanied by a relative lack of insulin or impaired release of insulin.

Documenting the care provided to the patient is essential and must be completed in a timely manner. Which of the following is NOT appropriate with respect to documentation? -When a charting error is made, use white-out material to correct the text -When a charting error is made, it must be clearly indicated that a change was made without deleting the original record -Medically approved symbols or abbreviations can be used for documentation -Mistakes must be crossed out with a single line through the error, and then both initialed and dated by the therapist

When a charting error is made, use white-out material to correct the text Documentation should be consistent with the Guidelines for PT Documentations, APTA. According to the Guidelines, white-out material must never be used to correct text in a medical record. All other statements are true.

myasthenia gravis

a chronic autoimmune disease that affects the neuromuscular junction and produces serious weakness of voluntary muscles. Loss of function with many cranial nerves and progresses as day goes on with fatigue. Bilateral ptosis of the eyelids, difficulty chewing, dysphasia, and bladed razee eyebrows, upper extremity weakness, double vision.

Lymphedema vs Lipedema

lymph: unilateral, distal, cellulitis, stemmer sign liped: bilateral, proximal, pain on pressure, easy bruising of area, NO stemmer

ACSM guidelines for obese patients

moderate (40% to <60% of heart rate reserve/ HRR). Frequency should be ≥ 5 days to maximize caloric expenditure and duration should be a minimum of 30 minutes per day progressing to 60 min per day of moderate intensity, aerobic activity.

ACSM guidelines for obese/metabolic syndrome patients Same for patients recovering from cancer or undergoing chemo

moderate intensity (40-60% mvo2) for at least 5 days a week or more. In addition to this, initial exercise is recommended for 30 minutes to get the optimum cardiovascular and health benefits.

A patient with ESRD comes to clinic. The PT monitors their vitals, and will expect which value to be MOST affected based on the patient's diagnosis?

normal or decreased DBP. Patients with end stage renal diseases have decreased Diastolic BP. Increase in BP is a cause for renal failure

pulmonary fibrosis sxs and look on radiograph

on radiograph as a reticular or net like pattern due to destruction and fibrosis of the lung tissue. dry hacking cough, fatigue, muscle weakness, shortness of breath, and weight loss due to loss of appetite

Normal ABG values

pH = 7.35-7.45 PaCO2 = 35-45 HCO3 = 22-26

Supraspinatus painful arc

painful arc between 70 degrees and 120 degrees of shoulder abduction

Multiple sclerosis types

relapsing remitting: attacks with bouts of plateau (no symptoms) primary progressive: straight progression of disease - no attacks secondary progressive: start with attacks and progression, then straight progression with no attacks progressive relapsing: attacks with the time in between being progression of disease, no plateaus

A physical therapist treats a patient who has a C6 spinal cord injury. Which of the following muscles would not be innervated based on the patient's level of injury? Biceps Deltoid Triceps Diaphragm

Triceps this patient would not possess motor, sensory or reflex function below the C6 level. So any muscle or structure innervated below this level would not be active. -The triceps muscle is innervated by the radial nerve or C7 - C6. -Biceps are innervated by the musculocutaneous nerve C5-C6 -The deltoid muscles innervated by the axiliary nerve C5-C6 -Diaphragm is innervated by the phrenic nerve C3-C5.

EMG exam wave types

-Complex repetitive discharges are characterized by an extended train of potentials with the same or nearly the same waveform. They will present with regular and repetitive waveforms. -Positive sharp waves are characterized by waves that are typically biphasic with a sharp initial positive deflection (below baseline) followed by a slow negative phase. -Polyphasic Potentials are seen during voluntary contraction not by rest! They are classified as motor unit potentials with five or more phases. -F waves are seen with nerve conduction velocity testing and allows one to study the proximal nerve segments. The wave is elicited by the supramaximal stimulation of a peripheral nerve at a distal site.

What are the capsular restrictions of the hip?

Hip flexion, abduction, internal rotation.

A PT is assessing a patient with a chief complaint of low back pain and a diagnosis of L2-L3 nerve compression. Which of the following gait deviations at the hip and pelvis are NOT to be expected in this patient? -Trunk lurches backward and toward the unaffected stance leg from heel off to mid swing -Posterior tilt of the pelvis during initial swing -Semicircle movement of the hip during swing—combining hip flexion, hip abduction and forward rotation of the pelvis -Excessive hip flexion at initial and mid swing

Excessive hip flexion at initial and mid swing L2-L3 nerve compression leads to weak hip flexors. Hence excessive hip flexion will not be seen at the initial and mid-swing phase of the gait cycle. Options A: Hip flexion is passively generated by a backward movement of the trunk Option B: Abdominals are used during initial swing to advance the swing leg Option C: Semicircular movement combining hip flexion, hip abduction, and forward rotation of the pelvis to compensate for hip flexor weakness.

A physical therapist treats a patient diagnosed with lateral epicondylitis. The patient exhibits pain which limits resisted testing and tenderness with soft tissue palpation that is consistent with a diagnosis. Which muscle is most likely to exhibit the described findings? Extensor carpi radialis brevis Flexor carpi radialis Flexor carpi ulnaris Brachioradialis

Extensor carpi radialis brevis This muscle has a tendinous attachment at the lateral epicondyle and is most commonly affected by microscopic repetitive trauma. Lateral epicondylitis tends to half pain with resisted wrist extension motions and passive wrist flexion motions.

A physical therapist evaluate a patient who is six months pregnant and reports right knee pain. The therapist would like to refer the patient for further diagnostic imaging before starting treatment, but is concerned for the risk of radiation exposure to the developing fetus. Which of the following imaging techniques will have the greatest risk of radiation exposure? X-ray Fluoroscopy MRI Ultrasound

Fluoroscopy -An x-ray uses very low doses of radiation which should be safe for a pregnant patient, especially when the beams are directed at the knee and not the abdomen. -fluoroscopy uses x-ray imaging with injection of a contrast dye, instead of a single x-ray image, the x-ray beam is passed through the body continuously to allow for the visualization of movement. Because the x-ray beam is continuous the level of radiation exposure is much higher. -MRI utilizes magnetic fields and radio waves and does not use radiation. -ultrasound utilizes sound waves and does not use radiation.

A child is seen in physical therapy after being diagnosed with the right thoracic scoliotic curve. Which of the following clinical features would be most likely given the patient's diagnosis? Increased space between the right arm and trunk Left rib hump with a Forward bend test Excessive anterior tilt of the pelvis Increase elevation of the right shoulder

Increase elevation of the right shoulder -a patient with a right thoracic scoliosis curve would have the convexity of the curve facing to the right. If there was a rib hump it would be on the right side with the forward bend test. -scoliosis is a lateral curvature of the spine, occurring in the frontal plane. So it and access it anterior tilt would not normally be seen. -there would be decreased space between the right arm in the trunk due to this patient laterally flexing towards the right

Physical therapist treat a patient rehabilitating from a lower extremity injury with a cryo-therapeutic agent. Which finding is most accurate when using this type of intervention? Increased spasticity Increased pain threshold Increased nerve conduction velocity Increased metabolic rate

Increased pain threshold -cryotherapy increases the pain threshold by reducing sensation of pain. Possible factors contributing to this phenomenon include reduce sensory nerve conduction velocity, reduce muscle spasm, and counter irritation via the gate control mechanism -Cryotherapy will temporary decrease spasticity not increase it. -cryotherapy will decrease the nerve conduction velocity of both sensory and motor nerves not increase it. -cryotherapy decreases metabolic reactions including those involved in the inflammatory process, not increase them.

A Physical Therapist is treating a patient with a weak tibialis anterior. The Physical Therapist has decided to utilize Functional electrical stimulation to improve ambulation. In which phase of the gait cycle should FES be applied? Mid stance to Terminal Stance Terminal Stance to Initial Contact Loading response to Pre-Swing Initial Swing to Mid Swing

Initial Swing to Mid Swing Ground clearance occurs during swing phase from initial swing to mid swing.

A patient with chronic lower extremity edema is being seen by physical therapist for recommendations on compression therapy. The patient reports that they are very sanitary, only walking short distances in home a few times per day. Which form of compression therapy would be the most beneficial for reducing the patient edema? Long stretch bandages Short stretch bandages Antiembolism stockings Low stretch bandages

Long stretch bandages -compression bandages apply both resting pressure and working pressure. Resting pressure is the pressure exerted by the bandage when the patient is at rest, while working pressure is the pressure exerted by the bandage as the contracting muscles push against it. -Long stretch bandages are characterized by high levels of extensibility that provide their greatest pressure when the patient is at rest. Because these bandages have significant stretch, they do not provide large amounts of working pressure. They describe the patient is sanitary and would therefore need a form of compression therapy that provides high resting pressure. -antiembolism stockings provide low levels of compression and are primarily used to prevent formation of deep vein thrombosis and immobile patients. They do not provide enough compression to reduce or control edema. -low stretch bandages are synonymous for short stretch bandages. These have high working pressure and low resting pressure

Describe the differences between nociceptive pain, neuropathic pain and central sensitization

Nociceptive pain: having evidence of actual or threatened tissue damage in the periphery Neuropathic pain: lesion or disease in a somatosensory system causing the pain i.e. radiating pain, numbness, or tingling in a dermatomal or specific nerve distribution. Central sensitization: increased excitatory and decreased inhibitory neural signaling in the central nervous system resulting in hypersensitivity. Can occur when there is no evidence of nociceptive or neuropathic pain reasoning.

A patient status post knee surgery receives instructions on the use of a continuous passive motion machine. Which of the following would be the most essential to ensure patient safety? Instructions on progression of range of motion Utilization of proximal and distal stabilization straps Recommendations for cryotherapy following treatment sessions Orientation to remote on/off switch

Orientation to remote on/off switch Orienting the patient to the on and off switch allowed the patient to terminate the treatment immediately without direct assistance from a healthcare provider if something were to go wrong.

prostate/bladder issues

Prostatitis: more common in men over the age of 45. back pain and burning with urination as well as a fever. The patients back ROM will have no limitation and pain is not changed with altering body positions. Nephrolithiasis or kidney stones: characterized by sudden sharp severe pain that may radiate toward the testical in the male. Renal cancer: associated with painful urination however there will also be blood in the urine and flank/side pain. There will also be a lump or mass on the side or abdomen. Benign Prostatic Hyperplasia (enlarged prostate): common in men over the age of 50. Although urination becomes difficult it is not typically associated with fever.

abnormal respiratory sounds

Rhonchi: continuous and low-pitched - snoring wheeze: high pitched on exhalation with obstructive crackle: breathe in or out. brief, discontinuous, cellophane, high-pitched, CARDIO problems pleural rub: breathe in or out. inflammation of pleura

cor pulmonale is heart failure of which side of the heart

Right

Chemotherapy side effects

1. bone marrow suppression 2. nausea and vomiting (GI dysfunction) 3. altered immunologic response 4. impaired oral mucous membrane 5. stomatitis 6. fatigue 7. Increased possibility of hemorrhage following heavy resistance if the patient's platelet count drops too low. 8. anorexia 9. ulcers 10. anemia 11. thrombocytopenia 12. phlebitis 13. hair loss 14. neuropathies NOT myopathies

Physical therapist attempt to determine the appropriate back height of a wheelchair for a patient that is able to propel the wheelchair independently but has poor trunk control. The therapist records the distance from the seat of the chair to the floor of the axilla is 20 inches. What is the most appropriate back height for the patient given the presence of poor trunk control? 17 inches 16 inches 15 inches 14 inches

17 inches -a standard wheelchair has a back height from 16 to 16.5". Back height is determined by measuring from the seat of the chair to the floor of the axilla with the user's shoulder flexed to 90° and then subtracting 4 inches. A patient's physical and mental abilities and limitations must be considered when making final recommendations for wheelchair prescription -a back height of 16 inches would be the standard back height based on the obtained measurement of 20 inches for many patients (20 inches -4 inches). But this level of support would not likely be adequate for a patient with poor trunk control, so the extra inch of back height offers additional support.

If physical therapist recognizes the sine of an airway obstruction for a patient who is eight months pregnant. The patient is choking and unable to independently clear the airway, so they are responsive. What is the most appropriate therapist action to help clear the airway? Administer abdominal thrusts Administer chest thrusts Administer rescue breaths Administer finger sweeps

Administer chest thrusts -abdominal thrusts are considered the most effective method for clearing the airway of an adult who is choking. However they should be avoided in patients with late-term pregnancies, infants, and individuals that are obese. -chest thrusts are an appropriate alternative to abdominal thrusts as they eliminate the possibility of harming the fetus. The therapist would be positioned behind the patient and the therapist would place the thumb side of one fist on the sternum and grasp the fist with the other hand. The therapist should provide five Chest thrusts in rapid succession. If the patient's airway remains obstructed, the chest thrusts should be repeated. -keep breast are performed as part of cardio pulmonary resuscitation on the patient is unresponsive and not breathing. -finger sweeps are only used when a foreign object is visible

Patient presents to physical therapy with the signs and symptoms of adhesive capsulitis of the shoulder. Which of the following imaging studies would be the most appropriate to confirm the diagnosis? Fluoroscopy Myelography Arthrography Computed Tomography

Arthrography Adhesive capsulitis is a shoulder disorder characterized by inflammation and fibrotic thickening of the anterior joint capsule of the shoulder. A decrease in space within the joint capsule leads to a decrease of synovial fluid and further irritation to the glenohumeral joint. -fluoroscopy is designed to show motion within the body with use of x-ray imaging and injection of contrast dye. I can show motion within joints or movement of the die within the digestive track. This is not a likely choice to confirm adhesive capsulitis - myelography is an invasive test that combines fluoroscopy and radiography to evaluate the spinal subarachnoid space. And utilize as a contrast medium that is injected into the epidural space by spinal puncture. It is used to identify bone displacement, disc herniation, spinal cord compression or tumors. -arthrography is an invasive procedure that uses x-ray imaging and an injection contrast dye to visualize joint structures. It can assist with a diagnosis of adhesive capsulitis by detecting a decrease volume of fluid within the joint capsule. -a CT scan is a noninvasive imaging procedure in which x-ray images are taken from multiple angles using a large circular scanner. These are most commonly used to diagnose spinal lesions and a diagnostic studies of the brain

A physical therapist prepares to measure the blood pressure of a patient with suspected hypertension. Which action by the patient would be unnecessary when conducting the measurement? Avoid caffeine for 30 minutes before Sit with the arm resting on a table during measurement Sit quietly for five minutes before measurement Avoid eating for one hour before the measurement

Avoiding eating for an hour before -caffeine stimulates the central nervous system which can cause increases in blood pressure -arm should be at roughly chest height, if not it can result in Arrhenius blood pressure measurements -activity has a potential to increase blood pressure so sitting quietly for five minutes allows it to return to normal resting state

A patient diagnosed with epilepsy has a history of generalized tonic clonic seizures. Which class of pharmacological agent would most likely be prescribed to treat this type of seizure? Barbiturates Anticholinergics Cholinergic stimulants Beta adrenergic agonists

Barbiturates -epilepsy is a neurological disorder characterized by seizure activity. Tonic clonic seizures (or grand mal seizures) have a tonic phase (when the patient loses consciousness and the muscles become tense) followed by a clonic phase (where the muscles contract and relax rapidly causing convulsions). -barbiturates function to decrease neuronal activity in the brain by increasing the inhibitory effects of GABA. There are used in the treatment of seizures but especially in tonic clonic seizures. -anticholinergics are used to treat certain gastrointestinal disorders such as peptic ulcer disease or IBS. So they are also used sometimes with Parkinson's disease, cardiac arrhythmias, motion sickness, urinary frequency and incontinence and bronchoconstriction -cholinergic stimulants are used to treat the decrease in smooth muscle tone in the gastrointestinal tract or urinary bladder that may occur following abdominal surgery or trauma. Other uses include Alzheimer's disease, glaucoma, and myasthenia gravis. -beta adrenergic agonists are used to treat conditions that involve bronchoconstriction. These medications cost smooth muscle relaxation within the bronchioles resulting in bronchodilation

Broca's vs. Wernicke's Aphasia

Broca (expressive) is a non-fluent aphasia. These patients understand, may be able to write or speak somewhat, but not well. Can read fine. Wernicke's (receptive) aphasia is failure to comprehend language, while still being able to speak fluently. Difficulty reading and writing. Broca's area is the left frontal region. Wernicke's is the left temporal/parietal. Brocas supplied by superior MCA division Wernickes supplied by inferior MCA division

A physical therapist prepares to auscultate the lungs as part of an examination. Which of the following breath sounds for the therapist most expect to hear when auscultating over the larger airways in the anterior chest at the second and third intercostal spaces in a healthy lung? Vesicular Bronchial Ronchi Tracheal

Bronchial -vesicular breath sounds are soft, high pitch, breezy sounds that are normally heard over the distal airways and healthy lung tissue. The inspiratory phase is longer than the expiratory phase and there is no pause between them. -bronchial sounds are more tubular, hollow sounding, and louder than vesicular sounds and are normally heard over the large airways in the anterior chest near the second and third intercostal spaces. The inspiratory phase is shorter than the expiratory phase and there's a slight pause between them -rhonchi are continuous, low pitched, abnormal breath sounds described as having a snoring or gurgling quality that may be heard during both inspiration and expiration. Rhonchi are typically caused by air passing through an airway that is obstructed by inflammatory secretions or liquid, bronchial spasm or neoplasms in a smaller or larger airways. -tracheal breath sounds are loud, tubular sounds normally her directly over the trachea. Tracheal breath sounds are more harsh than bronchial sounds since the sound is like air being blown through a pipe, and this case the wind pipe i.e. the trachea

capsular and non-capsular patterns

CAPSULAR: shoulder: EAR hip: FABIR knee: lose flexiom elbow: lose flexion ankle: lose PF 2 ROM joints: lose more flexion NON-CAPSULAR: lost due to muscle tightness, etc

different cathodes (-) and anodes (+) for iontophoresis

CATHODES: Acetic acid: Ca+ deposits Chlorine Iodine: sclerotic scars Dexamethasone: MSK inflammation Salicylate: analgesic ANODES: Zinc: dermal ulcers Copper: fungal infections Calcium: muscle spasm Magnesium: muscle spasm Lidocaine: analgesic Xylocaine: analgesic Hyaluronidase: edema reduction

Rancho Los Amigos Scale

I. No Response: Patient appears to be in a deep sleep and is unresponsive to stimuli. II. Generalized Response: Patient reacts inconsistently and nonpurposefully to stimuli in a nonspecific manner. Reflexes are limited and often the same, regardless of stimuli presented. III. Localized Response: Patient responses are specific but inconsistent, and are directly related to the type of stimulus presented, such as turning head toward a sound or focusing on a presented object. He may follow simple commands in an inconsistent and delayed manner. IV. Confused-Agitated: Pt in heightened state of activity and severely confused, disoriented, and unaware of present events. Behavior frequently bizarre and inappropriate to immediate environment. Unable to perform self-care. If not physically disabled, may perform automatic motor activities such as sitting, reaching and walking as part of agitated state, but not necessarily as a purposeful act. V. Confused-Inappropriate, Non-Agitated: Pt appears alert and responds to simple commands. More complex commands, however, produce responses that are non-purposeful and random. Pt may show some agitated behavior in response to external stimuli rather than internal confusion. Pt is highly distractible and generally has difficulty in learning new information. Can manage self-care activities with assistance. Memory is impaired and verbalization is often inappropriate. VI. Confused-Appropriate: Pt shows goal-directed behavior, but relies on cueing for direction. Can relearn old skills such as activities of daily living, but memory problems interfere with new learning. Has beginning awareness of self and others. VII. Automatic-Appropriate: Pt goes through daily routine automatically, but is robot-like with appropriate behavior and minimal confusion. Has shallow recall of activities, and superficial awareness of, but lack of insight to, condition. Requires at least minimal supervision because judgment, problem solving, and planning skills are impaired. VIII. Purposeful-Appropriate: Pt alert and oriented, and is able to recall and integrate past and recent events. Can learn new activities and continue in home and living skills, though deficits in stress tolerance, judgment, abstract reasoning, social, emotional, and intellectual capacities may persist.

After a stroke, a patient's visual field is as shown in the picture. Which of the following is the MOST LIKELY location of the lesion? (can only see bilateral medial fields of vision - no peripheral on either side) L optic nerve R optic tract Optic chiasm L occipital lobe

Optic chiasm The deficit shown in the image is bitemporal hemianopsia which is caused due to a lesion in the optic chiasm. L optic nerve damage would cause monocular vision. R optic tract lesion would cause C/L homonymous hemianopsia. Lesion in the L occipital lobe would cause C/L homonymous hemianopia with macular sparing.

A physical therapist listens to the lung sounds of a patient with chronic bronchitis. The patient was admitted to the hospital two days ago after complaining of shortness of breath and difficulty breathing. While performing auscultation, the therapist identifies distinct lung sounds with a high Constant pitch during exhalation. What is this type of sound most consistent with? Crackles Rales Rhonchi Wheezes

Wheezes -these are described as high-pitched, musical sounds made by air passing through the narrowed tracheobronchial airways -Crackles are discontinuous due to fluid accumulation in the distal airways or when collapsed alveoli reopened during inspiration -Rales are synonymous with crackles -Rhonchi or lower pitched, though continuous, but they occur during inspiration or expiration period caused by turbulence of air passing through secretions in bronchi

by what age do those with DMD normally require power-assisted mobility equipment?

age 14

What is Legg-Calve-Perthes disease?

avascular necrosis of femoral head presents in children 2 - 13 years of age with insidious onset hip or knee pain and an antalgic gait. limiyed ROM in abduction and extension. Patients will move and hip external rotation, selection, and adduction along with exaggerative trunk and pelvic movements four times more likely in males than females. Often misdiagnosed as growing pains. Though, children experiencing growing pains typically present with increased pain at night and do not commonly exhibit loss of range of motion or a dysfunctional gait

Do you apply a P to A glide on the superior or inferior vertebral transverse process to induce closing?

inferior TP you would P to A glide the superior vertebrae to induce increase space or opening

If the hypoglossal nerve is damaged, which way will the tongue deviate?

tongue deviates TO (ipsilateral) the affected side with atrophy on that side

dehydration and HR in young and old

young: tachycardia Old: bradycardia

at what age is a child able to receive spinal surgery for scoliosis

~10-12 and older

Hyperglycemia vs. Hypoglycemia

HYPOGLYCEMIA: Feeling shaky. Being nervous or anxious. Sweating, chills and clamminess, pallor Irritability or impatience. Confusion. Fast heartbeat/ palpitations. Feeling lightheaded or dizzy. Hunger. blurred vision slurred speech (seems like about to pass out) HYPERGLYCEMIA: Fruity-smelling breath. Nausea and vomiting. decreased apetite. Shortness of breath. Dry mouth. increased thirst. frequent, scant urination. Weakness. Confusion. Coma. Abdominal pain. flushed. dehydration rapid breathing rapid, weak pulse diminished reflexes and senses.

A 59-year old teacher who is right hand dominant complains of stiffness in the neck and right hand (especially in the 1st CMC) upon waking in the morning. It subsides within thirty minutes as she moves around the house and performs some household chores. There is no associated history of other systemic symptoms, although she has noticed some bumps on her DIP joint. The presence of outgrowths can be documented as which of the following? Heberden's nodules Rheumatoid nodules Bouchard's nodules Neurofibromatosis

Heberden's nodules The patient's age and symptoms indicate OA. Nodules on the DIP are called Heberden's nodules and nodules on the PIP are called Bouchard nodules. Absence of systemic systems and morning stiffness lasting less than sixty minutes rule out rheumatoid arthritis, and thus rule out rheumatoid nodules. With neurofibromatosis, the nodules would not be restricted to the DIP joints.

A patient status post CVA exhibit significant weakness in the affected lower extremity. When strengthening the affected abductor muscle group using irradiation, which muscle group on the unaffected lower extremity with the physical therapist utilized resist a maximal isometric contraction? Hip flexor muscles Hip extensor muscles Hip abductor muscles Hip lateral rotator muscles

Hip abductor muscles -the radiation techniques following a CVA may be utilized to increase activity in the weekend musculature. This is typically facilitated by providing maximal resistance to the contralateral muscle group to recruit active movement of the weaker musculature. -resisting the unaffected hip abductor muscles will typically increase muscle tension and the contralateral, or affected, hip abductor muscles.

A physical therapist asked the patient several questions prior to administering cervical mechanical traction. During the questioning, the patient indicates they are wearing dentures and that the dentures are securely in place. What is the most appropriate therapist action? Initiate traction using normal treatment parameters with dentures in place Initiate traction using normal treatment parameters after removing the dentures Initiate traction using 50% of the normal recommended force with the dentures in place Avoid using traction since the intervention is contraindicated for the patient

Initiate traction while using normal treatment parameters with the dentures in place -a patient who wears dentures should be instructed to keep the dentures in place during cervical traction. At the dentures are removed the alignment of the TMJ may be altered, causing problems if pressure is applied through the mandible. -the presence of dentures is not a contra indication for cervical traction, just a precaution.

Patient with lateral epicondylitis reports pain in the lateral elbow, as well as paresthesias that radiate into the forearm and hand. Based on the diagnosis and location of the patient symptoms, which of the following nerves is most likely affected? Median Musculocutaneous Ulnar Radial

Radial -Lateral epicondylitis is characterized by inflammation or degenerative changes at the common extensor tendon that attaches to the lateral condyle of the elbow. It has pain along the lateral aspect of the elbow sometimes radiates into the dorsum of the hand and affects the extensor muscles of the forearm. -the radial nerve runs most posteriorly in the upper arm after branching off the posterior cord of the brachial plexus. As it crosses the elbow joint, it runs close to the lateral epicondyle and then branches off into smaller nerves that run along the posterior forearm and dorsum of the hand, innervating the extensor muscles of the forearm. When the radial nerve is affected, pain and or paresthesias will typically occur in the posterior forearm and dorsum of the hand. Lateral epicondylitis could lead to irritation of the radial nerve due to its proximity.

Which of the following conditions is a CONTRAINDICATION for mechanical traction of the cervical spine? 1. Muscle spasm 2. Rheumatoid arthritis 3. Hypomobility of the spine 4. Decreased upper extremity sensation

Rheumatoid arthritis Joint capsules, ligaments, and bones are fragile in patients with rheumatoid arthritis (RA). A patient with RA is subject to atlantoaxial subluxation or to developing instability next to areas of hypomobility

What conditions are the abnormal heart sounds S3 and S4 associated with, respectively? S1 and S2 normal sounds

S3 = CHF; (insufficient L ventrical - can't push blood out to body, so CHF) S4 =MI or HTN, L ventricular hypertrophy or increased pressure, pulmonary HTN and stenosis S1 represents closure of mitral and tricuspid valves S2 represents closure of both aortic and pulmonary valves.

A physical therapist works with a patient with a C7 tetraplegia on transferring from a manual wheelchair to a mat table with a sliding board. According to the head hips relationship principal, which direction should the patient move their head to initiate the transfer? Down and away from the mat Up and towards the mat Down and towards the mat Up and away from the mat

Down and away from the mat -the head hips principal is a compensatory movement strategy used to perform transfers and bed mobility in patients with complete and incomplete spinal cord injuries. -the patient utilizes momentum through voluntary head movement in one direction to produce movement of the buttocks in the opposite direction A patient with a C7 tetraplegia Woodlock the elbows using muscle substitution or stabilize using the triceps. The patient will then lean forward onto the bilateral upper extremities, while tucking the chin into the chest. The patient quickly and forcefully moves the head and shoulders down and away from the destinations surface in order to lift and move the buttocks to perform a transfer in the direction of the destinations.

A physical therapist inspects the progress of a partial thickness wound on a patient's anterior forearm. The therapist notes evidence of resurfacing of the wound with notable changes in the edges of the wound. This observation is most consistent with which of the following conditions? Maceration Granulation Epithelialization Infection

Epithelialization -A partial thickness wound extends through the epidermidis and possibly into, but not through, the dermis. Examples include abrasions, blisters, and skin tears. A partial thickness wound will typically heal by re-epithelialization or epidermal resurfacing depending on the depth of the injury. -epithelialization refers to the process of epidermal resurfacing and appears as a pink or red skin. This process is a function of keratinocytes, which make up the layers of the dermis epidermidis as well as the linings of various body organs. -maceration refers to a softening of connective tissue fibers due to excessive moisture. The result is a loss of pigmentation around a wound that is highly susceptible to break down or enlargement. -granulation refers to perfused, fibrous connective tissue that replaces a fibrin clot in a healing wound. The tissue is highly vascular and feels the defects of full thickness wounds. -an infection may produce pass, redness, pain, and swelling. Other signs and symptoms of infection include fever, chills, and an increased pulse rate. Lab values may also showing increased every throw site sedimentation rate and white blood cell count.

A patient who has ankylosing spondylitis is referred to physical therapy for instruction in home exercise program. Strengthening of which of the following muscles would be the most beneficial for the patient? Rectus abdominous Internal and external obliques Quadratus lumborum Erector spinae

Erector spinae ankylosing spondylitis is a form of systemic rheumatic arthritis that results in inflammation of the axial skeleton with subsequent back pain. This condition is associated with an increase in thoracic kyphosis and loss of the lumbar curve. The patient often develops at forward stooped posture observed in standing. Extension exercises are often an important component of a comprehensive plan of care to assist patient with ankylosing spondylitis to maintain the normal curves of the spine while the same time limiting the forward bending nature of the disease process.

A patient recently diagnosed with a deep venous thrombophlebitis is placed on heparin. What is the primary side effect associated with heparin? Hypotension Depression Excessive Anticoagulation Thrombocytopenia

Excessive anticoagulation -Heparin will delay or prevent blood coagulation or clotting. Thus, increasing risk of bleeding. Heparin is used to treat things like pulmonary embolism. Heparin prevents the conversion of prothrombin to thrombin and prevents the release of thromboplastin from platelets. -thrombocytopenia is an abnormal decrease in the number of blood platelets and is not a primary side effect of heparin

what chronic hormonal responses to exercise may contribute to menstrual dysfunction

High levels of cortisol, low levels of follicle-stimulating hormone (FSH)

For arm exercise as compared with leg exercise, at a given workload, what kind of therapist expect?

Higher heart rate and systolic/diastolic blood pressure. Armor, trees is a smaller muscle mass than leg ergometry with resultant lower maximal oxygen uptake. Enough extremity exercise, both heart rate and blood pressure will be higher than for the same level of work in the lower extremities. Because the lower extremities have more muscle to do the work.

A physical therapist analyzes the gate of child with spastic diplegia cerebral palsy in the school setting. Based on this diagnosis, which muscle group is most likely to be shortened? Hip adductors Knee extensors Hip abductors Ankle dorsiflexors

Hip adductors -spastic diplegia is an upper motor neuron lesion in the motor cortex of the cerebrum mainly affecting the trunk in the lower extremities. -the gait pattern for this diagnosis produces a scissoring pattern, involving excessive hip flexion, abduction and internal rotation. -knee flexors would be shortened not knee extensors. -plantar flexors would be shortened not dorsiflexors

A physical therapist assigns a grade of good after performing a manual muscle test for the patient and prone. What is the most likely muscle group associated with the describe testing procedure? Hip abductors Hip adductors Hip internal rotators Hip extensors

Hip extensors -no need to explain this one

Physical therapist examines normative values for lung volumes prior to performing pulmonary function testing on a patient. Which of the following lung volumes would typically have the greatest value? Tidal volume Inspiratory reserve volume Expiratory reserve volume Residual volume

Inspiratory reserve volume -these are all forms of lung volumes and capacities. On average healthy adult male has a total lung capacity of approximately 6000 mL while a female has a total lung capacity of 4200 mL -tidal volume is the amount of air inspired and expired with each breath during quiet breathing and it accounts for approximately 10% of total lung volume. TV for males is typically 600 mL while 500 for female -IRV is the maximal volume of air that can be inspired after normal tidal inspiration and it accounts for approximately 50% of total lung volume. IV for males is typically 3000 mL while for females it is 1900 mL -ERV is the maximum volume of air that can be exhaled after a normal tidal exhalation and it accounts for approximately 15% of total lung volume. ERV for males typically 1200 mL while it is 800 for females -residual volume is the volume of gas remaining in the lungs at the end of a maximal expiration and it accounts for approximately 25% of total lung volume. RV for males is typically 1200 mL while 1000 for females

When checking vital signs of a patient with a blood disorder, the PT notices that the patient's BP is around 150/85 mm Hg. Which of the following conditions is MOST likely associated with the observed BP changes? Anemia Thrombocytopenia Leukopenia Polycythemia

Polycythemia Polycythemia is characterized by increases in both the number of red blood cells and the concentration of hemoglobin. People with polycythemia have increased whole blood viscosity and increased blood volume which can cause elevated blood pressure -Anemia is a reduction in the oxygen-carrying capacity of the blood as a result of an abnormality in the quantity or quality of erythrocytes. -Thrombocytopenia is decrease in platelet levels and -Leukopenia is decrease in WBC levels. -Options A, B&C do not cause an increase in SBP.

Physical therapist considers an ankle foot orthosis for a patient who has difficulty with toe clearance during ambulation due to left dorsiflexion weakness. Which objective findings would make a posterior leafspring orthosis and inappropriate orthotic selection? Poor plantar flexion strength Mild spasticity Good knee stability and strength Right leg 1 cm longer

Poor plantar flexion strength -a Post your leafspring orthosis is the most common ankle foot orthosis. It was a narrow calf shell Enero ankle with a trim line behind the malleoli. This orthosis resists plantar flexion at heel strike and during swing phase, that's preventing foot slapping and toe dragging. It is indicated for weakness of the ankle dorsiflexors. -a patient with poor plantar flexion strength would not be a candidate for a PLS orthosis because they would require adequate plantar flexion strength to overcome the resistance of the orthosis during toe off. So a solid AFO would be more appropriate for this patient.

A physical therapist measures a patient's shoulder complex medial rotation in standard position. The therapist records the patient's shoulder medial rotation is 0-70° in classifies the end feel as firm. Which portion of the joint capsule is primarily responsible for the firm and feel? Anterior Posterior Inferior Superior

Posterior -Think of the arthrokinematic's of the ball and socket of the glenohumeral joint. The humeral head rolls anteriorly but glides posteriorly with medial rotation. So this would mean that a firm end feel with medial rotation is due to the humerus gliding into the posterior aspect of the joint. -a firm end feel cause by the anterior joint capsule would most likely happen with lateral rotation -a firm end feel caused by the inferior joint capsule would most often be associated with flexion and abduction -a firm end feel caused by the superior joint capsule would most often be associated with extension and adduction

A young male comes to the clinic complaining of significant pain in his thoracic spine. Upon palpation, the therapist finds tenderness at the right T6 inferior facet joint which is exacerbated with right rotation. On examination, the therapist notes closing restriction at right T6-T7 facet joint. Which of the following mobilization technique would be MOST beneficial? Posterior to anterior mobilizing force on the right transverse process of T5 Posterior to anterior mobilizing force on the left transverse process of T6 Posterior to anterior mobilizing force on the right transverse process of T7 Posterior to anterior mobilizing force on the right transverse process of T6

Posterior to anterior mobilizing force on the right transverse process of T7 Posterior to anterior mobilizing force on the right transverse process of T7 To promote closing, posterior-anterior glide is given on the transverse process of the lower vertebra on the affected side. So, to improve closing restriction between right T6-T7, posterior to anterior mobilizing force on the right transverse process of T7 will be most beneficial.

A patient was referred to physical therapy after a removal of the right upper extremity cast which extended to the forearm. Upon examination, the patient presented with decreased internal rotation of the forearm at the proximal radio ulnar joint. Which of the following mobilization technique is likely to improve internal rotation of the forearm? Postero-lateral glide and anterior roll of radius Antero-lateral glide and posterior roll of radius Postero-lateral glide of radius and anterior roll of ulna Antero-lateral glide of radius and posterior roll of ulna

Postero-lateral glide and anterior roll of radius At the proximal radioulnar joint, convex rim of the radial head articulates with the concave radial notch on the ulna. In pronation (internal rotation of forearm), convex rim of the radial head slides posteriorly on the radial notch, opposite to the bone motion i.e. anterior

Physical therapist treat a patient that has been diagnosed with a restrictive lung disease. Which of the following conditions could be considered an extra pulmonary cause for restrictive lung disease? Scoliosis Sarcoidosis Tuberculosis Chronic bronchitis

Scoliosis -scoliosis is the only extrapulmonary etiology of restrictive lung disease mentioned. The rest are all of potential pulmonary Etiology. -sarcoidosis is a systematic disease is characterized by granulomatous inflammation throughout the body. The lungs and thoracic lymph nodes are the most often involved. These granuloma normally progressed to fibrosis. -tuberculosis is an infection and inflammatory systemic disease that results in fibrosis within the lungs. -chronic bronchitis is an obstructive lung disease in which there is an increase in total lung capacity, residual volume, and functional residual capacity. As well as significantly reduced expiratory flow rates.

A physical therapy student is studying about falls and objective measures to assess balance and mobility. Which of the following scores represents a higher risk of falls in the elderly? Score of 18 seconds on Timed Up and Go Test Score of 18 on Performance-Oriented Mobility Assessment Score of 49 on Berg Balance scale Score of 25 on Functional Gait Assessment

Score of 18 on Performance-Oriented Mobility Assessment -POMA: < 19 = high fall risk -Timed Up and Go Test: Scores > 30 seconds are indicative of impaired functional mobility. -Berg's Balance Scale: Score of < 45 indicates a greater risk of falling -Functional Gait Assessment: Scores of <22/30 are effective in predicting falls in community-dwelling older adults. -Functional reach less than 10 = fall risk

A physical therapist works with a patient with hemiparesis who uses a hemi plegic chair for mobility. Which activity would become more challenging for the patient based on the specific type of wheelchair? Reaching for objects outside of the base of support Standing up from the seat of the chair Performing independent pressure relief Elevating the legs for edema management

Standing up from the seat of the chair -a hemiplegic chair incorporates a seat that has approximately 2 inches lower than a standard chair to enable to use or to use the lower extremities to propel the chair. The patient typically uses one hand ram and one or both feet to help propel and steer the wheelchair. -Standing up from the seat of the chair would be more challenging the hemi plegia chair since the lower seat would require significantly more upper and lower extremity strength to stand -Performing independent pressure relief, reaching for objects outside of the base of support, and elevating the leg rests all may be challenging for the patient depending on the extent of involvement, however a hemiplegic chair would not increase the complexity of these tasks.

Incontinence types

Stress Incontinence Stress incontinence may happen when there is an increase in abdominal pressure -- such as when you exercise, laugh, sneeze, or cough. Urine leaks due to weakened pelvic floor muscles and tissues. (so strengthen them) Urge Incontinence Urge incontinence is often referred to as overactive bladder: You have an urgent need to go to the bathroom and may not get there in time, leaking urine. Causes of overactive bladder include: Damage to the bladder's nerves; Damage to the nervous system; Damage to muscles Conditions such as multiple sclerosis, Parkinson's disease, diabetes, and stroke can affect nerves, leading to urge incontinence. Other conditions such as bladder infections, bladder stones, and use of certain medications can also contribute to symptoms. (biofeedback to detrusor muscle) Overflow Incontinence You may have overflow incontinence if you are not able to empty your bladder appropriately. As a result, you may have leakage once the bladder is already full. This is more common in men with symptoms of frequent dribbling of urine. Causes of overflow incontinence include: Weak bladder muscles; Blockage of the urethra, such as by prostate enlargement; Medical conditions such as tumors causing obstruction of urine flow; Constipation Functional Incontinence With functional incontinence, physical problems such as arthritis, or cognitive problems such as dementia prevent you from getting to the bathroom in time. (declutter path to bathroom)

An elderly patient who has moderate osteoarthritis of the knees has been referred to physical therapy for an exercise program. Which of the following would be the most appropriate activity to help maintain healthy articular cartilage? Walking program Plyometric exercises Jogging on the treadmill Lunges

Walking program -osteoarthritis is a degenerative chronic disorder resulting from the biomechanical breakdown of articular cartilage in the synovial joints. The process is usually a result of excessive loading of a healthy joint or normal loading of an abnormal joint. -moderate pain free exercise on a regular basis is typically the most desirable activity for a patient with osteoarthritis. Nourishment of articular cartilage is facilitated by the milking action of articular surface deformation during intermittent loading. -plyometric exercises and jogging on the treadmill would both likely be too provocative for this patient. -lunges can normally be affective however might be too aggressive for an elderly patient with OA.

A physical therapist works with a four month old infant. During mat activities the infant suddenly becomes unconscious. Which of the following arteries is the most appropriate for the therapist to palpate to assess the infants pulse? Radial Brachial Popliteal Carotid

Brachial The radio and carotid arteries are the most commonly assessed arteries in the adult patient, the brachial artery is the most appropriate artery to assess in the infant. The artery can be easily palpated on the medial aspect of the midshaft humerus and therefore provides a physical therapist with a timely and accurate method to assess the patient's pulse. -the carotid artery lies inferior to the angle of the mandible and anterior to the sternocleidomastoid muscle. The infants typical stature, small and chubby neck, make locating the carotid artery difficult especially in an emergent situation. -the popliteal artery is often difficult to palpate in general.

Tropia vs. Phoria

-tropia- eye is always misaligned -phoria - eye is intermittently misaligned when eyes arent looking at same thing test: cover-uncover

A patient with a genetically inherited neuromuscular disorder reports weakness in her hands and feet with normal strength elsewhere. The patient experiences difficulty with grasping objects and demonstrates of steppage gait pattern. The clinical presentation is most consistent with which of the following medical conditions? Guillain-Barré syndrome Charcot-Marie-tooth disease Post polio syndrome Huntington's disease

Charcot Marie tooth disease -Guillan-barre syndrome has various clinical presentations, the most common subtype is characterized by weakness that begins distally and rapidly progresses to involve more proximal musculature. Eye muscle weakness, difficulty swallowing, pins and needles of hands/feet, unsteadiness in gait) GPS is not a genetically inherited condition. The exact ideology is not known, though its onset has been associated with bacterial and viral infections, surgery, and vaccinations -Charcot Marie tooth disease is hereditary disorder characterized by motor and sensory neuropathy, progressive muscle wasting, and diminished deep tendon reflexes. Disease initially affects ankle dorsiflexors and evertors resulting in foot drop, and thus a compensatory steppage gait pattern. As CMT progresses, involves the intrinsic musculature of the hands leading to difficulty with fine motor tasks. -Post polio syndrome refers to new neuromuscular symptoms that occur years after recovery from the initial episode of poliomyelitis. The presentation of weakness can vary, but is usually focal and asymmetric. Just a musculature is not necessarily affected more than proximal musculature. Additionally, PPS occurs as the result of a viral infection and therefore is not hereditary disease. -Huntington disease is a genetically transmitted neurological disorder of the central nervous system. It is characterized by degeneration and atrophy of the basal ganglia and cerebral cortex. It presents with involuntary choreic movements and a mild alteration of personality. As the disease progresses, gait will become a toxic and a patient experiences choreoathetoid movement of extremities and trunk. Speech deterioration, unintentional facial expressions and mental deterioration are common.

A patient has an echocardiogram following a myocardial infarction. The echocardiogram shows damage to the posterolateral wall of the left ventricle. Which artery was most likely included based on the involved area of the heart? Right coronary artery Left subclavian artery Left anterior descending artery Circumflex artery

Circumflex artery -The right coronary artery supplies blood primarily to the right side of the heart. The example mentions damage to the left side. The left subclavian artery branches off the aorta and supplies blood primarily to the left arm, not back to the heart. The left anterior descending artery is a branch of the left coronary artery. It supplies blood to the anterior portion of the left ventricle. The example is asking about the posterior portion. -the circumflex artery is a branch of the left coronary artery that supplies blood to the lateral portion of the left ventricle both anterior and posterior.

motor learning stages

Cognitive: novice learner. patient develops an understanding of the task. decrease stimuli/distractions. don't change environment Associated: established the movement sequence and is able to refine his / her strategies. does not need to rely so heavily on external instructions and is better able to respond to the context / environment Autonomous: enhance performance so that it becomes automatic. internalised the skill and is able to perform it with minimal cognitive monitoring.

A physical therapist treats a patient that is seven months pregnant. The therapist positions the patient in a hook line position and ask the patient to raise their head from the plinth. The therapist notices a bulge in the central abdominal area. Which medical condition is most consistent with a describe scenario? DeQuerveins disease Diastasis recti Thoracic outlet syndrome Piriformis syndrome

Diastasis recti -this is a separation of the rectus abdominis muscle along the linea alba that can occur during pregnancy. Testing for this should be performed at all pregnant women prior to prescribing exercises that require the use of the of abdominals. It is diagnosed if there's a separation greater than the width of two fingers when the woman lifts her head and shoulders off the plinth. If observed, focused abdominal activities and rotational exercises should be avoided until healing has reduced to the separation. -dequerveins tenosynovitis is an inflammatory process involving the tendons of the abductor pollicis longus and extensor pollicis brevis at the thumb -thoracic outlet syndrome is a disorder that presents with symptoms secondary to neurovascular compression of fibers of the brachial plexus. -piriformis syndrome is persistent and severe radiating low back and buttock pain spanning from the sacrum to the hip and posterior thigh. Though, this could be normally observed during pregnancy due to the piriformis potentially shortening or spasming due to postural changes and hip lateral rotation with gait. Primary symptom is sciatic pain due to nerve entrapment as the sciatic nerve passes under or through the piriformis muscle.

A physical therapist notes the tendency of a patient to become dizzy and lightheaded with any positional change after starting a new medication. Which of the following medication would most likely produce the described side effects? Furosemide (Lasix) Clopidogrel (Plavix) Warfarin (Coumadin) Simvastatin (Zocor)

Furosemide (Lasix) -Furosemide is a diuretic agent. Diuretics increase the formation and excretion of urine and can be used as antihypertensive medication since they decrease the volume of fluid within the vascular system. Orthostatic hypotension may occur as a side effect due to the fluid depletion produced by these medication's. -clopidogrel is an anti-thrombotic agent which inhibits platelet aggregation in clot formation. Side effects include hemorrhage, thrombocytopenia, potential liver toxicity with the use of aspirin, and gastrointestinal distress. -warfare in is an anticoagulant agent. They inhibit platelet aggregation and thrombus formation side effects are the same as clopidogrel -simvastatin is an anti-hyperlipidemia agent known as a statin. Statins inhibit enzyme action in cholesterol synthesis, write down low density Lipo proteins, decrease triglyceride levels, and increase high density lipoprotein levels. Side effects include headache, G.I. distress, myalgia, and skin rash.

What would an ECG look like for a patient with hypokalemia, hypercalcemia, hyperkalemia, hypocalcemia?

Hypokalemia presents with a flattened or inverted T wave and a U wave. (hypokalemia can come from excessive peeing - diuretics) Hypercalcemia presents with a widened QRS segment and shortened QT interval. Hyperkalemia shows a widened QRS segment with a flattened P-wave and a peaked T waves. Hypocalcemia presents with a prolonged QT interval. hypers both have widened QRS segments. calcemias have changed in QT intervals (hyper is shortened and hypo is prolonged)

What would be some findings in a patient with upper cervical spine instability?

Intermittent and variable symptoms that include dizziness, arm heaviness, occasional headaches, and limited range of motion in all planes. -Plain radiographs in a CT scan would have ruled out a fracture.

precautions after ACL reconstruction

NO: -knees moving past toes in squatting -CKC 60-90 degrees -OKC ~45 degrees flexion to full extension -weights at ankle - move them above tibia

A physical therapist prepares a patient status post CVA with global aphasia for discharge from a rehabilitation hospital. The patient will be returning home with her husband and daughter. What is the most appropriate form of education to facilitate a safe discharge? Perform hands-on training sessions with the patient and family members Videotape the patient performing transfers and activities of daily living Provide written instructions on all activities of daily living and functional tasks Meet with family members to discuss the patient present status and abilities

Perform hands-on training sessions with the patient and family members This is the only option in which the patient can safely go home with her family members and ensuring that the family will understand how to aid this patient at home safely.

Physicians order in the medical record indicates that a patient has been placed on hold from physical therapy due to a suspected pulmonary embolism. Which diagnostic test would be the most valid method to confirm a suspected pulmonary embolism? MRI Pulmonary angiography Bronchoscopy Ultrasonography

Pulmonary angiography -a pulmonary embolism is an embolism that blocks a pulmonary artery. They normally result from Venus thrombi that have detached and traveled from elsewhere in the body before lodging in a pulmonary artery. SxS include dyspnea, coughing, hypoxia, and chest pain. -Pulmonary angiography is the most conclusive and accurate means of identifying a pulmonary embolism since it is a test of how blood flows through the lungs. -an MRI could be ordered to rule out the presence of additional thrombi, especially in the lower extremities but it isn't the most conclusive for the lungs. -bronchoscopy has direct visualization of the bronchial tree. It transmits an image to an eyepiece or video camera and can identify tumors, bronchitis, foreign bodies and bleeding. not used to identify pulmonary embolism. -ultrasonography uses sound waves to produce images of structures within the body. Doppler ultrasound is used for detection of deep vein thrombosis (legs) but not pulmonary embolism in the lungs.

A patient sustained a fracture of the acetabulum that was treated with an open reduction and internal fixation. The injury occurred in a motor vehicle accident approximately seven weeks ago. Which objective measure would be the most influential variable in determining the patient's weight-bearing status? Visual analogue pain scale Radiographic confirmation of bone healing Lower extremity manual muscle testing Balance and coordination assessment

Radiographic confirmation of bone healing The primary determinant of weight-bearing status following a fractures based on the relative stability of the fracture. Amount of time since the injury should allow for bone healing to be visible using diagnostic imaging.

A patient presents to the clinic four weeks after a reverse arthroplasty of the right shoulder. Which of the following movements would you perform while treating the patient at this stage? Shoulder extension beyond 0 degree Shoulder ER beyond 20 degree Shoulder IR up to 30 degree Shoulder elevation in scapular plane up to 90 degree

Shoulder elevation in scapular plane up to 90 degree Phase 1 of rehabilitation post reverse shoulder arthroplasty lasts for up to 6 weeks. The motions that are restricted during this period include:- No GH extension or internal rotation past neutral- No combined GH extension, adduction, and internal rotation- 0°-20° external rotation- Up to 90°-120° arm elevation in scapular plane

What is the most effective position for a pregnant patient in the third trimester to rest in at night?

Sidelying on the left. lying on the right side will cause compression of internal organs as well as the aorta thereby reducing cardiac output.

Brunnstrom Stages of stroke Recovery

Stage #1: No voluntary movement - flaccid Stage #2: Weak associated reactions, Spasticity developing. Minimal or no active joint movement. Stage #3: Basic limb synergies. Spasticity increased, possibly marked. Peak spasticity. Stage #4: Begins to deviate from synergy. Spasticity decreases. Stage #5: Partial independence from synergy Stage #6: Isolated joint movement, but speed impaired. Stage #7: Full, normal movement.

Which medications would you take for high cholesterol?

Statins. These run the risk of development of myopathy.

central vs peripheral vestibular issues

CENTRAL: CVA, cerebellum (abnormal smooth pursuit and saccades) PERIPHERAL: BPPV and UVH (head thrust - vestibular neuritis, labrynthitis, acoustic neuroma, meniers)

American College of Rheumatology classes

Class 1 is defined as a patient who is completely able to perform usual activities of daily living (self-care, vocational and avocational). Class 2 is defined as able to perform usual self-care and vocational activities but limited in avocational activities. Class 3 is defined as able to perform usual self-care activities, but limited in vocational and avocational activities. Class 4 is limited in ability to perform usual self-care, vocational and avocational activities.

Gall stones versus pancreatitis

Classic symptoms of gallstones including nausea and right upper quadrant pain, which can radiate to the right scapula. They are more commonly found in patients who are overweight or obese, female, and around the fourth decade of life. Commonly found in patients eating meals high in fat and cholesterol. Pancreatitis can have findings that include epigastric pain that radiates to the back, nausea, vomiting, diarrhea, fever and sweating, tachycardia, malaise, bluish discoloration of the abdomen and jaundice.

A patient reports to physical therapist that they completely tore one of the ligaments in her ankle. Which injury classification is most accurate based on the patient's subjective statement? Grade 1 sprain Grade 3 sprain Grade 1 strain Grade 3 strain

Grade 3 sprain -ligamentous injuries are termed sprains -injuries involving a muscle or tendon are termed strains A first-degree sprain involves a stretch of the ligament where there is no discontinuity of the ligament Third-degree sprain involves a complete rupture or break in the continuity of a ligament. -same grading system applies for strains in muscles and tendons.

grade 1 vs grade 4 manual oscillations

Grade I, i.e. small-amplitude rhythmic oscillations performed at the beginning of the range is primarily used for treating joints limited by pain or muscle guarding. Grade IV, i.e. small-amplitude rhythmic oscillations performed at the limit of the available motion and stressed into the tissue resistance is used as stretching maneuver to increase range of motion.

Can you continue exercise with first degree AV block?

Yes

skin conditions

-Cafe-au-lait the patient would have light brown macules on the skin. It is a benign skin condition. -Eczema is characterized by skin that is red, brown or gray that is sore, itchy and sometimes swollen. The skin will have blisters and weeping sores. -Vitiligo patients will have a lack of pigmentation from melanocyte destruction. It can be hereditary or caused by conditions such as hyperthyroidism and diabetes mellitus. Lesions can occur anywhere in the body but tend to be present in skin exposed areas. -Xanthomas are benign fatty fibrous yellow plaques, nodules or tumors that develop in the subcutaneous layer of the skin, especially around tendons. They are seen in disorders that affect lipid metabolism as well as in uncontrolled diabetes -Pustules are puss filled eruptions. -Vesicles filled with serous fluid and small in size . -blisters filled with serous fluid and large in size. -wheals a red, swollen mark left on flesh by a blow or pressure (welt), not fluid filled. -Herpes zoster: pain and tingling over a localized area on the posterior trunk, fever and chills, red papules over a small area in a dermatomal distribution. -Herpes simplex: vesicular eruptions on face and mouth and is caused by a virus as well

scleroderma vs warts vs lupus vs psoriasis vs rosacea

-Scleroderma is characterized by inflammation and fibrosis of many parts of the body, including the skin, blood vessels, synovium, skeletal muscle, and certain internal organs such as kidneys, lungs, heart, and GI tract. -Warts are benign infection caused by human papilloma virus seen on skin especially hands, fingers, pressure points of feet. -Psoriasis is an autoimmune disease of skin characterized by silvery scale plaques causing itching, dryness over knees, scalp, elbows, genitals etc. -Lupus is chronic autoimmune disorder affecting skin, joints, kidney, and nervous system. It has characteristic butterfly rash across bridge of nose. no migraines. -Rosacea is chronic facial skin disorder, it causes redness of the cheeks, nose, chin, forehead that comes and goes. With advanced Rosacea swelling of the nose (rhinophyma) is seen. Bumps and pimples appear. Chronic facial flushing causes the telangiectasia (spider veins). Migraines have found to be associated with Rosacea.

A patient presents with difficulty comprehending language, impaired problem-solving, and agitation. The family reports the patient has become progressively more forgetful and agitated over the past several years. There is no evidence of infection or change in medication. Which of the following medical conditions is MOST consistent with this scenario? Alzheimer's disease Delirium Mild cognitive impairment Normal age related cognitive impairment

Alzheimer's disease Difficulty comprehending language, impaired problem-solving, behavioral disturbances, and memory deficits are all commonly associated with dementia. Alzheimer's disease, progressive form of dementia, and sound largely in older adults and usually advances of the individual ages. -Alzheimer's disease is a progressive neurodegenerative disorder that results in deterioration and irreversible damage within the cerebral cortex and sub cortical areas of the brain -delirium is a form of transient cognitive and behavioral impairment that subsides once a contributing factor is removed. The contributing factor could be an infection or medication related -mild cognitive impairment is defined as having lower than expected cognitive performance when compared to others within the age group. It is not progressive -normal age related changes in cognition do not involve problem-solving deficits, behavioral disturbances, or significant impact on memory. These are more commonly associated with Alzheimer's disease or dementia

aerobic vs anaerobic exercise

Anaerobic exercises are high intensity short duration exercises which do not use oxygen to produce energy. They have greater effect in improving resting metabolism or burning calories at rest. (increase lean muscle mass/strength gains) Aerobic exercises are lower intensity and long duration exercises utilizing oxygen, which has a greater effect on cardiovascular health with more efficient blood circulation by improving myocardial strength, endurance, glucose tolerance, enhancing immunity and promote weight loss. (helps the heart, so increased SV and CO, increases weight loss due to increased glucose tolerance, improves endurance and decreases resting hr)

A patient presents with a small, round, partial thickness wound on the lateral malleolus with distinct wound edges. Upon examination, the wound bed is a gray color without evidence of granulation tissue and appears to be dry. The wound is extremely tender to palpation and the patient reports calf pain during walking. These findings are most associated with which of the following types of ulcers? Venous stasis Neuropathic Arterial Decubitus

Arterial -arterial ulcers results from arterial insufficiency secondary to inadequate circulation of oxygenated blood (ischemia). Arterial ulcers are often small, round, with distinct wound margins and the wound bed typically lacks granulation tissue due to the lack of blood supply. Pain is often severe and these ulcers are typically located on the distal limb and foot, especially the toes, the dorsal surface of the foot, or the lateral malleolus. Pain is not alleviated with elevation of foot due to this making even less blood get to the limb. -venous stasis ulcers result from Venus insufficiency resulting in an adequate venous return and eventual tissue damage and ulceration. venous ulcers typically have a shallow and irregular shape, have moderate to heavy exudate, and are usually located on the lower limb proximal to the medial malleolus. Involved lower limb often has edema with trophic changes like a dark purple or rusty color (hemosiderin staining). Pain can be alleviated by elevation due to helping clear blood from limb. -neuropathic ulcers are commonly associated with a combination of ischemia and neuropathy. Commonly associated with diabetes. Neuropathic ulcers are most commonly located on the plantar aspect of the foot and areas susceptible to pressure or shear forces during weight-bearing. These are normally not painful due to a loss of protective sensation. -decubitus ulcers are synonymous with pressure ulcers and a result of sustained or prolonged pressure on tissue at levels greater than that of capillary pressure. Susceptible areas include skin covering bony prominences. This normally would not have pain with walking.

A patient post cardiac surgery is monitored using an arterial line. The primary purpose of an arterial line is to monitor which of the following measures? Right atrial pressure Heart rate and oxygen saturation Pulmonary artery pressure Blood pressure

Blood pressure -An arterial line is inserted directly into an artery and is used to continuously monitor blood pressure or to obtain blood samples. -Right atrial pressure is measured by a balloon tipped catheter advanced from a major vein into the pulmonary artery also called a Swan-Ganz. -An arterial line CAN be used as an access point for sampling arterial blood in which oxygen saturation can be determined, however the arterial line does not monitor heart rate -Pulmonary artery pressure is also measured by Swan-Ganz catheter or other pulmonary artery catheter

Physical therapist reviews the medical record of a patient admitted to the hospital with suspected renal involvement. Which laboratory test would be the most useful to assess the patient's present renal function? Platelet count Hemoglobin Blood urea nitrogen Hematocrit

Blood urea nitrogen -blood urea nitrogen is a common measure used to assess renal function. The normal blood urea nitrogen level for adults is 10 to 20 mg/dL -BUN is used to assess kidney function. And increased BUN level can be indicative of dehydration, renal failure or heart failure. A decreased BUN level can be indicative of malnourishment, hepatic failure or pregnancy -platelet count identifies the number of platelets present in the whole blood. If the play levels high, it indicates increased risk of thrombosis and if the level is low, indicates increased risk of bruising and bleeding -hemoglobin is the iron containing pigment in red blood cells that function securely auction in the blood. Low hemoglobin may indicate anemia or blood loss. Elevated hemoglobin suggests polycythemia (rare blood disorder in which there is an increase in all blood cells, particularly red blood cells. This makes blood thicker) or dehydration -hematocrit measures the percentage of red blood cells and a volume of blood. Hematocrit may be decreased with anemia, nutritional deficiency, and leukemia. Hematocrit may be increase with dehydration, polycythemia, and burns

A patient two days following cesarean delivery complains of incisional pain with coughing and sneezing. What is the most appropriate initial physical therapy intervention? Brace incision with a pillow when cough or sneeze Contract with all the floor muscles Perform friction massage around the incision Initiate postural awareness activities

Brace the incision with pillow when cough or sneeze -C-section is considered major abdominal surgery. Bracing the abdominal incision with a pillow when coughing or sneezing or support the healing tissues by limiting the abdominal pressure associated with the activity. This can provide an appropriate and immediate method of improving overall comfort. -contract in the pelvic floor muscles may help limit pain from pelvic floor dysfunction, but unlikely to relieve abdominal incision pain. -friction massage may help limit future pain by minimizing and heat information however it will not specifically influence the patient's immediate complaints, friction massage can begin once incision is sufficiently healed. -postural correction may assist in preventing future pain due to pregnancy related faulty posture and muscle imbalance, but it is not going to specifically influence the patient's immediate complaint.

A physical therapist works with a patient with an obstructive lung disease characterized by permanent widening and distortion of the medium size airways. The patient presents with cough, copious amounts of purulent sputum, hemoptysis, and recurrent pulmonary infections. The signs and symptoms are most characteristic of which of the following pulmonary conditions? Bronchiectasis Idiopathic pulmonary fibrosis Pulmonary edema Emphysema

Bronchiectasis -Bronchiectasis is a condition characterized by permanent, abnormal dilation and distortion of the bronchi and bronchioles that is caused by destruction of the muscular and elastic components of the bronchial walls. Common clinical presentation for this disease includes cough, copious mucopurulent sputum, fetid breath, recurrent pulmonary infections, and recurrent hemoptysis -idiopathic pulmonary fibrosis is a condition characterized by inflammation and the resulting destruction of the alveolar walls. The destruction of the long causes decreased lung compliance, therefore, IPF is a restrictive lung disease. The clinical presentation of RLD includes decreased chest mobility, decreased breath sounds, shortness of breath, hypoxemia, tachypnea, and effective cough, and increased use of accessory muscles. -Edema is a condition characterized by excess fluid within the lungs. This often occurs in the left ventricles unable to adequately pump blood to the systemic circulation (left-sided heart failure). Common clinical presentation of pulmonary edema includes extreme shortness of breath, wheezing or gasping, anxiety, a cough that produces frothy sputum, chest pain, and palpitations. Acute pulmonary edema is considered a medical emergency. -Emphysema is a form of chronic obstructive pulmonary disease characterized by the abnormal and permanent enlargement of the distal airways not the bronchioles. Symptoms of emphysema worsen with the progression of the disease and include a persistent cough, wheezing, difficulty breathing especially with expiration, and increased respiration rate.

A physical therapist reviewed the medical record of a patient diagnosed with a lower motor neuron disorder. Which finding would be least likely based on the presented information? Diminished deep tendon reflexes Clonus Hypotonia Fasciculations

Clonus -lower motor neuron disorders result from lesions that affect the anterior horn cell and/or peripheral nerve. -Upper motor neuron disorders result from lesions that affect motor pathways in the spinal cord and/or brain. -Diminish deep tendon reflexes (hyporeflexia), hypotonia and fasciculations are all characteristic of lower motor neuron disorders. -Fasciculations are random and spontaneous twitches in a muscle group which may be visible through the skin. -Clonus is characterized by cyclical, spasmodic, alternation of muscular contraction and relaxation in response to a sustained stretch of a spastic muscle.

Physical therapist reviewed the medical record of a patient who sustained a spinal cord injury. A note recently entered by the physician indicates that the patient contracted a respiratory infection. Which type of spinal cord injury would be the most susceptible to this condition? Complete C4 tetraplegia Cauda equina lesion Brown-sequard's syndrome Posterior cord syndrome

Complete C4 tetraplegia -the primary muscle of respiration, the diaphragm, is innervated by C3 - C5. This would be impaired in a C for spinal cord injury so the patient will be unable to voluntarily or effectively ventilate. The patient will exhibit limited ability to clear secretions, impaired chest mobility, and alveolar hypoventilation. -cauda equina lesion occurs below the L1 spinal level where the long nerve roots transcend. These injuries are frequently incompletes due to the large number of nerve roots in the area and are often considered to be peripheral nerve injuries. Characteristics include flaccidity, areflexia, and impairment of bowel and bladder function. -brown-Séquard syndrome is an incomplete lesion usually caused by Stabwound which produces hemi section of the spinal cord. There's paralysis and lots of vibratory and position sensor on the same side as the lesion due to damage to the corticospinal tract and dorsal columns. But lots of pain and temperature sensor on the opposite side of the lesion due to damage to the lateral spinothalamic tract. Posterior cord syndrome is extremely rare and presents with a loss of: proprioception, two point discrimination, graphesthesia (ability to recognize writing on the skin purely by the sensation of touch), and stereognosis (ability to perceive and recognize the form of an object in the absence of visual and auditory information). Patients typically present with a wide base steppage gait.

A physical therapist evaluates a three-year-old child recently diagnosed with a genetic disorder. The physical therapy plan of care includes monitoring the progression of muscle weakness, maintenance of range of motion and flexibility, and family education. Which diagnosis would most likely require this form of intervention? Down syndrome Cerebral palsy Duchenne muscular dystrophy Osteogenesis imperfecta

Duchenne muscular dystrophy -DMD presents with Progressive weakness, falling, toy walking, excessive lordosis, and pseudohypertrophy of muscle groups. Intervention should focus on maintaining range of motion and flexibility. -Down syndrome presents with developmental delay, hypotonia, laxity in the ligaments, and poor strength. It does not present with progressive weakness or loss of range of motion -cerebral palsy includes abnormal muscle tone, impaired modulation of movement, presence of abnormal reflexes, and impaired mobility. Interventions include normalization of tone, stretching, strengthening, motor learning, and acquisition of developmental milestones. It does not include progressive weakness because it is a non-progressive disorder. -osteogenesis imperfecta is an autosomal disorder of collagen synthesis that affects bone metabolism. Children normally have delayed developmental milestones secondary to ongoing fractures that result in immobilization, hypermobile joints and poorly developed muscles. This is a non-progressive disorder

A physical therapist utilizes a manual assist cough technique on a patient who has a mid thoracic spinal cord injury. When completing the technique with the patient and the supine position, which of the following locations is the most appropriate for placement of the therapists hands? Manubrium Epigastric area Xiphoid process Umbilical region

Epigastric area The degree of respiratory impairment is related to the level of the spinal cord injury as well as residual muscle function, trauma at the time of injury, and pre-morbid respiratory status. Weakness or paralysis of the abdominal muscles compromises the patient's ability to cough and expel secretions. -The manubrium is the broad, quadrangular shaped upper part of the sternum - this region is too high to provide effective pressure support for coughing. -The epigastric area is the upper central region of the abdomen, located between the costal margins and subcostal plane. Applying manual hand pressure inwards and upwards over this area can assist the patient's cough and promote airway clearance. -The xiphoid process is a small cartilaginous extension to the lower part of the sternum that is usually ossified in the adult - pressure of this region should be avoided. -The umbilical region of the area surrounding the belly button. This region is too low to provide effective pressure support coughing.

A physical therapist instructs a patient status post abdominal surgery how to use an incentive spirometer in order to prevent the occurrence of pulmonary complications. What is the minimum frequency for the patient to use the incentive spirometer when they are awake? Every 5 to 10 minutes Every one to two hours Twice per day Once per day

Every one to two hours -An incentive spirometer provides visual feedback to encourage the patient to take long, slow, deep inhalation to maximize ventilation. -Prolonging and sustaining the maximum inspiration promotes recruitment of inspiratory muscle fibers and enhances distribution of inhaled air purpose the purpose is to treat and prevent atelectasis, especially patients who do not take deep breaths because of pain after abdominal or thoracic surgery. -the recommended frequency for using the incentives barometer is 5 to 10 breaths every hour while awake

Physical therapist reviewed the medical record of a 22-year-old athlete rehabilitating from an anterior cruciate ligament reconstruction. Which piece of objective data would serve as a barrier to the patient returning to high demand competitive athletics? 0-135° knee range of motion Hamstrings/quadriceps strength ratio of 50% Quadriceps strength of 90% of the contralateral limb Negative lateral pivot shift test

Hamstrings/quadriceps strength ratio of 50% -0-135° of knee range of motion is within normal limits -hamstrings/quadriceps strength ratio should be greater than 70% prior to returning to hide a man competitive athletics. This means that the hamstring should possess a minimum of 70% of the strength of the quadriceps. Not having this ratio may result in the patient being susceptible to re-injury. -the acceptable level of strength to possess prior to returning to hide a man competitive athletics would be 85-90% quadriceps strength compared to the contralateral limb. -the lateral pivot shift test is commonly used to diagnose an ACL sprain or tear. So a negative test would be a good thing.

Physical therapist educated patient status post transfer moral amputation on the importance of frequent skin checks. What is the most appropriate resource for the patient utilized when inspecting the posterior aspect of the residual limb? Hand mirror Video camera Caregiver Prosthetist

Hand mirror this will enable the patient to view the entire line without being dependent on another person. Out of all of the options the hand mirror is the least cumbersome and the most readily available for all patients.

A physical therapist participate in a research study that examines body composition as a function of aerobic exercise and diet. Which method of data collection would provide the therapist with the most valid measurement of body composition? Anthropometric measurements Bioelectrical impedance Hydrostatic weighing Skinfold measurements

Hydrostatic weighing -body composition is defined as the relative percentage of body weight that is comprised of fat and fat free tissue. Common anthropometric measurements include height, weight, BMI, waist hip ratio, and percentage of body fat. These measurements are then compared to reference standards -Bioelectrical impedance measures body composition by using electrical current to determine the resistance or opposition to current flow through the body. This is based on the principle that resistance to electrical current is inversely related to the composition of water within the body. -Hydrostatic weighing calculates the density of the body by immersing a person in water and measuring the amount of water that becomes displaced this measurement technique is considered the criterion or gold standard for determining body composition -Skinfold measurements can be used to determine the overall percentage of body fat through the measurement of nine standardize sites. The theory is that the amount of subcutaneous fat is proportional to the total amount of body fat.

A physical therapist designed an exercise program for a patient that minimizes the use of isometric work. Which medical condition would most necessitate the need for this type of program? Anemia Diabetes Hemophilia Hypertension

Hypertension -Isometric work can result in sharp increases in blood pressure. Since those with hypertension already have an elevation and systolic blood pressure, isometric work would be contraindicated and we would want those with hypertension to emphasize low resistance and low repetition isotonic exercise -Anemia is where the number of red blood cells is reduced. So delivery of oxygen to the tissues is impaired. Symptoms of anemia include weakness and malaise which would make it challenging for patients to engage in any form of resistance training. -diabetes is in those patients with high blood glue close levels isometric work has the same inherent risks as other forms of resistance training for these patients. -Hemophilia is a bleeding disorder of genetic ideology. It is a sex linked autosomal recessive trait. These patients are prone to hemarthrosis, intramuscular hemorrhage, and secondary complications. Isometric work as the same inherent risks as other forms of resistance training for patients with hemophilia.

Physical therapist reviewed the chart prior to treating a patient in the hospital coronary care unit. A recent entry indicates that the patient is taking an angiotensin converting enzyme inhibitor. Which of the following side effects is common to ACE inhibitors and Wood most impact the therapist performing transfer training with this patient? Bradycardia Dehydration Hypokalemia Hypotension

Hypotension -ACE inhibitor agents decrease blood pressure and afterload by suppressing the enzyme that converts angiotensin one to angiotensin two. These are most often indicated to treat patients with hypertension and congestive heart failure. Common side effects include hypotension, dizziness, fatigue, dry cough, hyperkalemia, and hyponatremia -patient seeking a CE inhibitor should avoid sudden changes in posture due to the risk of dizziness and fainting from hypotension. -Bradycardia can be a potential side effect for patients taking beta blockers since the medication decreases the myocardial oxygen demand by decreasing heart rate and contractility -dehydration is a potential side effect for patients taking diuretics since the medication increases the excretion of sodium and urine, causing a reduction in plasma volume. This can lead to symptoms of electrolyte in balance. -hypokalemia refers to low levels of potassium in the blood. Potassium is an important electrolyte for nerve and muscle cell function. Symptoms of low potassium include muscle cramps, weakness or fatigue.

Patient reports brief episodes of significant dizziness when standing during the last 24 hours. The patient states that their symptoms began shortly after they began taking Lasix to treat congestive heart failure. What is the most likely rationale for the reported dizziness? Hypertension due to a decrease in blood plasma volume Hypertension due to an increase in blood plasma volume Hypotension due to decrease in blood plasma volume Hypotension due to an increase in blood plasma volume

Hypotension due to a decrease in blood plasma volume. -Lasix is a commonly prescribed diuretic. It is normally used to reduce blood pressure and symptoms of congestive heart failure. -diuretics act to reduce blood plasma volume by enhancing the excretion of water and electrolytes by the kidneys. -This increases the likelihood of orthostatic hypotension since reduced plasma volume makes it difficult for the body to quickly adapt to positional changes.

A physical therapist evaluate a pediatric patient using the modified Ashworth scale. Which condition would least benefit from the use of this tool? Hypotonia Hyperreflexia Spasticity Rigidity

Hypotonia -the modified Ashworth scale is a widely used qualitative scale for the assessment of increased tone i.e. hypertonicity and spasticity. It measures the amount of resistance to passive stretch. -hyperreflexia refers to hyper active or clonic reflexes. Most of the time patience that exhibit hyperreflexia may also experience an increase in skeletal tone -spasticity is a component of an upper motor neuron lesion which is characterized by exaggerated Phasic (tendon jerks) and tonic (spastic) stretch reflexes. -rigidity is a state of severe hypertonicity were sustained muscle contraction does not allow for any movement at a specified joint. -hypotonia is a decrease in expected tension with interesting muscle or the amount of resistance to stretch that a resting muscle possesses. The modified Ashworth scale does not evaluate diminished tone.

A physical therapist notices that a patient status post sternotomy has significantly altered their breathing pattern. The therapist is concerned that the alteration and breathing may lead to atelectasis. What is the most appropriate intervention to treat and prevent atelectasis? Active cycle of breathing Inspiratory muscle training Incentive spirometry Postural drainage

Incentive spirometry -after sternotomy, it may be painful for the patient to take a deep breath. Shallow breathing will lead to hypoventilation of the alveoli, which may cause atelectasis. Breathing exercises may be used to help prevent and/or treat atelectasis. -Active cycle of breathing technique is most commonly used to assist secretion clearance in patients with asthma. It includes three phases: breathing control, thoracic expansion exercises, and forced expiratory technique. -inspiratory muscle training uses a handheld breathing training device to provide resistance to increase the strength and endurance of the muscles of inspiration -when using incentive spirometry, a maximal inspiratory effort is held at the point of maximum inspiration before exhalation. Prolonging and sustaining the maximum inspiration promotes recruitment of inspiratory muscle fibers and the drop in intrathoracic pressure enhances distribution of inhaled air. The technique promotes air passage past mucus obstructions in airways and maximizes alveolar expansion. -postural drainage is when you position the patient so that gravity will help drain bronchial secretions from specific lung segments toward the gravity dependent central airways where they can be removed by call for mechanical aspiration

A patient is gay training with patella tendon bearing transtibial prosthesis. Upon inspection of the residual limb, the physical therapist notes slight redness over the proximal patella tendon and inferior border of the patella as well as the distal anterior residual limb. What is the most appropriate therapist action? incrdease the number of soccer ply Decrease the number of sock ply Continue gait training with frequent reassessment Discontinue gait training until prosthetic modifications are made

Increase the number of Sock ply When a patient presents with redness on the proximal patella tendon and inferior border of the patella, it is an indication that the residual limb is sitting too low within the socket. The distal anterior residual and will also present with redness since the residual limb is making contact distal to the Internet areas of relief bill into the socket. the therapist would continue gait training and reassessment of the skin if the existing areas of redness were located on the more traditional areas associated with a properly fitting prosthesis like the patella tendon, medial tibial flare, both sides of the tibial shaft, femoral condyles.

Increased myelin basic protein VS increased serum uric acid VS increased erythrocyte sedimentation rate VS increased creatine kinase.

Increased Myelin basic protein: determined using a lumbar puncture with aspiration of cerebral spinal fluid. Elevated levels are suggestive of demyelinating diseases Increased serum uric acid: seen in patients with gout and may be seen in other conditions such as diabetes, hypothyroidism, and obesity Increased creatine kinase: shown in those with rhabdomyolysis Increased erythrocyte sedimentation rate: this is a general marker of inflammation and his elevated in patients with inflammatory disorder such as rheumatoid arthritis, polymyalgia rheumatica, ankylosing spondylitis.

A seven month old infant is able to initiate rolling from supine to prone when playing on a level floor surface. Which scenario would most likely be associated with this developmental milestones? Integration of the tonic labyrinthine reflex Persistence of the asymmetrical tonic neck reflex Integration of the rooting reflex Persistence of the Gallant reflects

Integration of the tonic labyrinthine reflex -The tonic labyrinthine reflex is stimulated through the labyrinth in the inner ear. If the infant is a supine position, the body and extremities are held an extension while in a prone position, the body extremities are held in flexion. If this reflex persists, it could interfere with the ability to roll, but also the ability to prop up on elbows, balance setting, and obtain an upright posture from a supine position. Integration of the reflex is often associated with the ability to roll from supine to prone. -ATNR is stimulated through head position, specifically turning on one side. When the infant turns they had one side the arm and leg on the face side or extended while the arm and leg on the scalp side or flexed. The persistence of this reflex will interfere with rolling as well as visual tracking, midline use of hands, and feeding. -The rooting reflex is stimulated by a touch of the cheek. When the infant is stimulated the infant will turn the head toward the side of the simulation, this would have no effect on the ability to roll. -The gallant reflex is stimulated by touching the skin along the spine from the shoulder to the hip. The infants response is lateral flexion of the trunk towards the side of stimulus. Persistence of this reflex interferes with the development of sitting balance.

A patient is scheduled to undergo a transtibial amputation of the left lower extremity. In addition, the patient is one month post right total knee arthroplasty. Given the patient's past and current surgical history, the physical therapist should expect which of the following activities to be the most difficult for the patient falling amputation? Rolling from supine to sidelying Moving from supine to sitting Moving from sitting to standing Ambulating in the parallel bars

Moving from sitting to standing -Rolling from supine to sidelying and moving from supine to sitting utilize more of extremity strength than lower extremity strength -went inside the parallel bars, the patient is able to use them to provide a stable base of support, using upper extremity strength to decrease some of the demand on the lower extremities. -Transferring from sitting to standing would be the most difficult for the patient since the activity requires adequate strength and dynamic balance. Strength will be decreased in the right lower extremity after the recent total knee arthroplasty and balance will be altered due to the left transtibial amputation

A physical therapist treats a patient refer to an outpatient rehabilitation facility. During the session the therapist identifies weakness, hypertonia, and hyper reflexia. What is the most likely diagnosis? Carpal tunnel syndrome Poliomyelitis Bells palsy Multiple sclerosis

Multiple sclerosis This is an upper motor neuron disease and it includes weakness of involved muscles, hypertonicity, hyperreflexia, mild disuse atrophy, and abnormal reflexes. -Multiple sclerosis is demyelination of the myelin sheath that surround nerves within the brain and spinal cord. The decrease is the efficiency of nerve impulse transmission and symptoms may vary based on location of the myelination. There is subsequent plaque development and eventual failure of impulse transmission. -Carpal tunnel syndrome is a lower motor neuron pathology comprised of peripheral nerve entrapment when the median nerve is compressed through the carpal tunnel. -Poliomyelitis is a lower motor neuron pathology comprised of a viral infection resulting in neuropathy that includes focal and asymmetrical motor impairments. -Bell's palsy is a lower motor neuron pathology comprised of temporary unilateral facial paralysis secondary to trauma with demyelination and/or axonal degeneration of the FACIAL NERVE (would have deficits raising eyebrows, showing teeth, smiling, closing eyes tightly, and puffing the cheeks).

When treating a geriatric patient with osteoporosis, which of the following will be MOST important to include in a treatment plan to improve bone density? Treadmill with high incline Lumbar flexion resistance training Seated rowing machine Leg press resistance training

Muscle contraction (strengthening exercises, resistance training) and mechanical loading (weight bearing) deform bone which stimulates osteoblastic activity and improves BMD. Since, leg press causes weight bearing it can cause bone formation (Wolf's Law). Seated row machine will not allow weight bearing, hence not beneficial. Treadmill with high incline and lumbar flexion should be avoided in an osteoporotic patient to prevent vertebral compression fracture.

A patient is evaluated in physical therapy one week following a lateral ankle reconstruction. Which of the following weight-bearing limitations would be most appropriate for this patient? Nonweightbearing with bilateral axillary crutches Partial weight-bearing with a single axillary crutch Weight-bearing as tolerated with bilateral axillary crutches Weight-bearing as tolerated with a single axillary crutch

Nonweightbearing with bilateral axillary crutches -Lateral ankle reconstruction surgeries commonly perform secondary to a complete tear of the anterior talofibular ligament or calcaneofibular ligament or secondary to chronic ankle instability. -it is expected that the patient will be in a hard cast and nonweightbearing for at least a short period of time (two weeks or more) following the surgery. They would need to use bilateral axillary crutches in order to maintain her nonweightbearing status

Physical therapist treats a patient diagnosed with Alzheimer's disease. Which of the following would be recommended as a primary intervention for this patient population? Physical activity Stretching Coordination training Pulmonary therapy

Physical activity Alzheimer's disease is a progressive neurodegenerative disorder that results in deterioration within the brain. Amyloid plaques and neurofibrillary tangles result in further damage to the nervous system. -during the early stages of Alzheimer's a patient should continue daily physical activity. As a disease progresses, patient will rely on caregiver support process with daily exercise program. Activity and structured exercise can assist with memory as well as decrease the effects of Alzheimer's disease including restlessness and wandering. -stretching is typically most appropriate for patients that exhibit contractures or present with hypertonicity. -coordination training is most appropriate for patients that present with ataxia, nystagmus, impaired balance, and equilibrium deficits typically due to a lesion within the cerebellum. -pulmonary therapy is most appropriate for patients that present with a compromised pulmonary system and may include airway clearance, breathing techniques, assisted cough, inventory muscle training.

primary and secondary risk factors for atherosclerosis

Primary: HTN, hyperlipidemia, and cigarette smoking secondary: obesity

A physical therapist examines the output from a single lead electrocardiogram as a patient with first-degree atrioventricular heart block. What is the defining feature of this condition? Inverted T wave Prolonged PR interval Bizarre QRS complex ST segment depression

Prolonged PR interval -The PR interval is the time required to conduct the cardiac impulse from the Sino atrial node to the atrial ventricular node. Her blocks, also known as conduction blocks, or characterized as first, second or 3rd degree according to the severity of the disturbance in the conduction of the cardiac impulse through the electrical conduction system. -a prolonged PR interval is a defining feature of first-degree atrioventricular block. -the PR intervals will gradually become more prolonged with increasing degrees of AV heart block -inverted T waves are commonly associated with myocardial ischemia -a bazaar QRS complex can be characteristic of a premature ventricular contraction (PVC) -ST segment depression is also a classic sign of myocardial ischemia

A patient reports numbness and tingling in the Palmer aspect of the thumb index finger and middle fingers examination findings reveal pain with resisted pronation and negative tinel's at the wrist which of the following conditions will be the most likely suspect? Carpal tunnel syndrome Pronator syndrome Anterior interosseous syndrome Cubital tunnel syndrome

Pronator syndrome -pronator syndrome is an entrapment of the median nerve at the elbow. It is often associated with activities that require repetitive pronation while grasping an object. Paresthesias and the distribution of the median nerve would be expected with both pronator syndrome and carpal tunnel syndrome, however, the negative Tinel sign at the wrist and pain with resisted pronation or more typical of pronator syndrome. -carpal tunnel syndrome would most likely have a positive Tinel's sign at the wrist -anterior interosseous syndrome is a neuropathy resulting in an isolated palsy of the muscles innervated by this motor branch of the median nerve. The three muscles innervated by this nerve are in the deep layer of the anterior forearm and include flexor pollicis longus, index and long fingers of the flexor digitorum profundus, and pronator quadratus. The classic sign of anterior interosseous syndrome is an inability to perform the OK sign by approximating the tip of the index finger to the tip of the thumb. -Cubital tunnel is in a treatment of the owner nerve at the elbow when it passes between the medial epicondyle of the humerus and the olecranon of the ulna. This would normally have paresthesias and the ulnar nerve distribution

Physical therapist examines a patient following a Traumatic crush injury to the forearm. The patient displays her wrist drop deformity and is unable to strongly grasp objects with their hand. Which nerve most likely was effective? Median nerve Ulnar nerve Radial nerve Musculocutaneous nerve

Radial nerve -wrist drop deformity is the inability to actively extend the wrist and fingers, resulting in the wrist being flaccidly held in a flexed position. Patients with wrist drop deformity have diminished grip strength since they are unable to actively extend their wrist. -the radial nerve innervates the triceps, brachioradialis, wrist extensors, and finger extensors. Injury to this nerve often causes weakness with a wrist and finger extension, resulting in a wrist drop deformity -musculocutaneous innervates the coracobrachialis, biceps, and brachialis. Injury to this nerve would cause weakness with elbow flexion and forearm supination. -the ulnar nerve innervates the flexor carpi ulnaris, flexor digitorum profundus, third and fourth lumbricals, muscles of the hypothenar eminence, interossei, and other small muscles within the hand. Injury to this nerve often causes weakness with wrist flexion and flexion of the ulnar half of the digits, resulting in a claw hand deformity. -the median nerve innervates the forearm pronators, most of the wrist flexors, first and second lumbricals, and muscles of the thenar eminence. Injury to the median nerve often causes weakness with wrist flexion and forearm pronation

What are the signs and symptoms of pulmonary embolism

Rapid onset of dyspnea, sudden chest pain, and cyanosis.

A patient presents with a stage three pressure ulcer with a moist, necrotic wound. Hydrocolloid dressing is being used. During the dressing change, the therapist attests a strong odor, and the wound drainage has a yellow color. What is the therapists best course of action?

Reapply a new hydrocolloid dressing and record the findings in the chart. Hydrocolloid dressings are typically changed every 3 to 5 days when drainage leaks out. An odor and yellowish color is to be expected as the dressing material melts. The decision about what type of dressing to apply to a wound as the physicians in collaboration with the wound care team.

Physical therapist attempt to improve a patient's lower extremity strength. Which proprioceptive neuromuscular facilitation (PNF) technique would be most appropriate to achieve the therapists goal? Contract relax Repeated contractions Rhythmic stabilization Hold relax

Repeated contractions -repeated contractions is designed to initiate movement and promote strength while the other listed options are designed to increase range of motion or promote stability. -contract relax is used to INCREASE ROM. As the extremity reaches the point of limitation the patient performs a maximal contraction of the antagonistic muscle group. The therapist resist the movement followed by relaxation and passive movement into newly gained range of motion. (isometric of antagonist) -repeated contractions are used to sustain a contraction through the range of motion. The therapist provides a quick stretch followed by isometric or isotonic contractions. Providing resistance at the point of weakness can enhance the effectiveness of repeated contractions. -rhythmic stabilization increases range of motion. The technique requires isometric contractions of all muscles around a joint against progressive resistance. -hold relax uses isometric contractions to INCREASE ROM. The contractions are facilitated for all muscle groups at the limiting point within the range of motion. Relaxation occurs in the extremity moves through the newly acquired range to the next point of limitation. (isometric of agonist) rhythmic initiation: utilized to INITIATE MOVEMENT, reduce hypertonicity and relax the muscle Resisted progression: can help improve ENDURANCE

A patient post stroke presents with significant impairments in kinesthesia during sensory testing of the involved extremities. Based on this finding, which of the following proprioceptive neuromuscular facilitation techniques would be the most appropriate intervention? Rhythmic initiation Alternating isometrics Rhythmic stabilization Hold relax

Rhythmic initiation -kinesthesia refers to the ability to identify the direction and extent of movement of a joint or body part. -PNF techniques typically involves movement patterns that follow diagonals that possess a flexion, extension, and rotary component and are directed toward or away from midline. -rhythmic initiation is used to assist in initiating a movement pattern and helps a patient learn the movement pattern. Movement progresses from passive, to active assistive, to slightly resistive. Rhythmic initiation also observation understand the rate at which movement should occur and would be the most useful of the options to address the patient's impairments in kinesthesia -alternating isometrics are primarily used to improve isometric strength and stability. Isometric contractions are performed alternating from muscles on one side of the joint to the other side without rest. Since no joint movement occurs, this PNF technique would be more appropriate for impairments and proprioception, which is the ability to identify a static position of an extremity or body part. -rhythmic stabilization is used as a progression of alternating isometrics and is also designed to primarily promote stability. The technique requires isometric contractions of all muscles around a joint against Progressive resistance. This is often performed in weight-bearing position to incorporate joint approximation to further facilitate co-contraction. Since this primarily focuses on stability, it would be more appropriate to address impairments in proprioception. -hold relax is primarily used to facilitate stretching. An isometric contraction is used at the limiting point in the range of motion to stretch and shorten muscle. Relaxation occurs in extremities move through the early acquired range the next point of limitation until no further increases in range of motion occur. Since muscle shortening is not part of the presented scenario, this technique would not be the best option.

Physical therapist examines a patient post surgery. The patient has diabetes, however, has no other significant past medical history. Which of the following situations would most want immediate medical attention? Signs of confusion and lethargy Systolic blood pressure increase of 20 mmHg during exercise Lack of significant clinical findings following the examination Discovery of significant past medical history unknown to the physician

Signs of confusion and lethargy -confusion and lethargy in a patient with diabetes are signs of hypoglycemia, or low blood glucose. If untreated, hypoglycemia can rapidly progressed towards a life-threatening situation. -systolic blood pressure increasing 20 mmHg during exercise is normal -lacking significant clinical findings in the exam does not warrant immediate medical attention -Discovery of significant unknown medical history warrants referral to the physician for possible future examination follow up, but not immediate medical attention

A patient is seen in physical therapy to treat a foot ulcer. The physical therapist observes edema and redness in a lower leg and is concerned that the patient may have developed cellulitis. Which action would be the most beneficial to assess a therapist's hypothesis? Stand for five minutes and reassess color Sit in a chair with the legs in a dependent position and reassess color Sit with the legs in an elevated position and reassess color Perform active ankle pumps and recess color

Sit with the legs and an elevated position and reassess color -cellulitis refers to the fast spreading information that occurs as a result of a bacterial infection of the skin and connective tissues. It most commonly affects the extremities. Symptoms may include localized redness that may spread quickly, skin is warm or hot to the touch, local abscess or ulceration, tenderness to palpation, chills, fever, and Malaise. -you would not want to put the lower leg in a dependent position because this would be unlikely to change the current coloration of the leg. This is due to gravity mainly contributing to the accumulation of fluid in areas of the body farthest from the heart. Thus being unable to discern the presence of cellulitis. -sitting in a chair with the legs and a dependent position would also be unlikely to discern the presence of cellulitis. -performing active ankle palms would improve blood flow and circulation in the lower leg however it would not change the current coloration of the lower leg. -sitting with the legs in an elevated position and reassessing the color provides a therapist with the opportunity to see if the redness in the lower leg changes with positioning. Dependent rubor (the redness would resolve once the extremities placed an elevated position) is most often observed with peripheral artery disease the failure of the redness to disappear with elevation may provide additional support for cellulitis.

The physical therapist discusses the importance of a proper diet with a patient diagnosed with congestive heart failure. Which of the following substances would most likely be restricted in the patient's diet? Hi density lipoprotein Low-density lipoprotein's Sodium Triglycerides

Sodium -Patients with congestive heart failure tend to have excessive fluid retention in the pulmonary and systemic circulation. As a result a diet high in potassium as prescribed with items low in sodium due to sodium causing fluid retention. -High density Lipo proteins are the smallest Lipo proteins. They are composed of proteins, cholesterol, and a small amount of triglyceride. They play an important role in lipid metabolism by transporting cholesterol back to the liver from the cells. High levels of HDL reduce the incidence of coronary artery disease with no link to congestive heart failure. -low-density lipoprotein's are the major carriers of cholesterol and plasma. Elevated LDL is a cause of coronary artery disease but there is no association between it and congestive heart failure. -triglycerides are a combination of glycerol and fatty acids. Elevated triglycerides are not independently predictive of coronary artery disease, but are associated with known risk factors for atherosclerosis, colluding low HDL levels. There is no association between triglycerides and congestive heart failure.

A physical therapist inspects a burn obtained as a result of iontophoresis. The therapist describes the burn as an alkaline reaction. Which of the following factors would most likely be the cause of this type of adverse reaction? Sodium hydroxide forming under the cathode Hydrochloric acid forming under the anode Size of the cathode being larger than the anode An increase in the amount of space between electrodes

Sodium hydroxide forming under the cathode -iontophoresis utilizes a continuous direct current which moves ions through the bodies tissues. The direct current alters the normal pH of the skin and makes the patient susceptible to a chemical burn. -The likelihood of a burn can be decreased, however, by increasing the size of the cathode relative to the anode, decreasing the current density, and increasing the space between electrodes. -sodium hydroxide is alkalytic -the patient could have an acidic reaction from the iontophoresis treatment as a result of hydrochloric acid forming under the positive electrode (anode). But the patient in this question had an alkaline reaction, not acidic.

Physical therapist is asked to assist in administering a graded exercise test to a patient referred to a phase 2 cardiac rehabilitation program which of the following objective findings with most warrant determination of the test? Diastolic blood pressure rising to 100 mmHg Systolic blood pressure rising to 200 mmHg Multifocal premature ventricular contractions Sustained ventricular tachycardia

Sustained ventricular tachycardia -a diastolic blood pressure reading greater than 115 mmHg is considered a relative indication to terminate -systolic blood pressure greater than 250 mmHg would be considered a relative indication to terminate -the presence of multifocal PVCs could be indicative of multiple areas within the ventricles that have become irritable due to hypoxia and are producing ectopic beats. This is a relative indication to terminate -sustained ventricular tachycardia is the only absolute indication on this list for terminating the exercise. If it last longer than 30 seconds it is considered to be life-threatening.

A physical therapist asks a patient to raise one hand to indicate that they felt a Semmes Weinstein monofilament contacting their skin. Which situation would best support this type of modification to the testing procedure? The patient does not speak English The patient has ideomotor apraxia The patient has DM The patient has brocas aphasia

The patient has brocas aphasia The first three options would still allow the patient to be able to speak. in brocas aphasia, The patient has difficulty speaking

What are the downward rotators of the scapula?

lats, rhomboids, levator scapula, pectoralis minor (Little minor pirates love rum and then fall down)

abdomen/organ pains

liver pain: right upper quadrant pain and pain in the liver during exercise. The pain will be constant and not change upon waking or throughout the day. Pseudorenal pain onset is usually acute and associated with a traumatic history such as lifting a heavy object. The patient will report having lower abdominal pain that is worse at night, especially when lying on the affected side. The pain is usually absent when arising but will get progressively worse throughout the day. Common bile duct: pain in the mid-epigastric with vague discomfort. The pain is constant, mild at first and increases steadily. Gallbladder: right upper quadrant pain and possibly right shoulder pain. They will describe dull aching pain and a sense of fullness in their abdomen or epigastric.

during auscultation, are sounds between S1 and S2 considered systolic or diastolic murmurs?

systolic


Conjuntos de estudio relacionados

Construction Management 1011 Don Schneider Midterm

View Set

11th Grade Anatomy Human Skeletal System Part 2 Notes

View Set

Reconstruction & Southern Redemption

View Set

Exam "Networks and The Internet"- past Video Q's

View Set

Civil War and Reconstruction Test

View Set

communication studies- verbal and non verbal communication

View Set

Why are cells small and SA/ volume concept

View Set

Med Surg - Chapter 40 - Care of Patients with Hematologic Problems

View Set

Penny Chapter 2: Liver REVIEW QUESTIONS

View Set